Задачи по теории вероятности с решениями для вузов экзамен

Примеры задач по теории вероятности

В данном разделе размещены задачи с решениями по теории вероятностей из разных разделов, которые изучаются студентами и даже школьниками. Помимо подробных решений типовых задач на сайте есть онлайн-учебник, онлайн-калькуляторы по теории вероятностей, формулы, списки учебников и многое другое (см. ссылки ниже).

Если у вас есть трудности с решением заданий по терверу, обращайтесь, мы готовы помочь. Стоимость консультации от 70 рублей, срок от 1 дня, оформление в Word. Подробнее тут: Теория вероятностей для студентов.

Каталог решенных задач

Комбинаторика

  • Элементы комбинаторики (10 задач)
  • Формула включений-исключений (6 задач)

Случайные события

  • Классическое определение вероятности (13 задач)
  • Геометрическое определение вероятности (7 задачи)
  • Формула Бернулли (8 задач)
  • Теоремы сложения и умножения вероятностей (11 задач)
  • Формула полной вероятности и формула Байеса (12 задач)
  • Локальная и интегральная теоремы Лапласа (10 задач)
  • Отклонение частоты от вероятности (8 задач)

Случайные величины

  • Дискретная случайная величина (12 задач)
    • Табличный закон распределения (10 задач)
    • Биномиальный закон распределения (10 задач)
    • Гипергеометрический закон распределения (10 задач)
    • Геометрический закон распределения (5 задач)
    • Закон распределения Пуассона (8 задач)
  • Двумерная дискретная случайная величина (10 задач)
  • Непрерывная случайная величина (12 задач)
    • Нормальная случайная величина (11 задач)
    • Равномерная случайная величина (8 задач)
    • Показательная случайная величина (10 задач)
  • Двумерная непрерывная случайная величина (8 задач)
  • Неравенства Маркова и Чёбышева, ЗБЧ (10 задач)
  • Производящая функция (5 задач)
  • Характеристическая функция (5 задач)

Случайные функции

  • Случайные процессы (7 задач)
  • Стационарные случайные процессы (6 задач)
  • Цепи Маркова (6 задач)

Консультируем по задачам теории вероятностей

Контрольные по вузам

  • ИДЗ по ТВиМС, МЭСИ
  • Контрольная по ТВ, ВЗФЭИ (ФУ)
  • Теория вероятностей, МАИ
  • Типовой расчет по теории вероятностей, МИРЭА

Решебник по теории вероятности

Больше 10000 решенных и оформленных задач по всем темам теории вероятности в решебнике:

Полезные ссылки для изучения ТВ

  • Онлайн калькуляторы
  • Онлайн учебник
  • Формулы и таблицы
  • Решенные контрольные
  • Что такое ТВиМС
  • Сдача тестов
  • Решение на заказ
  • Онлайн помощь

Сборник задач по теории вероятностей

(с решениями)

Разработка предназначена для   учащихся  9–11  классов для подготовки к ОГЭ и ЕГЭ по математике.

УМК любой

Цель: показать решение типовых задач по данной теме, закрепить умение учащихся решать данные задачи, подготовить учеников к сдаче ОГЭ и ЕГЭ

Методические рекомендации по использованию ресурса:                    Работу можно применить:

  • при проведении урока по систематизации и закреплении знаний учащихся
  • при проведении консультаций.

Источники информации:                                                       Открытый банк ЕГЭ ФИПИ  http://fipi.ru/                                                       

Теория вероятностей

Классическое определение вероятности
Вероятностью события A называется отношение числа благоприятных для A исходов к числу всех равновозможных исходов:
Р (А) =  
где 
n — общее число равновозможных исходов, m — число исходов, благоприятствующих событию A.
Противоположные события
Событие, противоположное событию A, обозначают Ā. При проведении испытания всегда происходит ровно одно из двух противоположных событий и  
   Объединение несовместных событий
Два события A и B называют несовместными, если отсутствуют исходы, благоприятствующие одновременно как событию A, так и событию B.
Если события A и B несовместны, то вероятность их объединения равна
сумме вероятностей событий  A и B:               P(A U B) =P(A) + P(B)                          

Пересечение независимых событий
Два события A и B называют независимыми, если вероятность каждого из них
не зависит от появления или непоявления другого события.
Событие C называют пересечением событий A и B                     (пишут C = A∩B), если событие C означает, что
произошли оба события A и B.
Если события
A и B независимы, то вероятность их пересечения равна произведению вероятностей  событий A и B:
 P(A∩B) = P(A) • P(B)

Формула сложения вероятностей совместных событий:

                           P(A U B) =P(A) + P(B)   P(A∩B)

1. Из 1000 собранных на заводе телевизоров 5 штук бракованных. Эксперт проверяет один наугад выбранный телевизор из этой 1000. Найдите вероятность того, что проверяемый телевизор окажется бракованным.
 Решение.  При выборе телевизора наугад возможны 1000 исходов, событию A «выбранный телевизор — бракованный» благоприятны 5 исходов. По определению вероятности                       P(A) = 5÷1000 = 0,005. Ответ: 0,005.

2. В урне 9 красных, 6 жёлтых и 5 зелёных шаров. Из урны наугад достают один шар. Какова вероятность того, что этот шар окажется жёлтым?                                                                       Решение. Общее число исходов равно числу шаров: 9 + 6 + 5 = 20. Число исходов, благоприятствующих данному событию, равно 6. Искомая вероятность равна 6÷20 = 0,3.  Ответ: 0,3.

 3. Петя, Вика, Катя, Игорь, Антон, Полина бросили жребий — кому начинать игру. Найдите вероятность того, что начинать игру должен будет мальчик.

Решение. Вероятность события равна отношению количества благоприятных случаев к количеству всех случаев. Благоприятными случаями являются 3 случая, когда игру начинает Петя, Игорь или Антон, а количество всех случаев 6. Поэтому искомое отношение равно 3:6=0,5. Ответ: 0,5.

4. В чемпионате мира участвуют 16 команд. С помощью жребия их нужно разделить на четыре группы по четыре команды в каждой. В ящике вперемешку лежат карточки с номерами групп: 1, 1, 1, 1, 2, 2, 2, 2, 3, 3, 3, 3, 4, 4, 4, 4. Капитаны команд тянут по одной карточке. Какова вероятность того, что команда России окажется во второй группе?

Решение: Обозначим через А событие «команда России во второй группе». Тогда количество благоприятных событий m  = 4 (четыре карточки с номером 2), а общее число равновозможных событий n = 16 (16 карточек) по определению вероятности         Р= 4: 16 = 0,25. Ответ:0,25

5.  В лыжных гонках участвуют 11 спортсменов из России, 6 спортсменов из Норвегии и 3 спортсмена из Швеции. Порядок, в котором спортсмены стартуют, определяется жребием. Найдите вероятность того, что первым будет стартовать спортсмен не из России.

Решение. Всего спортсменов 11 + 6 + 3 = 20 человек. Поэтому вероятность того, что первым будет стартовать спортсмен не из России равна 9:20 = 0,45. Ответ: 0,45.

 6. На каждые 1000 электрических лампочек приходится 5 бракованных. Какова вероятность купить исправную лампочку? 

Решение. На каждые 1000 лампочек приходится 5 бракованных, всего их 1005. Вероятность купить исправную лампочку будет равна доле исправных лампочек на каждые 1005 лампочек, то есть  1000:1005=0,995.Ответ: 0,995.

7. В группе туристов 8 человек. С помощью жребия они выбирают шестерых человек, которые должны идти в село в магазин за продуктами. Какова вероятность того, что турист Д., входящий в состав группы, пойдёт  в магазин? 6 : 8=0,75.

8. В чемпионате по футболу участвуют 16 команд, которые жеребьевкой распределяются на 4 группы: A, B, C и D. Какова вероятность того, что команда России не попадает в группу A?

Решение. Каждая команда попадет в группу с вероятностью 0,25. Таким образом, вероятность того, что команда не   попадает в группу равна 1-0,25=0,75. Ответ:0,75

9. На турнир по шахматам прибыло 26 участников в том числе Коля и Толя. Для проведения жеребьевки первого тура участников случайным образом разбили на две группы по 13 человек. Найти вероятность того, что Коля и Толя попадут в разные группы.                                                                                               Решение. Всего 26 мест. Пусть Коля займет случайное место в любой группе. Останется 25 мест, из них в другой группе 13. Исходом считаем выбор места для Толи. Благоприятных исходов 13. Р=13/25 = 0,52. Ответ:0,52                                                                     

10. В классе 16 учащихся, среди них два друга —Вадим и Сергей. Учащихся случайным образом разбивают на 4 равные группы. Найдите вероятность того, что Вадим и Сергей окажутся в одной группе.                                                                    Решение. Если Сергею первому досталось некоторое место, то Олегу остаётся 15 мест. Из них 3 — в той же группе, где Сергей. Искомая вероятность равна 3/15. Ответ:0,2                                                                                                     

11. В классе 21 учащийся, среди них два друга — Вадим и Олег. Класс случайным образом разбивают на 3 равные группы. Найдите вероятность того, что Вадим и Олег окажутся в одной группе.                                                                                                        Решение. Пусть один из друзей  находится в некоторой группе. Вместе с ним в группе окажутся 6 человек из 20 оставшихся учащихся. Вероятность того, что друг  окажется среди этих 6 человек, равна 6 : 20 = 0,3.          Ответ: 0,3

12. Перед началом первого тура чемпионата по настольному теннису участников разбивают на игровые пары случайным образом с помощью жребия. Всего в чемпионате участвует 16 спортсменов, среди которых 7 участников из России, в том числе Платон Карпов. Найдите вероятность того, что в первом туре Платон Карпов будет играть с каким-либо спортсменом из России? 6:15=0,4. Ответ:0,4.                                                                       

13. Перед началом первого тура чемпионата по шашкам участников разбивают на игровые пары случайным образом с помощью жребия. Всего в чемпионате участвует 26 шашистов, среди которых 3 участника из России, в том числе Василий Лукин. Найдите вероятность того, что в первом туре Василий Лукин будет играть с каким-либо шашистом из России?   2: 25=0,08. Ответ: 0,08.

14. В классе 26 учащихся, среди них два друга —   Сергей и Андрей. Учащихся случайным образом разбивают на 2 равные группы. Найдите вероятность того, что Сергей и Андрей окажутся в одной группе. Ответ  12 : 25 = 0,48.

15. В классе 21 ученик, среди них 2 друга – Тоша и Гоша. На уроке физкультуры класс случайным образом разбивают на 3 равные группы. Найдите вероятность того, что Тоша и Гоша попали в одну группу. Ответ 6 : 20 = 0,3.

16. В классе 21 учащийся, среди них две подруги — Аня и Нина. Класс случайным образом делят на семь групп, по 3 человека в каждой. Найдите вероятность того, что Аня и Нина окажутся в одной группе.             Ответ: 2: 20 = 0,1.                                                     

17. Механические часы с двенадцатичасовым циферблатом в какой-то момент сломались и перестали идти. Найдите вероятность того, что часовая стрелка остановилась, достигнув отметки 7, но не дойдя до отметки 1.                                                      Ответ. 6 : 12= 0,5 ( 6 делений между 12 и 7, всего 12 делений)

18. Механические часы с двенадцатичасовым циферблатом в какой-то момент сломались и перестали ходить. Найдите вероятность того, что часовая стрелка застыла, достигнув отметки 6, но не дойдя до отметки 9 часов. 3:12 = 0,25

 При решении задач с монетами число всех возможных исходов можно посчитать по формуле  п=2ª, где α –количество бросков

19.   В случайном эксперименте симметричную монету бросают 2 раза. Найдите вероятность того, что орел выпадет ровно 1 раз.

Решение. Всего возможны четыре исхода: решка-решка, решка-орёл, орёл-решка, орёл-орёл. Орёл выпадает ровно один раз в двух случаях, поэтому вероятность того, что орёл выпадет ровно один раз равна  2:4=0,5.  Ответ: 0,5.

20. В случайном эксперименте симметричную монету бросают дважды. Найдите вероятность того, что орёл не выпадет ни разу. Ответ: 1:4=0,25

21. В случайном эксперименте симметричную монету бросают трижды. Найдите вероятность того, что орёл не выпадет ни разу.  Решение. 1:8=0,125  Ответ. 0,125                                                                                    

22. В случайном эксперименте симметричную монету бросают четырежды. Найдите вероятность того, что орёл выпадет ровно 2 раза.                                                                           Решение. Составим список возможных вариантов. Бросают 2 раза может выпасть О — Орел, Р — Решка:
ОО, ОР, РО, РР. Всего 4 исхода из них только один случай удовлетворяет условию.   Вероятность (P) = 1 / 4 = 0.25
.   Ответ: 0.25

23. В случайном эксперименте симметричную монету бросают четырежды. Найдите вероятность того, что решка не выпадет ни разу.                                                                            Решение. Всего исходов   = 16, благоприятных  1 ( ОООО).  1:16 = 0,0625. Ответ: 0,0625                                                                                     

При решении задач с кубиками число всех возможных исходов можно посчитать по формуле  п=6ª, где α –количество бросков 

24. Определите вероятность того, что при бросании игрального кубика (правильной кости) выпадет нечетное число очков.                                                                                                          Решение. При бросании кубика равновозможных  шесть различных исходов. Событию «выпадет нечётное число очков» удовлетворяют три случая: когда на кубике выпадает 1, 3 или 5 очков. Поэтому вероятность того, что на кубике выпадет нечётное число очков равна 3:6=0,5. Ответ: 0,5.

25.  Определите вероятность того, что при бросании кубика выпало число очков, не большее 3.

Решение. При бросании кубика равновозможны шесть различных исходов. Событию «выпадет не больше трёх очков» удовлетворяют три случая: когда на кубике выпадает 1, 2, или 3 очка. Поэтому вероятность того, что на кубике выпадет не больше трёх очков равна  3:6=0,5 Ответ: 0,5.                                                                  

26. Игральную кость бросают дважды. Найдите вероятность того, что оба раза выпало число, большее 3.

Решение. При бросании кубика 6²= 36 различных исходов. Событию «выпадет больше трёх очков» удовлетворяют три случая: когда на кубике выпадает 4, 5, или 6 очков , благоприятных исходов 9 (4,4; 4,5; 4,6; 5,4; 5,5; 5,6;  6,4; 6,5; 6,6.)                       Ответ: 9: 36 = 0,25.

27. В случайном эксперименте бросают три игральные кости. Найдите вероятность того, что в сумме выпадет 7 очков. Результат округлите до сотых.                                      Решение. При бросании кубика 6³= 216  различных исходов, благоприятных  14.  14 : 216 = 0,07.  Ответ: 0,07.     

28.   Коля выбирает трехзначное число. Найдите вероятность того, что оно делится на 5.

Решение. Всего трехзначных чисел 900. На пять делится каждое пятое их них, то есть таких чисел 900:5=180. Вероятность того, что Коля выбрал трехзначное число, делящееся на 5, определяется отношением количества трехзначных чисел, делящихся на 5, ко всему количеству трехзначных чисел: 180:900=0,2.  Ответ: 0,2.

29.Для экзамена подготовили билеты с номерами от 1 до 50. Какова вероятность того, что наугад взятый учеником билет имеет однозначный номер?

Решение. Всего было подготовлено 50 билетов. Среди них 9 были однозначными. Таким образом, вероятность того, что наугад взятый учеником билет имеет однозначный номер равна 9:50=0,18. Ответ: 0,18.

30. В мешке содержатся жетоны с номерами от 5 до 54 включительно. Какова вероятность, того, что извлеченный наугад из мешка жетон содержит двузначное число?

Решение. Всего в мешке жетонов — 50. Среди них 45 имеют двузначный номер. Таким образом, вероятность, того, что извлеченный наугад из мешка жетон содержит двузначное число равна  45 : 50 = 0,9.  Ответ: 0.9. 

31. Какова вероятность того, что случайно выбранное натуральное число от 10 до 19 делится на 3?                                           3 : 10 = 0,3.  Ответ: 0,3.

Противоположные события.

32.  Вероятность того, что новая шариковая ручка пишет плохо (или не пишет), равна 0,19. Покупатель в магазине выбирает одну такую ручку. Найдите вероятность того, что эта ручка пишет хорошо.

Решение. Вероятность того, что ручка пишет хорошо, равна 1 − 0,19 = 0,81.  Ответ: 0,81.

  33. Вероятность того, что в случайный момент времени температура тела здорового человека окажется ниже 36,8°C     равна 0,87. Найдите вероятность того, что в случайный момент времени у здорового человека температура тела окажется 36,8°C или выше.  Ответ. 1-0,87=0,13                                    

34. При изготовлении подшипников диаметром 67 мм вероятность того, что диаметр будет отличаться от заданного не больше, чем на 0,01 мм, равна 0,965. Найдите вероятность того, что случайный подшипник будет иметь диаметр меньше чем 66,99 мм или больше чем 67,01 мм.

Решение. По условию, диаметр подшипника будет лежать в пределах от 66,99 до 67,01 мм с вероятностью 0,965. Поэтому искомая вероятность противоположного события равна 1 − 0,965 = 0,035. Ответ: 0,035.

Несовместные и независимые события.                                                 35. На экзамене по геометрии школьнику достаётся одна задача из сборника. Вероятность того, что эта задача по теме «Углы», равна 0,1. Вероятность того, что это окажется задача по теме «Параллелограмм», равна 0,6. В сборнике нет задач, которые одновременно относятся к этим двум темам. Найдите вероятность того, что на экзамене школьнику достанется задача по одной из этих двух тем.                                          Решение. Суммарная вероятность несовместных событий равна сумме вероятностей этих событий: P=0,6+ 0,1 = 0,7.  Ответ: 0,7.

36. Вероятность того, что на тесте по биологии учащийся О. верно решит больше 11 задач, равна 0,67. Вероятность того, что О. верно решит больше 10 задач, равна 0,74. Найдите вероятность того, что О. верно решит ровно 11 задач.

Решение. Рассмотрим события A = «учащийся решит 11 задач» и В = «учащийся решит больше 11 задач». Их сумма — событие A + B = «учащийся решит больше 10 задач». События A и В несовместные, вероятность их суммы равна сумме вероятностей этих событий:  P(A + B) = P(A) + P(B). Тогда, используя данные задачи, получаем: 0,74 = P(A) + 0,67, откуда P(A) = 0,74 − 0,67 = 0,07. Ответ: 0,07.

                                                                                                                       37. Вероятность того, что на тесте по химии учащийся П. верно решит больше 8 задач, равна 0,48. Вероятность того, что П. верно решит больше 7 задач, равна 0,54. Найдите вероятность того, что П. верно решит ровно 8 задач. Решение. Вероятность решить несколько задач складывается из суммы вероятностей решить каждую из этих задач. Больше 8:  решить 9-ю, 10-ю … Больше 7:  решить 8-ю, 9-ю, 10-ю …Вероятность решить 8-ю = 0,54-0,48=0,06.  Ответ:0.06

38. На клавиатуре телефона 10 цифр, от 0 до 9. Какова вероятность того, что случайно нажатая цифра будет меньше 4?    Ответ: 4 : 10 = 0,4.

 39. Биатлонист пять раз стреляет по мишеням. Вероятность попадания в мишень при одном выстреле равна 0,8. Найдите вероятность того, что биатлонист первые три раза попал в мишени, а последние два промахнулся. Результат округлите до сотых.

Решение. Поскольку биатлонист попадает в мишени с вероятностью 0,8, он промахивается с вероятностью 1 − 0,8 = 0,2. Cобытия попасть или промахнуться при каждом выстреле независимы, вероятность произведения независимых событий равна произведению их вероятностей. Тем самым, вероятность события «попал, попал, попал, промахнулся, промахнулся» равна 0,8•0,8•0,8•0,2•0,2=0,02048.  Ответ:0.02048.

40. Помещение освещается фонарём с двумя лампами. Вероятность перегорания лампы в течение года равна 0,3. Найдите вероятность того, что в течение года хотя бы одна лампа не перегорит.

Решение. Найдем вероятность того, что перегорят обе лампы. Эти события независимые, вероятность их произведения равно произведению вероятностей этих событий: 0,3·0,3 = 0,09. Событие, состоящее в том, что не перегорит хотя бы одна лампа, противоположное. Следовательно, его вероятность равна 1 − 0,09 = 0,91.   Ответ: 0,91.

41. Вероятность того, что батарейка бракованная, равна 0,06. Покупатель в магазине выбирает случайную упаковку, в которой две таких батарейки. Найдите вероятность того, что обе батарейки окажутся исправными.

Решение. Вероятность того, что батарейка исправна, равна 0,94. Вероятность произведения независимых событий (обе батарейки окажутся исправными) равна произведению вероятностей этих событий: 0,94·0,94 = 0,8836.   Ответ: 0,8836.

42.   Если гроссмейстер А. играет белыми, то он выигрывает у гроссмейстера Б. с вероятностью 0,52. Если А. играет черными, то А. выигрывает у Б. с вероятностью 0,3. Гроссмейстеры А. и Б. играют две партии, причем во второй партии меняют цвет фигур. Найдите вероятность того, что А. выиграет оба раза.

Решение. Возможность выиграть первую и вторую партию не зависят друг от друга. Вероятность произведения независимых событий равна произведению их вероятностей: 0,52 · 0,3 = 0,156.   Ответ: 0,156.

43.   В магазине три продавца. Каждый из них занят с клиентом с вероятностью 0,3. Найдите вероятность того, что в случайный момент времени все три продавца заняты одновременно (считайте, что клиенты заходят независимо друг от друга).

Решение. Вероятность произведения независимых событий равна произведению вероятностей этих событий. Поэтому вероятность того, что все три продавца заняты равна                       (0,3)³ = 0,027.    Ответ: 0,027.

   44. Из районного центра в деревню ежедневно ходит автобус. Вероятность того, что в понедельник в автобусе окажется меньше 20 пассажиров, равна 0,94. Вероятность того, что окажется меньше 15 пассажиров, равна 0,56. Найдите вероятность того, что число пассажиров будет от 15 до 19.

Решение. Рассмотрим события A = «в автобусе меньше 15 пассажиров» и  В = «в автобусе от 15 до 19 пассажиров». Их сумма — событие A + B = «в автобусе меньше 20 пассажиров». События A и  В несовместные, вероятность их суммы равна сумме вероятностей этих событий: P(A + B) = P(A) + P(B).

 Тогда, используя данные задачи, получаем: 0,94 = 0,56 + P(В), откуда P(В) = 0,94 − 0,56 = 0,38.Ответ: 0,38.

45. На экзамене по геометрии школьнику достаётся один вопрос из списка экзаменационных вопросов. Вероятность того, что это вопрос на тему «Вписанная окружность», равна 0,2. Вероятность того, что это вопрос на тему «Параллелограмм», равна 0,15. Вопросов, которые одновременно относятся к этим двум темам, нет. Найдите вероятность того, что на экзамене школьнику достанется вопрос по одной из этих двух тем.

Решение. Вероятность суммы двух несовместных событий равна сумме вероятностей этих событий: 0,2 + 0,15 = 0,35.

Ответ: 0,35.

 46.Вероятность того, что новый электрический чайник прослужит больше года, равна 0,97. Вероятность того, что он прослужит больше двух лет, равна 0,89. Найдите вероятность того, что он прослужит меньше двух лет, но больше года.

Решение. Пусть A = «чайник прослужит больше года, но меньше двух лет», В = «чайник прослужит больше двух лет», С = «чайник прослужит ровно два года», тогда A + B + С = «чайник прослужит больше года». События A, В и С несовместные, вероятность их суммы равна сумме вероятностей этих событий. Вероятность события С, состоящего в том, что чайник выйдет из строя ровно через два года — строго в тот же день, час и секунду — равна нулю. Тогда:  P(A + B+ С) = P(A) + P(B)+ P(С)= P(A) + P(B)

 откуда, используя данные из условия, получаем     0,97 = P(A) + 0,89.Тем самым, для искомой вероятности имеем: P(A) = 0,97 − 0,89 = 0,08.   Ответ: 0,08.

 

 47. В Волшебной стране бывает два типа погоды: хорошая и отличная, причём погода, установившись утром, держится неизменной весь день. Известно, что с вероятностью 0,8 погода завтра будет такой же, как и сегодня. Сегодня 3 июля, погода в Волшебной стране хорошая. Найдите вероятность того, что 6 июля в Волшебной стране будет отличная погода.

Решение. Для погоды на 4, 5 и 6 июля есть 4 варианта: ХХО, ХОО, ОХО, ООО (здесь Х — хорошая, О — отличная погода). Найдем вероятности наступления такой погоды: P(XXO) = 0,8·0,8·0,2 = 0,128; P(XOO) = 0,8·0,2·0,8 = 0,128; P(OXO) = 0,2·0,2·0,2 = 0,008;  P(OOO) = 0,2·0,8·0,8 = 0,128.Указанные события несовместные, вероятность их суммы равна сумме вероятностей этих событий:

 P(ХХО) + P(ХОО) + P(ОХО) + P(ООО) = 0,128 + 0,128 + 0,008 + 0,128 = 0,392.   Ответ: 0,392.

48. В магазине стоят два платёжных автомата. Каждый из них может быть неисправен с вероятностью 0,05 независимо от другого автомата. Найдите вероятность того, что хотя бы один автомат исправен. 

Решение. Найдем вероятность того, что неисправны оба автомата. Эти события независимые, вероятность их произведения равна произведению вероятностей этих событий: 0,05 · 0,05 = 0,0025. Событие, состоящее в том, что исправен хотя бы один автомат, противоположное. Следовательно, его вероятность равна 1 − 0,0025 = 0,9975. Ответ: 0,9975.

 49. В торговом центре два одинаковых автомата продают кофе. Вероятность того, что к концу дня в автомате закончится кофе, равна 0,3. Вероятность того, что кофе закончится в обоих автоматах, равна 0,12. Найдите вероятность того, что к концу дня кофе останется в обоих автоматах.

Решение. Рассмотрим событиеА = кофе закончится в первом автомате, В = кофе закончится во втором автомате.

Вероятность того, что кофе останется в первом автомате равна 1 − 0,3 = 0,7. Вероятность того, что кофе останется во втором автомате равна 1 − 0,3 = 0,7. Вероятность того, что кофе останется в первом или втором автомате равна 1 − 0,12 = 0,88. Поскольку P(A + B) = P(A) + P(B) − P(A·B), имеем: 0,88 = 0,7 + 0,7 − х, откуда искомая вероятность х = 0,52. Ответ: 0,9975.

50. Две фабрики выпускают одинаковые стекла для автомобильных фар. Первая фабрика выпускает 45% этих стекол, вторая — 55%. Первая фабрика выпускает 3% бракованных стекол, а вторая — 1%. Найдите вероятность того, что случайно купленное в магазине стекло окажется бракованным.

Решение. Вероятность того, что стекло куплено на первой фабрике и оно бракованное: 0,45 · 0,03 = 0,0135. Вероятность того, что стекло куплено на второй фабрике и оно бракованное: 0,55 · 0,01 = 0,0055.  Поэтому по формуле полной вероятности вероятность того, что случайно купленное в магазине стекло окажется бракованным равна 0,0135 + 0,0055 = 0,019.                       Ответ: 0,019.

51. Ковбой Джон попадает в муху на стене с вероятностью 0,9, если стреляет из пристрелянного револьвера. Если Джон стреляет из непристрелянного револьвера, то он попадает в муху с вероятностью 0,2. На столе лежит 10 револьверов, из них только 4 пристрелянные. Ковбой Джон видит на стене муху, наудачу хватает первый попавшийся револьвер и стреляет в муху. Найдите вероятность того, что Джон промахнётся.

Решение. Джон попадает в муху, если схватит пристрелянный револьвер и попадет из него, или если схватит непристрелянный револьвер и попадает из него. По формуле условной вероятности, вероятности этих событий равны соответственно 0,4·0,9 = 0,36 и 0,6·0,2 = 0,12. Эти события несовместны, вероятность их суммы равна сумме вероятностей этих событий: 0,36 + 0,12 = 0,48. Событие, состоящее в том, что Джон промахнется, противоположное. Его вероятность равна 1 − 0,48 = 0,52. Ответ. 0,52

 

 52. Чтобы поступить в институт на специальность «Лингвистика», абитуриент должен набрать на ЕГЭ не менее 70 баллов по каждому из трёх предметов — математика, русский язык и иностранный язык. Чтобы поступить на специальность «Коммерция», нужно набрать не менее 70 баллов по каждому из трёх предметов — математика, русский язык и обществознание.

Вероятность того, что абитуриент З. получит не менее 70 баллов по математике, равна 0,6, по русскому языку — 0,8, по иностранному языку — 0,7 и по обществознанию — 0,5.

Найдите вероятность того, что З. сможет поступить хотя бы на одну из двух упомянутых специальностей.

Решение. В силу независимости событий, вероятность успешно сдать экзамены на лингвистику: 0,6·0,8·0,7 = 0,336, вероятность успешно сдать экзамены на коммерцию: 0,6·0,8·0,5 = 0,24, вероятность успешно сдать экзамены и на «Лингвистику», и на «Коммерцию»: 0,6·0,8·0,7·0,5 = 0,168. Успешная сдача экзаменов на «Лингвистику» и на «Коммерцию» — события совместные, поэтому вероятность их суммы равна сумме вероятностей этих событий, уменьшенной на вероятность их произведения. Тем самым, поступить на одну из этих специальностей абитуриент может с вероятностью 0,336 + 0,24 − 0,168 = 0,408.  Ответ: 0,408.

53. По отзывам покупателей Иван Иванович оценил надёжность двух интернет- магазинов. Вероятность того, что нужный товар доставят из магазина А, равна 0,8. Вероятность того, что этот товар доставят из магазина Б, равна 0,9. Иван Иванович заказал товар сразу в обоих магазинах. Считая, что интернет-магазины работают независимо друг от друга, найдите вероятность того, что ни один магазин не доставит товар.                                                                                          Решение. Вероятность того, что первый магазин не доставит товар равна 1 − 0,9 = 0,1. Вероятность того, что второй магазин не доставит товар равна 1 − 0,8 = 0,2. Поскольку эти события независимы, вероятность их произведения (оба магазина не доставят товар) равна произведению вероятностей этих событий: 0,1 · 0,2 = 0,02.  Ответ: 0,02.

54.Перед началом волейбольного матча капитаны команд тянут честный жребий, чтобы определить, какая из команд начнёт игру с мячом. Команда «Статор» по очереди играет с командами «Ротор», «Мотор» и «Стартер». Найдите вероятность того, что «Статор» будет начинать только первую и последнюю игры. Решение. Требуется найти вероятность произведения трех событий: «Статор» начинает первую игру, не начинает вторую игру, начинает третью игру. Вероятность произведения независимых событий равна произведению вероятностей этих событий. Вероятность каждого из них равна 0,5, откуда находим: 0,5·0,5·0,5 = 0,125.   Ответ: 0,125.

  

55. Всем пациентам с подозрением на гепатит делают анализ крови. Если анализ выявляет гепатит, то результат анализа называется положительным. У больных гепатитом пациентов анализ даёт положительный результат с вероятностью 0,9. Если пациент не болен гепатитом, то анализ может дать ложный положительный результат с вероятностью 0,01. Известно, что 5% пациентов, поступающих с подозрением на гепатит, действительно больны гепатитом. Найдите вероятность того, что результат анализа у пациента, поступившего в клинику с подозрением на гепатит, будет положительным.

Решение.  Анализ пациента может быть положительным по двум причинам: А) пациент болеет гепатитом, его анализ верен; B) пациент не болеет гепатитом, его анализ ложен. Это несовместные события, вероятность их суммы равна сумме вероятностей этих событий. Имеем: Р(А)=0,9•0.05=0,045;                                         Р(В)= 0,01•0,95=0,0095  ,Р(А+В)=Р(А)(В)=0,045+0,0095=0,0545.

 Ответ:0,0545.

  56. Автоматическая линия изготавливает батарейки. Вероятность того, что готовая батарейка неисправна, равна 0,02. Перед упаковкой каждая батарейка проходит систему контроля. Вероятность того, что система забракует неисправную батарейку, равна 0,99. Вероятность того, что система по ошибке забракует исправную батарейку, равна 0,01. Найдите вероятность того, что случайно выбранная батарейка будет забракована системой контроля.

Решение. Ситуация, при которой батарейка будет забракована, может сложиться в результате событий: A = батарейка действительно неисправна и забракована справедливо или В = батарейка исправна, но по ошибке забракована. Это несовместные события, вероятность их суммы равна сумме вероятностей эти событий. Имеем: Р(А+В)=Р(А)+Р(В)=0,02•0,99+0,98•0,01=0,0198+0,0098=0,0296 Ответ: 0,0296.

 57. Стрелок стреляет по мишени один раз. В случае промаха стрелок делает второй выстрел по той же мишени. Вероятность попасть в мишень при одном выстреле равна 0,7. Найдите вероятность того, что мишень будет поражена (либо первым, либо вторым выстрелом).

Решение. Пусть A — событие, состоящее в том, что мишень поражена стрелком с первого выстрела, B — событие, состоящее в том, что мишень поражена со второго выстрела. Вероятность события A равна P(A) = 0,7. Событие B наступает, если, стреляя первый раз, стрелок промахнулся, а, стреляя второй раз, попал. Это независимые события, их вероятность равна произведению вероятностей этих событий: P(B) = 0,3·0,7 = 0,21. События A и B несовместные, вероятность их суммы равна сумме вероятностей этих событий:  P (A + B) = P(A) + P(B) = 0,7 + 0,21 = 0,91.   Ответ: 0,91.

58.Перед началом футбольного матча судья бросает монетку, чтобы определить, какая из команд будет первой владеть мячом. Команда А должна сыграть два матча — с командой В и с командой С. Найдите вероятность того, что в обоих матчах первой мячом будет владеть команда А.

Решение. Рассмотрим все возможные исходы жеребьёвки.

 · КомандаА в матче в обоих матчах первой владеет мячом.

 · КомандаА в матче в обоих матчах не владеет мячом первой.

 · КомандаА в матче с командой В владеет мячом первой, а в матче с командой С — второй.

 · КомандаА в матче с командой С владеет мячом первой, а в матче с командой В — второй.

Из четырех исходов один является благоприятным, вероятность его наступления равна 1:4=0,25. Ответ: 0,25.

59. Стрелок 4 раза стреляет по мишеням. Вероятность попадания в мишень при одном выстреле равна 0,5. Найдите вероятность того, что стрелок первые 3 раза попал в мишени, а последний раз промахнулся.

Решение. Вероятность промаха равна 1 − 0,5 = 0,5. Вероятность того, что стрелок первые три раза попал в мишени равна 0,53 = 0,125. Откуда, вероятность события, при котором стрелок сначала три раза попадает в мишени, а четвёртый раз промахивается равна 0,125 · 0,5 = 0,0625.     Ответ: 0,0625.

60. Перед началом матча по футболу судья бросает монету, чтобы определить, какая из команд будет первой владеть мячом. Команда «Байкал» играет по очереди с командами

«Амур», «Енисей», «Иртыш». Найти вероятность того, что команда «Байкал» будет первой  владеть мячом только в игре с «Амуром».

Решение. Монету бросают 3 раза.

Для команды «Байкал» возможные исходы в трех бросках {О О О},{Р О О}, {О Р О}, {О О Р},                  {Р Р О},{Р О Р}, {О Р Р},{Р Р Р}. Всего исходов 8, благоприятныx1(выпадение орла в первой игре)  {О Р Р, 1:8=0,125.Ответ 0,125.

61.У Пети в кармане лежат  шесть монет: четыре монеты по рублю и две монеты по два рубля. Петя, не глядя, переложил какие-то три монеты в другой карман. Найдите вероятность  того, что теперь две двухрублевые монеты лежат в одном кармане.

 Решение. Пронумеруем монеты: рублевые – 1, 2, 3, 4;  двухрублевые – 5, 6. {123} {124} {125} {126} {134} {135} {136} {145} {146} {156} {234} {235} {236} {245} {246} {256} {345} {346} {356} {456} 

 n = 20     – число всех исходов .Взять три монеты можно так: (числа в порядке возрастания,чтобы не пропустить комбинацию)  m = 8  – число благоприятных  исходов

(комбинации, в которых монеты 5 и 6 (двухрублевые) не взяты или взяты обе. 8:20=0,4

Теория вероятностей задачи с решением

Прежде чем изучать готовые решения задач по теории вероятности, нужно знать теорию, поэтому для вас я подготовила краткую теорию по предмету «теория вероятностей», после которой подробно решены задачи.

Эта страница подготовлена для школьников и студентов.

Если что-то непонятно — вы всегда можете написать мне в WhatsApp и я вам помогу!

Введение в теорию вероятностей

Прежде чем переходить к строгим определениям основных понятий теории вероятностей, мы рассмотрим несколько простых и в то же время типичных ситуаций, призванных проиллюстрировать идейную сторону дальнейшего изложения.

Схема случаев

Потребность в теоретико-вероятностных методах, как правило, возникает в ситуации, когда исход изучаемого явления по тем или иным причинам не может быть однозначно спрогнозирован. Одной из простейших моделей такого сорта ситуаций может служить схема случаев. Под схемой случаев мы понимаем такую ситуацию, когда, во-первых, множество возможных исходов рассматриваемого эксперимента образует конечную совокупность, а во-вторых, каждый из исходов имеет такие же шансы на осуществление, как и любой другой. При этом предполагается, что исследуемое явление может наблюдаться в идентичных условиях неограниченное число раз — эксперимент обладает свойством повторяемости. Очевидно, что в этом случае шансы на осуществление того или иного исхода в каждом отдельно взятом эксперименте тем меньше, чем больше самих исходов, а шансы на осуществление какой-либо группы исходов пропорциональны количеству исходов в рассматриваемой группе.

Пример 1. Рассмотрим эксперимент, состоящий в подбрасывании монеты. Пренебрегая «нештатными» возможностями — монета стала на ребро, закатилась в щель, прилипла к потолку и т. п. — будем считать, что возможные исходы этого эксперимента — это выпадение герба или решетки (решки). Если предположить дополнительно, что монета является физически симметричной и что эксперимент производится «честно», то мы получим простейший пример схемы случаев с двумя равновозможными исходами. Заметим, что если монета несимметрична, то рассмотренный эксперимент схемой случаев в нашем понимании описан быть не может, так как исходы уже не будут обладать свойством равновозможности.

Пример 2. Пусть в урне лежит N физически идентичных шаров, пронумерованных от 1 до N. Эксперимент состоит в извлечении шара из урны, возможный исход однократного извлечения — любой номер от 1 до N. Если шары тщательно перемешаны и извлекаются из урны наугад, то мы имеем пример схемы случаев с N равновозможными исходами.

Вероятность исхода. Событие. Вероятность события

Для описания возможности осуществления того или иного исхода в схеме случаев введем количественную характеристику указанной возможности — вероятность исхода, которую определим как величину, обратно пропорциональную общему количеству N равновозможных при однократном проведении эксперимента исходов:

Теория вероятностей

где к — некоторый коэффициент пропорциональности

Для того чтобы понять, каким он должен быть, введем понятие события, которое может (или не может) осуществляться в эксперименте:

Событием в эксперименте, описываемом схемой случаев, назовем любую совокупность исходов рассматриваемого эксперимента.

Мы будем говорить, что событие осуществилось, если в результате однократного проведения эксперимента реализовался один из составляющих это событие исходов.

Пример 3. Рассмотрим эксперимент, состоящий в извлечении ровно одной карты из тщательно перетасованной колоды, содержащей 36 карт. Этот эксперимент может быть описан схемой случаев с 36 равновозможными исходами.

Примерами событий в рассматриваемой ситуации могут служить следующие:
Теория вероятностей = {извлеченная карта имеет масть пик}. Это событие состоит из всех (9) пиковых карт. Оно осуществляется (происходит) в эксперименте, если мы извлекли из колоды любую пиковую карту.
— Десятка = {извлеченная карта — десятка}. Это событие состоит из всех (4) карт с изображением десятки. Оно происходит, если мы извлекли из колоды любую десятку.
— Картинка = {извлеченная карта — валет, дама, король или туз}.

Если у нас есть пара событий А и В, то можно сконструировать из них новые события, пользуясь следующими простыми правилами действий:

Суммой двух событий А и В назовем событие, происходящее, если происходит либо событие А, либо событие В, либо оба эти события одновременно. Легко понять, что сумма событий составляется из всех исходов входящих либо в А, либо в В, при этом общие исходы (т. е. входящие одновременно и в Л, и в В) входят в сумму однократно.

Обозначение для суммы: A U В.

Пример 4. В описанном выше эксперименте с извлечением одной карты из 36-листовой колоды суммой событий А = {извлеченная карта масти пик} = {6Теория вероятностей,…, ТТеория вероятностей} и В = {извлеченная карта — король} = {КТеория вероятностей, КТеория вероятностей, КТеория вероятностей, КТеория вероятностей} будет событиеТеория вероятностей, состоящее в извлечении карты масти пик или любого короля.

Совмещением (произведением) двух событий А и В назовем событие, происходящее, если события Л и В осуществляются одновременно.

Совмещение событий состоит из всех общих для событий А и В исходов.

Обозначение для совмещения: Теория вероятностей Чаще всего знак совмещения Теория вероятностей опускают, обозначая совмещение событий А • В = АВ.

Если у событий отсутствуют общие исходы, то такие события вместе не происходят. Они называются несовместными. При сложении несовместных событий обычно используется значок «+» вместо значка объединения Теория вероятностей — сумма обозначается в этом случае как А + В.

Пример 5. В описанном выше эксперименте совмещением событий А и В будет событие, состоящее в извлечении короля пик.

Здесь же события Теория вероятностей и Теория вероятностей — несовместны.

Отрицанием события А или событием, противоположным событию А, назовем событие, происходящее, когда событие А не происходит. Отрицание (противоположное событие) состоит из всех тех исходов эксперимента, которые не входят в А.

Обозначение для отрицания (противоположного события): Теория вероятностей

Пример 6. В описанном выше эксперименте отрицанием события В будет событие, состоящее в извлечении любой карты, не являющейся королем.

Рассмотрим теперь событие, которое в дальнейшем будем обозначать буквой Теория вероятностей, составленное из всех исходов рассматриваемого эксперимента. Очевидно, что при любой реализации эксперимента какой-нибудь исход обязательно осуществится, а следовательно, осуществится событие Теория вероятностей, что дает основание назвать это событие достоверным. Оно происходит всегда, когда проводится эксперимент. Пополним множество возможных в рассматриваемом эксперименте событий событием невозможным, которое в данном эксперименте не происходит. Невозможное событие будем обозначать символом Теория вероятностей.

Легко понять, что если мы имеем дело со схемой случаев с N равновозможными исходами, то общее количество всех событий в рассматриваемом эксперименте равно Теория вероятностей.

◄ Действительно, событий, состоящих ровно из одного исхода, будет N, из двух исходов — Теория вероятностей, и вообще, событий, состоящих из S исходов, будет Теория вероятностей. Таким образом, число возможных событий равно

Теория вероятностей

если добавить теперь к этому количеству еще одно — невозможное — событие, получим искомый результат, так как известно, что

Теория вероятностей

откуда и следует искомое. ►

Отметим несколько очевидных соотношений:

Теория вероятностей

Естественно под вероятностью события понимать величину, пропорциональную количеству входящих в него исходов — если некоторое событие составлено S исходами, то его вероятность положим равной

Теория вероятностей

При этом ясно, что чем больше исходов входят в событие (говорят — благоприятствуют осуществлению события), тем больше шансы на его осуществление, как следствие — тем больше его вероятность. Все события, осуществляющиеся в эксперименте, с точки зрения шансов на осуществление естественно располагаются между невозможным и достоверным событиями.

Вероятность невозможного события положим равной нулю, отмечая тем самым, что шансов на осуществление невозможного события нет.

Вероятность достоверного события может быть принята равной любому положительному числу — никаких запретов или ограничений на это значение нет. Так как достоверное событие включает все возможные исходы, то его вероятность больше вероятности любого другого события в этом эксперименте и равна, в силу (2), сумме вероятностей всех исходов, т. е. Теория вероятностейТаким образом, коэффициент пропорциональности в соотношении (1) равен вероятности достоверного события.

Поскольку выбор значения вероятности достоверного события не влияет на содержательную сторону описания возможности осуществления того или иного исхода в схеме случаев, а меняет только масштаб шкалы измерения вероятностей, положим

Теория вероятностей

и, тем самым, завершим определение вероятностей исхода и события в схеме случаев.

Суммируя вышеизложенное, еще раз отметим, что все события, происходящие в эксперименте, могут быть естественным образом ранжированы в соответствии с их шансами на осуществление при однократной реализации эксперимента. В этой ранжировке они располагаются между невозможным событием, которое не происходит никогда, и достоверным, которое реализуется всегда, когда реализуется эксперимент.

Мерой осуществимости любого события А выступает его вероятность, определяемая как отношение количества S благоприятствующих осуществлению события исходов к общему числу N всех возможных исходов:

Теория вероятностей

Отметим некоторые свойства вероятности события в схеме случаев.

1. Вероятность любого события, происходящего в рассматриваемом эксперименте, задается положительным числом, заключенным в пределах между 0 и 1

Теория вероятностей

2. Если события А и В — несовместны, то вероятность суммы равна сумме вероятностей

Теория вероятностей

в частности, справедлив так называемый принцип дополнительности

Теория вероятностей

Рассмотрим несколько примеров, иллюстрирующих технику нахождения вероятностей событий в схеме случаев.

Пример 7. Пусть в урне лежит m+n физически идентичных шаров, окрашенных соответственно в белый (n шаров) и черный (m шаров) цвета. Эксперимент состоит в извлечении из урны одного шара. Найдем вероятность извлечения шара белого цвета

Теория вероятностей Поскольку всего возможных исходов N = m + n, а благоприятствующих извлечению белого шара — п, то искомая вероятность дается отношением n/(m + n). ►

Пример 8. В условиях предыдущего примера производится извлечение двух шаров. Какова вероятность того, что оба извлеченных шара — белые? Извлечены разноцветные шары?

Теория вероятностей Ответ на первый вопрос задачи может быть получен, например, с помощью следующих рассуждений.

Всего различных пар шаров из урны, содержащей m + n физически идентичных шаров, можно составить Теория вероятностей, так что можно считать, что всего различных исходов в рассматриваемом эксперименте Теория вероятностей. В то же время количество пар белых шаров дается числом Теория вероятностей, откуда искомая вероятность равна

Теория вероятностей

Аналогичные соображения для второго вопроса дают.

Теория вероятностей

Заметим, что рассматриваемый эксперимент — извлечение пары шаров — эквивалентен двукратному последовательному извлечению шаров из урны.

Пример 9. В условиях предыдущего примера производится последовательное извлечение двух шаров с возвращением каждого извлеченного шара обратно в урну. Какова вероятность того, что оба извлеченных шара — белые’ Извлечены разноцветные шары?

Теория вероятностей В отличие от предыдущей ситуации, общее количество возможных исходов в рассматриваемом эксперименте будет уже равно Теория вероятностей, а количество S исходов, благоприятствующих интересующему нас событию (оба шара — белые) — Теория вероятностей, и, следовательно, искомая вероятность равна

Теория вероятностей

Аналогичные рассуждения для второго вопроса дают:

Теория вероятностей

Пример 10. Ребенок, играя с четырьмя карточками разрезной азбуки, на которых изображены буквы А, А, М, М, случайным образом выкладывает их в ряд. Какова вероятность того, что у него получится слово МАМА?

Теория вероятностей Предполагая, что все возможные расстановки четырех карточек в ряд равновозможны, получаем схему случаев с общим количеством исходов N = 4! = 24. Количество исходов, благоприятствующих интересующему нас событию, равно S = 4, откуда искомая вероятность равна 4/24 =1/6. ►

Пример 11. Студент подготовил к экзамену 40 из 50 вопросов, охватывающих программу изученного курса. На экзамене ему предлагается дать ответ на два случайным образом выбранных из общего списка вопроса. Какова вероятность того, что студент знает ответ на оба предложенных ему вопроса?

Теория вероятностей Легко видеть, что всего различных вариантов выбора пары различных вопросов из общего списка, содержащего 50 вопросов, будет Теория вероятностей. Интересующее нас событие состоит из таких пар вопросов, оба из которых известны студенту. Их количество Теория вероятностей Таким образом, искомая вероятность дается отношением

Теория вероятностей

Пример 12. В урне лежит всего 10 черных и белых шаров. Из урны извлекают без возвращения пару шаров. Известно, что вероятности извлечения одноцветных шаров относятся как 2 : 5. Можно ли по этим данным установить состав шаров в урне?

Теория вероятностей Пусть в урне лежит п белых (черных) (соответственно, 10 — n черных (белых)) шаров. Тогда вероятность извлечения из урны пары белых (черных) шаров будет равна

Теория вероятностей

аналогично, пары черных (белых) шаров

Теория вероятностей

Из условия задачи следует

Теория вероятностей

откуда для n получаем уравнение

Теория вероятностей

единственное целое положительное решение (n = 4) которого дает ответ — в урне возможно наличие 4 белых и 6 черных, либо 4 черных и 6 белых шаров. ►

Пример 13. В урне лежит некоторое количество белых и черных шаров, так что вероятность извлечения пары белых шаров равна 0,5. Какое минимально возможное количество шаров находится в урне? Каков при этом состав шаров в урне?

Теория вероятностей Пусть в урне лежит всего N шаров, из которых n < N — белые. Вероятность извлечения пары белых шаров (см. предыдущий пример) дается соотношением

Теория вероятностей

и условие задачи приводит к уравнению, связывающему N и n

Теория вероятностей

Учитывая, что величины n и N — целые положительные числа, удовлетворяющие условию n < N, выразим из этого соотношения п через N

Теория вероятностей

и, придавая величине N последовательно значения 1, 2,… , найдем, что наименьшее значение N, при котором n — целое положительное, равняется 4. Значение n при этом равно 3. ►

Пример 14. Среди выпущенных N лотерейных билетов n — выигрышных. Некто приобрел r < N — n лотерейных билетов. Какова вероятность того, что среди них по крайней мере один выигрышный?

Теория вероятностей Заметим, что событие А = {среди приобретенных r билетов по крайней мере один выигрышный} противоположно событию Теория вероятностей= {среди приобретенных r билетов выигрышных нет}. Для решения задачи воспользуемся принципом дополнительности: Теория вероятностей

Найдем вероятность Теория вероятностей. Всего возможных исходов (т. е. различных наборов из r лотерейных билетов) Теория вероятностей, среди них таких, которые не содержат ни одного выигрышного билета —Теория вероятностей. Для вероятности Теория вероятностей получаем

Теория вероятностей

откуда искомая вероятность дается соотношением

Теория вероятностей

Пример 15. Среди выпущенных N лотерейных билетов n — выигрышных. Некто приобрел r < min{n, N — n} лотерейных билетов. Какова вероятность того, что среди них ровно k выигрышных?

Теория вероятностей Как и выше, всего возможных исходов (т. е. различных наборов из r лотерейных билетов) будет Теория вероятностей. Набор, содержащий ровно k выигрышных билетов, образуется в результате объединения любых k выигрышных билетов с любыми r — k невыигрышными. Количество различных наборов из к выигрышных билетов равно Теория вероятностей, количество различных наборов из r — k невыигрышных билетов равно Теория вероятностей. Следовательно, количество различных комбинаций из выигрышных и невыигрышных билетов дается числом Теория вероятностей, а искомая вероятность равна

Теория вероятностей

Геометрические вероятности

Другая схема описания экспериментов с неоднозначно прогнозируемыми исходами, которая позволяет довольно просто ввести количественную характеристику осуществимости того или иного события — это схема геометрических вероятностей, которая , как и рассмотренная выше схема случаев, эксплуатирует идею о равновозможности исходов эксперимента.

Аналогично тому, как это было проделано в схеме случаев, количественная характеристика осуществимости события — его вероятность — определяется как нормированная некоторым образом величина, пропорциональная запасу исходов, благоприятствующих осуществлению события.

Пусть множество исходов исследуемого эксперимента может быть описано как множество Теория вероятностей точек некоторого «геометрического континуума» — каждому исходу соответствует некоторая точка и каждой точке отвечает некоторый исход. В качестве «геометрического континуума» Теория вероятностей может выступать отрезок на прямой, дуга спрямляемой кривой на плоскости или в пространстве, квадрируемое множество на плоскости (треугольник, прямоугольник, круг, эллипс и т. п.) или часть квадрируемой поверхности, некоторый объем в пространстве (многогранник — призма, пирамида, шар, эллипсоид и т. п.)

Событием назовем любое квадрируемое подмножество множества Теория вероятностей.

Как и в схеме случаев, событие состоит из точек-исходов, однако уже не любая совокупность исходов образует событие, атолькотакая, меру которой (длину, площадь, объем) мы можем измерить.

Предполагая равновозможность исходов, назовем вероятностью события А число, пропорциональное мере подмножества А множества Теория вероятностей:

Теория вероятностей

Если Теория вероятностей — событие, невозможное в данном эксперименте, a Теория вероятностей — достоверное, то положим Теория вероятностейВероятность любого события А будет при этом заключена между нулем — вероятностью события невозможного, и единицей — вероятностью события достоверного. Условие нормировки позволяет найти константу k — коэффициент пропорциональности, задающий вероятность. Он оказывается равен Теория вероятностей.

Таким образом, в схеме геометрических вероятностей вероятность любого события определяется как отношение меры подмножества А, описывающего событие, к мере множества Теория вероятностей, описывающего эксперимент в целом:

Теория вероятностей

Отметим некоторые свойства так определенной вероятности:

  1. Теория вероятностей
  2. Если Теория вероятностей

◄ Свойство очевидно следует из того обстоятельства, что множество, содержащееся внутри другого, не может быть больше последнего. ►

Как и в схеме случаев, события в схеме геометрических вероятностей можно объединять, совмещать и строить на их основе противоположные — при этом будут получаться, вообще говоря, отличные от исходных события. Следующее свойство весьма важно.

Если события А и В — несовместны, то Р(А U В) = Р(А)+Р(В), в частности, справедлив принцип дополнительности: Теория вероятностей

◄ Это свойство, называемое обычно правилом сложения вероятностей, очевидно следует из аддитивности меры. ►

В заключение отметим, что вероятность осуществления любого исхода в схеме геометрических вероятностей всегда равна нулю, равно как равна нулю вероятность любого события, описываемого «тощим» множеством точек, т. е. множеством, мера которого (соответственно — длина, площадь, объем) равна нулю.

Рассмотрим несколько примеров, иллюстрирующих вычисление вероятностей в схеме геометрических вероятностей.

Пример 1. Эксперимент состоит в случайном выборе точки из отрезка [а, b]. Найти вероятность того, что выбрана точка, лежащая в левой половине рассматриваемого отрезка.

◄ По определению, вероятность выбора точки из любого множества на отрезке [а, b] пропорциональна длине этого множества. Следовательно, искомая вероятность равна 0,5:

Теория вероятностей

Пример 2. Эксперимент состоит в случайном выборе точки Теория вероятностей из квадрата Теория вероятностей Какова вероятность того, что уравнение

Теория вероятностей

имеет действительные корни? Равные корни?

Теория вероятностей Хорошо известно, что у квадратного уравнения корни действительны, если его дискриминант неотрицателен. В рассматриваемом случае дискриминант D дается соотношением

Теория вероятностей

и будет неотрицателен, если Теория вероятностей удовлетворяют условию

Теория вероятностей

т е если точка Теория вероятностей будет выбрана из множества А, являющегося пересечением квадрата К и множества точек, описываемого вышеприведенными условиями (рис. 1).

Теория вероятностей

Следовательно, для искомой вероятности получаем:

Теория вероятностей

Далее, корни квадратного уравнения совпадают, если D = 0. Этому значению дискриминанта отвечает отрезок оси Теория вероятностей от — 1 до +1 и отрезок биссектрисы первого и третьего координатного угла, лежащий внутри квадрата К (рис. 1). Легко понять, что площадь этого множества точек равна нулю и, следовательно, вероятность совпадения корней рассматриваемого уравнения равна нулю. ►

Следующий пример является классическим и призван проиллюстрировать то простое соображение, что понятие «случайности» не является очевидным и одинаково понимаемым всеми, а потому должно быть, вообще говоря, аккуратно формализовано, иначе использование вероятностных соображений может привести к недоразумениям.

Пример 3. В круге радиуса R случайным образом выбрана хорда. Какова вероятность того, что длина этой хорды больше радиуса?

◄ В первую очередь следует понять, что значит хорда выбрана случайно.

1. Поскольку длина хорды однозначно определяется расстоянием этой хорды от центра круга, то одна из возможных интерпретаций случайного выбора может выглядеть так:

Случайный выбор хорды эквивалентен случайному выбору точки на диаметре круга.

Длина хорды, находящейся на расстоянии d от центра, равна Теория вероятностей, и для того чтобы длина хорды превышала длину радиуса круга, нужно, чтобы выбранная точка была расположена от центра круга на расстоянии, не превышающем Теория вероятностей (рис. 2)

Поэтому искомая вероятность раана

Теория вероятностей

Теория вероятностей

2. Всякая хорда может быть задана парой точек на окружности, являющихся ее концами. Поэтому другая интерпретация случайного выбора хорды может быть сформулирована так.

Случайный выбор хорды эквивалентен случайному выбору пары точек на дуге окружности.

Теория вероятностей

Выбирая на окружности начало отсчета и задавая направление обходе (например, против часовой стрелки), пометим положение любой точки на окружности ее координатой, меняющейся в пределах от 0 до Теория вероятностей. Множество хорд может быть описано множеством упорядоченных пар чисел (x, у), Теория вероятностей — координат начала и конца каждой из хорд (рис. 3, слева). Это множество на плоскости координат (x, у) изображается треугольником ОАВ (рис. 3, справа). Понятно, что длина хорды будет больше радиуса, если координаты начала и конца хорды удовлетворяют условиям

Теория вероятностей

Последние могут быть записаны одним двойным неравенством

Теория вероятностей

Множество точек (х,у), удовлетворяющих этому условию, заштриховано на рис. 3 (справа). Теперь легко находим

Теория вероятностей

и искомая вероятность равна

Теория вероятностей

что отличается от результата, полученного выше. ►

Условные вероятности. Взаимное влияние и независимость

Информация о реализации некоторого события в эксперименте может менять наши представления о шансах на осуществление других событий.

Пример 1. Пусть в эксперименте с бросанием симметричной монеты рассматриваются события Г — выпадение герба и Р — выпадение решки. Очевидно, что если нам известно: выпал герб, т. е. осуществилось событие Г, то осуществление события Р — выпадение решки в этом эксперименте невозможно.

Пример 2. Если в эксперименте с извлечением карты извлечена карта КТеория вероятностей, то очевидно, что одновременно осуществились события Теория вероятностей = {извлечена карта масти пик} и К = {извлечен король}. Другими словами, осуществление события КТеория вероятностей влечет за собой осуществление и этих событий.

Но, конечно, может оказаться и так, что осуществление одного из событий в эксперименте ничего не говорит нам об осуществлении или неосуществлении другого, точнее, не меняет наших представлений о шансах на его осуществление.

Пример 3. Рассмотрим эксперимент, состоящий в двукратном извлечении шаров из урны с последующим возвращением извлеченного шара обратно в урну. Пусть в урне лежит N = m + n соответственно черных (m) и белых (n) шаров. Рассмотрим события: А — шар, извлеченный первым, белый, В — шар, извлеченный вторым, белый. Поскольку после каждого извлечения шар возвращается в урну, то ясно, что зависимости между этими событиями нет.

Из общих соображений понятно, что при условии осуществления одного из событий шансы на осуществление другого должны быть пропорциональны запасу их общих исходов — чем значительнее общая часть рассматриваемых событий, тем выше должны быть шансы на осуществление одного из них, в предположении, что другое произошло.

Введем соответствующее формальное понятие.

Условной вероятностью осуществления события А относительно события В назовем число

Теория вероятностей

где Теория вероятностей —запас исходов эксперимента, благоприятствующих осуществлению соответственно событий Теория вероятностей

Пусть событие В фиксировано и таково, что Р(В) > 0. Тогда условная вероятность обладает следующими очевидными свойствами:

1. Теория вероятностей

2. Если события А и В — несовместны, то Р(А|В) = 0, если же события А и В таковы, что В составляет часть А, то Р(А|В) = 1, в частности Р(В|В) = 1.

3. Для условных вероятностей справедливо правило сложения

Теория вероятностей

если только события Ai и А2 несовместны.

Таким образом, условная вероятность обладает всеми свойствами вероятности и описывает шансы на осуществление события А при уже происшедшем событии В. Очевидно, что, вообще говоря, Теория вероятностей

Сразу же заметим, что условная вероятность может быть вычислена как отношение вероятности совместного осуществления событий А и В к вероятности события-условия В:

Теория вероятностей

Из последнего соотношения следует правило умножения вероятностей

Теория вероятностей

справедливое для событий с положительной вероятностью.

Рассмотрим несколько примеров, иллюстрирующих введенное понятие.

Пример 4. Из урны, содержащей n белых и m черных шаров, извлекают без возвращения пару шаров. Какова вероятность извлечь вторым черный шар, если известно, что первым был извлечен черный?

Теория вероятностей Очевидно, что если первым был извлечен черный шар, то в урне осталось всего n+m— 1 шаров, m — 1 среди которых черных m — 1. Поэтому искомая вероятность равна Теория вероятностей

Пример 5. Эксперимент состоит в случайном выборе точки из квадрата Теория вероятностей. Найти вероятность того, что первая координата точки не превышает 0,5, если известно, что выбрана точка, лежащая выше биссектрисы первого и третьего координатных углов (рис. 4).

◄ Из рисунка легко усмотреть, что вероятность события В = {выбрана точка, лежащая выше биссектрисы} равна 0,5, а вероятность совместного осуществления событий В и Теория вероятностейТеория вероятностей. Отсюда для искомой вероятности получаем: Р(A|В) = 3/4. ►

Теория вероятностей

Понятие условной вероятности позволяет ввести также количественную меру, характеризующую степень влияния одного из событий на другое.

Будем говорить, что событие А не зависит от события В, если осуществление события А не меняет вероятности осуществления события В, т. е. если условная вероятность Р(А|В) совпадает с безусловной Р(А):

Теория вероятностей

В противном случае будем говорить, что событие А зависит от В.

Сразу же отметим, что понятия зависимости-независимости, несмотря на явную несимметричность определения, носят взаимный характер — если событие А зависит (не зависит) от события В, то и событие В зависит (не зависит) от события А.

◄ Действительно, пусть событие А не зависит от события В. Рассмотрим

Теория вероятностей

но, в силу независимости, Р (А|В) = Р (A), откуда и следует независимость В от А. ►

В случае независимых событий правило умножения вероятностей принимает особенно простой вид: вероятность совместного осуществления двух событий равна произведению их вероятностей:

Теория вероятностей

Соотношение (5) может быть принято в качестве определения независимости.

Нижеследующие примеры иллюстрируют использование правила умножения при вычислении вероятностей событий.

Пример 6. Из урны, содержащей п белых и т черных шаров, извлекают три шара. Какова вероятность того, что среди извлеченных есть хотя бы один белый шар?

◄ Заметим, что интересующее нас событие А противоположно событию — все извлеченные шары черные. В соответствии с принципом дополнительности Теория вероятностей Вероятность события Теория вероятностей найдем, воспользовавшись тем, что Теория вероятностей, где события Теория вероятностей означают, что шар, извлеченный і-м — черный. В соответствии с правилом умножения получаем

Теория вероятностей

Для вероятностей, участвующих в этом соотношении, легко получаем

Теория вероятностей

откуда ответ

Теория вероятностей

Пример 7. В круге радиуса R случайным образом независимо друг от друга выбрано N точек. Найти вероятность того, что расстояние от центра круга до ближайшей из них будет не менее r.

◄ Ясно, что если ближайшая из точек находится от центра на расстоянии не меньшем r, то и все прочие будут находиться от центра на не меньшем расстоянии.

Вероятность того, что случайная в круге точка находится от центра на расстоянии не меньшем r, дается отношением

Теория вероятностей

В соответствии с правилом умножения (5), искомая вероятность равна

Теория вероятностей

Пример 8. Некто забыл последнюю цифру номера телефона и набирает ее наугад. Какова вероятность того, что он дозвонится до нужного абонента не более чем за три попытки?

◄ Воспользуемся принципом дополнительности — противоположным рассматриваемому событию А будет событие Теория вероятностей, состоящее в том, что первые три попытки дозвониться до нужного абонента оказались безуспешными. Последовательные попытки дозаониться до нужного абонента Теория вероятностей — зависимые события, так как однажды набранная и не принесшая успеха цифра в дальнейшем уже не набирается. Применим прааило умножения (4):

Теория вероятностей

Для сомножителей очевидно имеем

Теория вероятностей

Отсюда

Теория вероятностей

и для искомой вероятности получаем Теория вероятностей

Пример 9. Исследовать связь между темным цветом глаз у отца (событие Теория вероятностей) и сына (событие Теория вероятностей) на основании следующих данных, полученных при переписи населения Англии и Уэльса в 1891 году.

Темноглазые отцы и темноглазые сыновья Теория вероятностей составляли 5% среди всех обследованных, темноглазые отцы и светлоглазые сыновья Теория вероятностей — 7,9%, светлоглазые отцы и темноглазые сыновья Теория вероятностей — 8,9%, светлоглазые отцы и светлоглазые сыновья Теория вероятностей — 78,2%.

◄ Для оценки исследуемой связи найдем условные вероятности Теория вероятностей и сравним их с соответствующей безусловной Теория вероятностей.

По определению имеем

Теория вероятностей

Условия задачи дают основания для следующей оценки вероятностей

Теория вероятностей

Поскольку очевидно, что Теория вероятностей, постолькуТеория вероятностей Отсюда

Теория вероятностей

В то же время Теория вероятностей. Сравнивая значения условной и безусловной вероятностей, делаем заключение о наличии связи между темным цветом глаз у отца и темным цветом глаз у сына — у темноглазых отцов темноглазые сыновья встречаются почти втрое чаще, чем вообще среди обследованных. Заметим, между прочим, что светлоглазые сыновья у темноглазых отцов встречаются примерно а 6 случаях из Теория вероятностей

Подсчитаем теперь вероятность Теория вероятностей. Рассуждения, аналогичные вышеприведенным, дают:

Теория вероятностей

Заключаем, что светлоглазые отцы, вообще говоря, могут иметь темноглазых сыновей, однако значительно реже, чем светлоглазых — примерно в одном случае из 10 у светлоглазых отцов темноглазые сыновья и, соответственно, в 9 случаях из 10 — светлоглазые. ►

Формула полной вероятности

Введенные в предыдущем разделе понятия условной вероятности и независимости событий позволяют получить простое соотношение, облегчающее вычисление вероятностей в многоальтернативных ситуациях — когда событие, вероятность которого отыскивается, может происходить совместно с другими событиями, относительно которых подсчет вероятностей интересующего нас события оказывается по каким-то причинам проще. Это соотношение носит название формулы полной вероятности.

Пусть события Теория вероятностей имеют ненулевую вероятность, несовместны и вместе исчерпывают все возможные исходы эксперимента:

Теория вероятностей

Теория вероятностей

Совокупность событий, обладающих перечисленными свойствами, задает альтернативное разбиение множества всех исходов эксперимента и обычно называется полной группой несовместных событий.

Если А — некоторое событие, то очевидно, что разбиение эксперимента, задаваемое полной группой событий Теория вероятностей задает и разбиение множества исходов, образующих событие А (рис. 5):

Теория вероятностей

(при этом, конечно, некоторые слагаемые в приведенной сумме могут оказаться невозможными событиями).

Поскольку события Теория вероятностей — несовместны, то и события Теория вероятностейТеория вероятностей — также несовместны, и по правилу сложения заключаем, что

Теория вероятностей

Правило же умножения (4) позволяет вычислить каждое из слагаемых

Теория вероятностей

Объединяя два последних соотношения, получаем искомую формулу

Теория вероятностей

Пример 1. На книжной полке стоит два десятка книг, из которых 4 уже прочитаны хозяином, а оставшиеся еще нет Хозяин выбирает случайным образом книгу и читает (или перечитывает) ее, после чего ставит обратно на полку После этого он выбирает наугад очередную книгу. Какова вероятность того, что вновь выбранная книга еще не была прочитана?

◄ Рассуждения выглядят следующим образом пусть Теория вероятностей — событие, состоящее в том, что первая книга читанная, Теория вероятностей — нечитанная, А — вновь выбранная книга еще не была прочитана

Легко установить, что имеют место следующие соотношения

Теория вероятностей

и формула (6) для искомой вероятности дает

Теория вероятностей

Пример 2. На перегоне между двумя остановками в автобусе едут 3 пассажира, каждый из которых, независимо от прочих, с вероятностью 0,1 покидает автобус на ближайшей остановке. На этой остановке ожидают транспорт 3 пассажира, каждый из которых, независимо от прочих ожидающих, садится в подошедший автобус с вероятностью 0,3. Какова вероятность того, что после отправления с этой остановки количество пассажиров в салоне автобуса не изменится?

Теория вероятностейОчевидно, что количество пассажиров в салоне автобуса останется неизменным, если количество вышедших будет равно количеству вошедших.

Пусть события Теория вероятностей состоят соответственно в том, что в автобус не сел никто, сел ровно один, два или три пассажира, событие А — количество пассажиров в салоне автобуса не изменилось. По формуле полной вероятности получаем

Теория вероятностей

Учитывая независимость посадки пассажиров в автобус, легко находим

Теория вероятностей

Заметим, что условные вероятности Теория вероятностей есть вероятности того, что из автобуса вышло на остановке ровно г пассажиров.

Теория вероятностей

Окончательно

Теория вероятностей

Формула полной вероятности вместе с формулой условной вероятности позволяет выносить некие суждения о «правдоподобности» гипотез:

Если событие А может происходить в эксперименте совместно с одним из альтернативных событий Теория вероятностей и в результате эксперимента это событие осуществилось, то можно попытаться ответить на вопрос — с каким именно из событий Теория вероятностей оно произошло вместе. Для этого оценим условную вероятность события (гипотезы) Теория вероятностей при условии, что событие А реализовалось:

Теория вероятностей

В соответствии с правилом умножения, вероятность, стоящая в числителе, дается выражением Теория вероятностей, а стоящая в знаменателе может быть подсчитана с помощью формулы полной вероятности (6), что дает

Теория вероятностей

Сравнивая вероятностиТеория вероятностей для различных значений i, будем считать ту из гипотез наиболее вероятной, для которой эта вероятность наибольшая.

Формула (7) называется формулой Бейеса, вероятности Теория вероятностейаприорными, т. е. доопытными вероятностями, вероятности Теория вероятностейапостериорными, т. е. послео-пытными вероятностями.

Пример 3. Из урны, в которой находится 4 белых и 6 черных физически идентичных шаров, извлекли наугад один швр и положили его в урну, содержащую 5 белых и 4 черных шаров. Случайно извлеченный из второй урны шар оказался черным. Что более вероятно — из первой урны был извлечен черный шар или белый?

◄ Пусть событие Теория вероятностей состоит в том, что из первой урны извлечен белый шар, Теория вероятностей — из первой урны извлечен черный шар, Ч — из второй урны извлечен черный шар.

Очевидно, что

Теория вероятностей

Для оценки апостериорной вероятности Теория вероятностей воспользуемся формулой (7):

Теория вероятностей

Полученный результат позволяет считать гипотезу о том, что из первой урны был извлечен белый шар менее предпочтительной в сравнении с гипотезой, предполагающей извлечение из первой урны черного шара. ►

Для содержательного заключения о правдоподобности той или иной гипотезы важно, чтобы рассматриваемые события были действительно случайными в контексте рассматриваемых проблем. В противном случае выводы могут оказаться неадекватными реальному положению дел.

Пример 4. В одном из телевизионных шоу ведущий предлагает игроку выбрать один из стоящих перед ним ларцов, предупреждая, что только в одном из ларцов заключен ценный приз (скажем, ключи от автомобиля). После того как игрок произвел выбор, ведущий открывает один из оставшихся ларцов и, демонстрируя, что в нем ничего нет, предлагает игроку еще раз подумать и, если захочется, изменить свое решение — выбрать оставшийся ларец, вместо того, который был выбран первоначально. Имеет ли смысл игроку менять свое решение?

Теория вероятностей Ответ на вопрос задачи зависит от того, случаен или нет выбор ведущим одного из ларцов для демонстрации его содержимого.

Пусть выбор ведущего случаен. Обозначим через Теория вероятностей событие, состоящее в том, что ларец, выбранный игроком, содержит приз, через Теория вероятностей — что он приза не содержит, через А — что ларец, выбранный ведущим, пуст. Тогда по формуле полной вероятности заключаем, что

Теория вероятностей

А формула Бейеса (7) дает следующую оценку, например, для вероятности Теория вероятностей:

Теория вероятностей

и, следовательно, в этом случае игроку нет нужды менять свой выбор, так как шансы его на получение приза одинаковы, остановится ли он на своем первоначальном выборе или сменит его.

2. Пусть выбор ведущего не случаен, и он, зная где лежит приз, всегда открывает для всеобщего обозрения ларец, приза не содержащий, т. е. событие А не является случайным и происходит с вероятностью 1 В этом случае по формуле условной вероятности заключаем, что

Теория вероятностей

и в двух случаях из трех игроку выгоднее изменить свой выбор, чем настаивать на первоначальном ►

Теория вероятностей

Теория вероятностей – это математическая наука, изучающая закономерности случайных явлений.

Теория вероятностей — раздел математики, в котором изучаются общие закономерности случайных явлений массового характера независимо от их конкретной природы. Она разрабатывает методы количественной оценки влияния случайных факторов на различные явления. Знание этих закономерностей позволяют предвидеть, как эти события будут протекать в реальном опыте.

Элементы комбинаторики

Пусть дано множество Теория вероятностей задачи с решением, состоящее из Теория вероятностей задачи с решением элементов

Теория вероятностей задачи с решением

Перестановками на множестве из Теория вероятностей задачи с решением элементов называются всякие упорядоченные множества, состоящие из этих Теория вероятностей задачи с решением элементов. Количество всех перестановок на множестве из Теория вероятностей задачи с решением элементов обозначается Теория вероятностей задачи с решением и определяется по формуле

Теория вероятностей задачи с решением

Таким образом, перестановки одинаковы по составу элементов, но различаются порядком их перечисления.

Размещениями на множестве из Теория вероятностей задачи с решением элементов по Теория вероятностей задачи с решением элементов называются всякие упорядоченные подмножества, состоящие из Теория вероятностей задачи с решением элементов. Два различных размещения отличаются либо составом элементов, либо их порядком. Число размещений на множестве из Теория вероятностей задачи с решением элементов по Теория вероятностей задачи с решением элементов обозначается Теория вероятностей задачи с решением и определяется формулой

Теория вероятностей задачи с решением

Сочетаниями из Теория вероятностей задачи с решением различных элементов но Теория вероятностей задачи с решением элементов называется подмножество, состоящее из Теория вероятностей задачи с решением элементов, каждый из которых встречается один раз. Два различных сочетания отличаются хотя бы одним элементом. Число сочетаний па множестве из Теория вероятностей задачи с решением элементов по Теория вероятностей задачи с решением элементов обозначается Теория вероятностей задачи с решением и определяется формулой

Теория вероятностей задачи с решением

Если среди Теория вероятностей задачи с решением элементов одного вида есть Теория вероятностей задачи с решением, второго вида — Теория вероятностей задачи с решением и т.д., то, поменяв местами элементы одного вида, получим ту же перестановку. Поэтому число перестановок с повторениями определяется формулой

Теория вероятностей задачи с решением

где

Теория вероятностей задачи с решением

Число размещений на множестве из Теория вероятностей задачи с решением элементов по Теория вероятностей задачи с решением элементов с повторениями определяется формулой

Теория вероятностей задачи с решением

Возможно эта страница вам будет полезна:

Задача №1

Имеется множество, состоящее из 5 цифр Теория вероятностей задачи с решением. Сколько различных пятизначных чисел можно составить из этих цифр?

Решение:

Так как пятизначные числа отличаются только порядком следованием цифр в числе, то количество различных пятизначных чисел будет равно количеству перестановок на множестве из 5 элементов

Теория вероятностей задачи с решением

Задача №2

Студентам нужно сдать пять экзаменов за 20 дней. Сколькими способами можно составит ь расписание экзаменов.

Решение:

Расписание определяется датами (пять дат) проведения экзаменов и последовательностью дисциплин, по которым они проводятся. Поэтому число различных вариантов расписаний экзаменов будет равно количеству размещений па множестве из 20 элементов по 5 элементов

Теория вероятностей задачи с решением

Задача №3

Из команды, состоящей из 10 человек, выбирают 4 кандидатов для эстафеты 4×100 м. Сколькими способами это можно сделать?

Решение:

Число различных комбинаций из 10 членов команды для участия в эстафете

4 кандидатов будет равно количеству сочетаний на множестве из 10 элементов по 4 элемента

Теория вероятностей задачи с решением

Задача №4

Имеется слово КОЛОКОЛ. Сколько различных слов можно составить из букв этого слова?

Решение:

В слово буквы входят с повторениями. Поэтому количество различных перестановок определяется по формуле (1.4)

Теория вероятностей задачи с решением

Определении вероятности

Пусть проводится случайный эксперимент. Элементарным событием или исходом в случайном эксперименте называется всякая конкретная реализация этого эксперимента. Множество всех исходов эксперимента образует пространство элементарных исходов. Случайным событием называется всякое подмножество пространства элементарных исходов.

Исход называется благоприятствующим событию Теория вероятностей задачи с решением, если появление исхода влечет появление события Теория вероятностей задачи с решением.

Пусть случайный эксперимент имеет Теория вероятностей задачи с решением равновозможных элементарных исходов.

Классическое определение вероятности. Вероятностью события Теория вероятностей задачи с решением называется отношение числа исходов, благоприятствующих событию Теория вероятностей задачи с решением к общему числу всех единственно возможных и равновозможных элементарных исходов опыта

Теория вероятностей задачи с решением

где Теория вероятностей задачи с решением число исходов, благоприятствующих событию Теория вероятностей задачи с решением; Теория вероятностей задачи с решением число всех равновозможных исходов.

Относительной частотой события Теория вероятностей задачи с решением называется отношение числа испытаний, в которых наступило событие Теория вероятностей задачи с решением, к общему числу проведенных испытаний

Теория вероятностей задачи с решением

где Теория вероятностей задачи с решением — общее число проведенных испытаний; Теория вероятностей задачи с решением — число испытаний, в которых наступило событие Теория вероятностей задачи с решением.

При неограниченном увеличении числа испытаний относительная частота события Теория вероятностей задачи с решением стремится к вероятности наступления события в отдельном испытании. На этом факте основано статистическое определение вероятности, когда вероятности полагаются равными относительным частотам событий при большом Теория вероятностей задачи с решением.

Пусть имеется некоторая область Теория вероятностей задачи с решением на плоскости или в пространстве и другая область Теория вероятностей задачи с решением. В область Теория вероятностей задачи с решением случайным образом ставится точка. Нужно найти вероятность того, что она попадет в область Теория вероятностей задачи с решением. Все отборы положения точки в области Теория вероятностей задачи с решением считаются равновозможными. Геометрической вероятностью называется отношение меры области Теория вероятностей задачи с решением к мере области Теория вероятностей задачи с решением

Теория вероятностей задачи с решением

Свойства вероятности

  • Вероятность невозможного события Теория вероятностей задачи с решением равна О
Теория вероятностей задачи с решением
  • Вероятность достоверного события Теория вероятностей задачи с решением равна 1
Теория вероятностей задачи с решением
  • Для любого случайного события Теория вероятностей задачи с решением
Теория вероятностей задачи с решением
Теория вероятностей задачи с решением

Задача №5

Набирая номер телефона, абонент забыл последние две цифры и. помня лишь, что эти цифры различны, набрал их наудачу. Найти вероятность того, что набраны нужные цифры.

Решение:

Обозначим через Теория вероятностей задачи с решением событие — {набраны две нужные цифры]. Для определения вероятности события Теория вероятностей задачи с решением будем использовать классическое определение вероятности Теория вероятностей задачи с решением. Всего можно набрать столько различных цифр по две цифры, сколько может быть составлено размещений из десяти цифр по две Теория вероятностей задачи с решением. Благоприятствует событию В только одна пара цифр: Теория вероятностей задачи с решением. Тогда Теория вероятностей задачи с решением.

Задача №6

На девять вакантных мест претендуют 15 кандидатов, из них 7 женщин, остальные мужчины. Какова вероятность того, что из девяти случайно отобранных кандидатов ровно пять женщин.

Решение:

Пусть событие Теория вероятностей задачи с решением состоит в том, что из 9 отобранных кандидатов 5 женщин. Для решения используем классическое определение вероятности. Общее число исходов будет равно числу способов, которыми можно выбрать 9 человек из 15 кандидатов

Теория вероятностей задачи с решением

Число благоприятствующих исходов

Теория вероятностей задачи с решением

Задача №7

В квадрат со стороной Теория вероятностей задачи с решением случайным образом ставится точка. Какова вероятность того, что эта точка попадет в круг, вписанный в этот квадрат.

Решение:

Пусть событие Теория вероятностей задачи с решением состоит в том, что {точка попадет в круг}. Для определения вероятности события Теория вероятностей задачи с решением используем геометрическую вероятность

Теория вероятностей задачи с решением

Теоремы сложения и умножения вероятностей

Теорема сложения. Вероятность суммы двух событий равна сумме вероятностей этих событий без вероятности их совместного появления

Теория вероятностей задачи с решением

Если события Теория вероятностей задачи с решением и Теория вероятностей задачи с решением несовместные, то вероятность суммы несовместных событий равна сумме вероятностей этих событий

Теория вероятностей задачи с решением

Суммой двух событий называется событие, состоящее из элементарных исходов, благоприятствующих либо первому событию, либо второму, либо обоим событиям.

Два события называются несовместными, если они не имеют общих исходов.

Произведением двух событий называется событие, состоящее из элементарных исходов, благоприятствующих и первому, и второму событиям.

Два события называются независимыми, если вероятность появления одного события не зависит от того, произошло или не произошло второе событие.

Условной вероятностью Теория вероятностей задачи с решением называют вероятность события Теория вероятностей задачи с решением, вычисленную в предположении, что событие Теория вероятностей задачи с решением произошло.

Теорема умножения вероятностей. Вероятность произведения двух событий равна произведению вероятностей одного события на условную вероятность второго события при условии, что произошло первое событие

Теория вероятностей задачи с решением

Если события Теория вероятностей задачи с решением и Теория вероятностей задачи с решением независимые, то вероятность произведения двух событий равна произведению вероятностей этих событий

Теория вероятностей задачи с решением

Задача №8

Найти вероятность того, что случайно взятое двузначное число будет кратным двум или пяти.

Решение:

Пусть событие Теория вероятностей задачи с решением состоит в том, что {случайно взятое число будет кратным двум или пяти}; Теория вероятностей задачи с решением — событие, состоящее в том, что {число, кратное двум}; Теория вероятностей задачи с решением — событие, состоящее в том, что {число, кратное пяти}. События Теория вероятностей задачи с решением и Теория вероятностей задачи с решением являются совместными, так как есть числа, которые одновременно делятся на два и пять. Так как Теория вероятностей задачи с решением, то Теория вероятностей задачи с решением. Вычислим вероятности этих событий, воспользовавшись классическим определением вероятности

Теория вероятностей задачи с решением

Тогда

Теория вероятностей задачи с решением

Задача №9

Для подготовки к экзамену студентам дано 60 вопросов. Студент, идя на экзамен, выучил 50 вопросов. Найти вероятность того, что студент сдаст экзамен, если для сдачи экзамена студенту нужно ответить на два вопроса из двух заданных.

Решение:

Пусть событие Теория вероятностей задачи с решением состоит в том, что студент сдаст экзамен. Событие Теория вероятностей задачи с решением = {студент ответил на первый вопрос}, Теория вероятностей задачи с решением = {студент ответил на второй вопрос}. Тогда Теория вероятностей задачи с решением. События Теория вероятностей задачи с решением и Теория вероятностей задачи с решением — зависимые. Применяя теорему умножения вероятностей, мы получаем

Теория вероятностей задачи с решением

Найдем вероятности событий, воспользовавшись классическим определением вероятности

Теория вероятностей задачи с решением

Задача №10

Стрелок делает независимо друг от друга два выстрела по мишеням. Вероятность попадания в мишень при первом выстреле равна 0,7, при втором — 0.9. Найти вероятность того, что при двух выстрелах будет только одно попадание в мишень.

Решение:

Пусть событие Теория вероятностей задачи с решением состоит в том, что {будет только одно попадание при двух выстрелах}, событие Теория вероятностей задачи с решением состоит в том, что {будет попадание при первом выстреле}, событие Теория вероятностей задачи с решением = {попадание при втором выстреле}.

Теория вероятностей задачи с решением

Тогда

Теория вероятностей задачи с решением

Формула полной вероятности. Формулы Баиеса

Пусть событие Теория вероятностей задачи с решением может произойти вместе с одним из событий Теория вероятностей задачи с решением. События Теория вероятностей задачи с решением образуют полную группу попарно несовместных событий, если они: 1) попарно несовместны; Теория вероятностей задачи с решением; 2) сумма событий Теория вероятностей задачи с решением является достоверным событием, то есть Теория вероятностей задачи с решением.

Теорема 4.1. Пусть событие Теория вероятностей задачи с решением может произойти совместно с одним из событий Теория вероятностей задачи с решением которые образуют полную группу попарно несовместных событий. Тогда вероятность события Теория вероятностей задачи с решением определяется по формуле полной вероятности

Теория вероятностей задачи с решением

События Теория вероятностей задачи с решением называются гипотезами.

Теорема 4.2. Пусть событие Теория вероятностей задачи с решением может произойти совместно с одной из гипотез Теория вероятностей задачи с решением Если событие Теория вероятностей задачи с решением произошло, то вероятности появления гипотез вычисляются по формулам Байеса

Теория вероятностей задачи с решением

Задача №11

Электролампы изготавливаются на трех заводах. Первый завод изготавливает 45% общего количества электроламп, второй — 40%, третий — 15%. Продукция первого завода содержит 70% стандартных электроламп, второго — 80%, третьего — 81%. Найти вероятность того, что случайно взятая электролампа будет стандартной.

Решение:

Пусть событие Теория вероятностей задачи с решением состоит в том. что {случайно взятая лампа стандартна). Введем гипотезы Теория вероятностей задачи с решением [лампа произведена на Теория вероятностей задачи с решением заводе]. Вероятность события Теория вероятностей задачи с решением определяется по формуле полной вероятности

Теория вероятностей задачи с решением

Найдем вероятности гипотез:

Теория вероятностей задачи с решением

Условные вероятности будут равны:

Теория вероятностей задачи с решением

Подставив в формулу полной вероятности, получим

Теория вероятностей задачи с решением

Задача №12

В пирамиде 10 винтовок, из них 6 снабжены оптическим прицелом, а остальные винговки — с обыкновенным прицелом. Вероятность попадания в цель из винтовки с оптическим прицелом равна 0,9; из обыкновенной винтовки — 0,7. Стрелок поразил цель из случайно взятой винтовки. Какова вероятность того, что он стрелял из обычной винтовки.

Решение:

Пусть событие Теория вероятностей задачи с решением состоит в том, что стрелок поразил цель, событие Теория вероятностей задачи с решением = {стрелял из обыкновенной винтовки}, событие Теория вероятностей задачи с решением = {из винтовки с оптическим прицелом}.

Теория вероятностей задачи с решением

Из условия задачи

Теория вероятностей задачи с решением

Схема повторных независимых испытаний (схема Бернулли)

Схемой Бернулли называется последовательность из Теория вероятностей задачи с решением независимых испытаний, в каждом из которых возможны только два исхода: событие Теория вероятностей задачи с решением может наступить или не наступить, и вероятность появления события Теория вероятностей задачи с решением в каждом испытании постоянна.

Формула Бернулли. Вероятность того, что в Теория вероятностей задачи с решением независимых испытаниях, в каждом из которых вероятность появления события Теория вероятностей задачи с решением равна Теория вероятностей задачи с решением, событие Теория вероятностей задачи с решением наступит ровно к раз, равна

Теория вероятностей задачи с решением

где

Теория вероятностей задачи с решением

Локальная теорема Муавра-Лапласа. Вероятность того, что в Теория вероятностей задачи с решением независимых испытаниях, в каждом из которых вероятность появления события равна Теория вероятностей задачи с решением, событие наступит ровно Теория вероятностей задачи с решением раз, приближенно равна 1

Теория вероятностей задачи с решением

где

Теория вероятностей задачи с решением

Значения функции Теория вероятностей задачи с решением находятся по таблице по вычисленным значениям Теория вероятностей задачи с решением. Интегральная теорема Муавра-Лапласа. Вероятность того, что в Теория вероятностей задачи с решением независимых испытаниях, в каждом из которых вероятность появления события Теория вероятностей задачи с решением равна Теория вероятностей задачи с решением событие Теория вероятностей задачи с решением наступит от Теория вероятностей задачи с решением до Теория вероятностей задачи с решением раз, приближенно равна

Теория вероятностей задачи с решением

где

Теория вероятностей задачи с решением

Значения функции Теория вероятностей задачи с решением находят по таблице по вычисленным значениям Теория вероятностей задачи с решением. Формула Пуассона. Если в схеме Бернулли число испытаний велико, а вероятность появления события Теория вероятностей задачи с решением мала, то вероятность того, что в Теория вероятностей задачи с решением независимых испытаниях событие Теория вероятностей задачи с решением наступит ровно Теория вероятностей задачи с решением раз приближенно равна

Теория вероятностей задачи с решением

Вероятность отклонения относительной частоты от постоянной вероятности.

Вероятность того, что в Теория вероятностей задачи с решением независимых испытаниях, в каждом из которых вероятность появления события Теория вероятностей задачи с решением равна Теория вероятностей задачи с решением, абсолютная величина отклонения относительной частоты от вероятности появления события Теория вероятностей задачи с решением не превосходит положительного числа Теория вероятностей задачи с решением, приближенно равна

Теория вероятностей задачи с решением

Наивероятнейшее число появлений события в независимых испытаниях

Число Теория вероятностей задачи с решением называют наивероятнейшим, если вероятность того, что в Теория вероятностей задачи с решением независимых испытаниях событие Теория вероятностей задачи с решением наступит ровно Теория вероятностей задачи с решением раз не меньше вероятностей остальных возможных значений Теория вероятностей задачи с решением.

Наивероятнейшее число определяется из неравенства

Теория вероятностей задачи с решением

причем:

а) если число Теория вероятностей задачи с решением дробное, то существует одно наивероятнейшее число; Теория вероятностей задачи с решением, где Теория вероятностей задачи с решением — целая часть числа Теория вероятностей задачи с решением,

б) сели число Теория вероятностей задачи с решением — целое, то существуют два наивероятнейших числа Теория вероятностей задачи с решением и Теория вероятностей задачи с решением;

в) если Теория вероятностей задачи с решением — целое, то Теория вероятностей задачи с решением.

Задача №13

Прибор состоит из четырех узлов. Вероятность безотказной работы каждого узла равна 0,8. Узлы выходят из строя независимо друг от друга. Найти вероятность того, что выйдут из строя росно два узла.

Решение:

Для решения задачи используем формулу Бернулли.

Теория вероятностей задачи с решением

Задача №14

Вероятность поражения мишени при одном выстреле равна 0,8. Найти вероятность того, что при 100 выстрелах мишень будет поражена 75 раз.

Решение:

Решаем задачу с использованием локальной теоремы Лапласа.

Теория вероятностей задачи с решением

Задача №15

В гараже имеется 100 автомашин. Вероятность того, что в течение рабочего дня машина находится вне гаража, равна 0,8. Найти вероятность того, что вне гаража будут находиться от 70 до 85 машин.

Решение:

Для решения используем интегральную теорему Муавра-Лапласа. По условию задачи

Теория вероятностей задачи с решением

тогда

Теория вероятностей задачи с решением

Функция распределения и плотность распределения случайных величин

Краткие теоретические сведения

Случайной величиной Теория вероятностей задачи с решением называется действительная функция Теория вероятностей задачи с решением, определенная на пространстве элементарных исходов Теория вероятностей задачи с решением и такая, что при любых действительных .v определена вероятность события Теория вероятностей задачи с решением.

Функцией распределения вероятностей называется функция Теория вероятностей задачи с решением, равная вероятности того, что Теория вероятностей задачи с решением

Теория вероятностей задачи с решением

Функция распределения обладает следующими свойствами:

  1. Теория вероятностей задачи с решением
  2. Теория вероятностей задачи с решением
  3. Теория вероятностей задачи с решением — неубывающая функция.
  4. Теория вероятностей задачи с решением — непрерывная слева, т.е. Теория вероятностей задачи с решением.
  5. Вероятность попадания Теория вероятностей задачи с решением в интервал Теория вероятностей задачи с решением определяется формулой
Теория вероятностей задачи с решением

Теория вероятностей задачи с решением называется дискретной, если она принимает конечное или счетное количество значений.

Теория вероятностей задачи с решением называется непрерывной на Теория вероятностей задачи с решением, если она принимает все значения из этого интервала.

Законом распределения дискретной Теория вероятностей задачи с решением называется соответствие, но которому каждому возможному значению Теория вероятностей задачи с решением Теория вероятностей задачи с решением ставится в соответствие вероятность его появления Теория вероятностей задачи с решением. Закон распределения дискретной Теория вероятностей задачи с решением записывается в виде таблицы.

Теория вероятностей задачи с решением

Плотностью распределения называется функция Теория вероятностей задачи с решением, удовлетворяющая условию

Теория вероятностей задачи с решением

Плотность распределения обладает следующими свойствами:

Теория вероятностей задачи с решением

Теория вероятностей задачи с решением

Чтобы задать закон распределения непрерывной Теория вероятностей задачи с решением, нужно задать либо плотность распределения, либо функцию распределения.

Задача №16

Закон распределения дискретной Теория вероятностей задачи с решением имеет вид

Теория вероятностей задачи с решением

Найти функцию распределения.

Решение:

По определению Теория вероятностей задачи с решением. Тогда

Теория вероятностей задачи с решением

Задача №17

Непрерывная Теория вероятностей задачи с решением задана плотностью распределения

Теория вероятностей задачи с решением

Нужно определить значение параметра Теория вероятностей задачи с решением и найти Теория вероятностей задачи с решением.

Решение:

Для определения параметра Теория вероятностей задачи с решением воспользуемся свойством плотности распределения

Теория вероятностей задачи с решением
Теория вероятностей задачи с решением

Функцию распределения определим из соотношения Теория вероятностей задачи с решением.

  1. Если Теория вероятностей задачи с решением, то Теория вероятностей задачи с решением.
  2. Если Теория вероятностей задачи с решением. то Теория вероятностей задачи с решением
  3. Если Теория вероятностей задачи с решением, то Теория вероятностей задачи с решением

Таким образом,

Теория вероятностей задачи с решением

Задача №18

Дана функция распределения Теория вероятностей задачи с решением

Теория вероятностей задачи с решением

Требуется найти плотность распределения и вероятность попадания Теория вероятностей задачи с решением в интервал

Теория вероятностей задачи с решением

Решение:

Вероятность попадания Теория вероятностей задачи с решением в интервал Теория вероятностей задачи с решением определяется по формуле

Теория вероятностей задачи с решением

Если известна функция распределенияТеория вероятностей задачи с решением, то Теория вероятностей задачи с решением

Теория вероятностей задачи с решением

Числовые характеристики случайных величин

Пусть дискретная Теория вероятностей задачи с решением имеет следующий закон распределения

Теория вероятностей задачи с решением

Математическим ожиданием Теория вероятностей задачи с решением называется сумма произведений всех возможных значений Теория вероятностей задачи с решением на соответствующие вероятности

Теория вероятностей задачи с решением

Математическое ожидание обладает следующими свойствами:

Теория вероятностей задачи с решением

Математическое ожидание характеризует среднее значение Теория вероятностей задачи с решением.

Для непрерывной Теория вероятностей задачи с решением математическое ожидание вычисляется по формуле

Теория вероятностей задачи с решением

Начальным моментом Теория вероятностей задачи с решением-го порядка называется математическое ожидание Теория вероятностей задачи с решением, т.е. Теория вероятностей задачи с решением. Начальные моменты Теория вероятностей задачи с решением-го порядка для дискретных и непрерывных Теория вероятностей задачи с решением вычисляются соответственно по формулам

Теория вероятностей задачи с решением

Центральным моментом Теория вероятностей задачи с решением-го порядка называется математическое ожидание Теория вероятностей задачи с решением

Теория вероятностей задачи с решением

Для дискретных и непрерывных Теория вероятностей задачи с решением центральный момент Теория вероятностей задачи с решением-го порядка вычисляется по формулам:

Теория вероятностей задачи с решением

Дисперсией Теория вероятностей задачи с решением называется центральный момент второго порядка

Теория вероятностей задачи с решением

Дисперсия характеризует степень разброса значений Теория вероятностей задачи с решением относительно математического ожидания. Дисперсия обладает следующими свойствами:

Теория вероятностей задачи с решением

Дисперсия Теория вероятностей задачи с решением равна разности математического ожидания квадрата Теория вероятностей задачи с решением и квадрата математического ожидания

Теория вероятностей задачи с решением

Средним квадратическим ожиданием Теория вероятностей задачи с решением называется корень квадратный из дисперсии

Теория вероятностей задачи с решением

Задача №19

Дискретная Теория вероятностей задачи с решением задана законом распределения

Теория вероятностей задачи с решением

Вычислить

Теория вероятностей задачи с решением

Решение:

Теория вероятностей задачи с решением

Дисперсию вычислим по формуле

Теория вероятностей задачи с решением

Задача №20

Непрерывная Теория вероятностей задачи с решением задана функцией распределения

Теория вероятностей задачи с решением

Вычислить

Теория вероятностей задачи с решением

Решение:

Найдем плотность распределения

Теория вероятностей задачи с решением

Вычислим математическое ожидание

Теория вероятностей задачи с решением

Дисперсия определяется по формуле

Теория вероятностей задачи с решением

Законы распределения дискретных случайных величин

Дискретная Теория вероятностей задачи с решением называется распределенной по биномиальному закону, если она принимает конечное число значений Теория вероятностей задачи с решением с вероятностями, которые определяются по формуле Бернулли

Теория вероятностей задачи с решением

Для дискретной Теория вероятностей задачи с решением, распределенной по биномиальному закону, справедливы следующие соотношения

Теория вероятностей задачи с решением

Дискретная Теория вероятностей задачи с решением называется распределенной по закону Пуассона, если она принимает счетное число значений Теория вероятностей задачи с решением с вероятностями, которые определяются по формуле Пуассона

Теория вероятностей задачи с решением

Для дискретной Теория вероятностей задачи с решением, распределенной по закону Пуассона справедливы соотношения

Теория вероятностей задачи с решением

Задача №21

О сигнализации о пожаре установлено три независимо работающих устройства. Вероятность того, что при пожаре сработает каждое устройство постоянна и равна 0,9. Теория вероятностей задачи с решением равна количеству срабатывающих устройств при пожаре. Требуется составить закон распределения Теория вероятностей задачи с решением и вычислить Теория вероятностей задачи с решением.

Решение:

Теория вероятностей задачи с решением принимает значение 0; 1; 2; 3. Определим вероятности Теория вероятностей задачи с решением по формуле (8.1).

Теория вероятностей задачи с решением

Проверка:

Теория вероятностей задачи с решением

Закон распределения Теория вероятностей задачи с решением имеет вид

Теория вероятностей задачи с решением

Вычислим

Теория вероятностей задачи с решением

Законы распределения непрерывных случайных величин

Непрерывная Теория вероятностей задачи с решением называется равномерно распределенной на Теория вероятностей задачи с решением, если плотность распределения вероятностей имеет вид

Теория вероятностей задачи с решением

Для Теория вероятностей задачи с решением. равномерно распределенной на Теория вероятностей задачи с решением, справедливы следующие соотношения:

Теория вероятностей задачи с решением

Непрерывная Теория вероятностей задачи с решением называется распределенной но показательному закону, если плотность распределения вероятностей имеет вид

Теория вероятностей задачи с решением

Для Теория вероятностей задачи с решением, распределенной по показательному закону, справедливы следующие соотношения

Теория вероятностей задачи с решением

Функция

Теория вероятностей задачи с решением

определяет вероятность отказа за время Теория вероятностей задачи с решением.

Вероятность безотказной работы за это время будет равна

Теория вероятностей задачи с решением

Функцию Теория вероятностей задачи с решением называют функцией надежности.

Непрерывная Теория вероятностей задачи с решением называется распределенной по нормальному закону, если плотность распределения вероятностей имеет вид

Теория вероятностей задачи с решением

Для нормально распределенной Решение задач по теории вероятностей справедливы следующие соотношения:

Решение задач по теории вероятностей

Задача №22

Решение задач по теории вероятностей распределена равномерно на (3;5). Требуется найти:

Решение задач по теории вероятностей

Решение:

На основании формул (9.1) и (9.2) имеем

Решение задач по теории вероятностей

Задача №23

Решение задач по теории вероятностей распределенная по показательному закону, имеет функцию распределения вида

Решение задач по теории вероятностей

Вычислить

Решение задач по теории вероятностей

Решение:

Согласно формуле (9.4) Решение задач по теории вероятностей. Тогда

Решение задач по теории вероятностей

Задача №24

Решение задач по теории вероятностей распределена по нормальному закону с параметрами

Решение задач по теории вероятностей

Требуется: 1) записать Решение задач по теории вероятностей и Решение задач по теории вероятностей; 2) вычислить

Решение задач по теории вероятностей

Решение:

Согласно формулам (9.5) и (9.6) имеем

Решение задач по теории вероятностей

Предельные теоремы теории вероятностей

Неравенство Чебышева. Вероятность того, что отклонение Решение задач по теории вероятностей от ее математического ожидания по модулю меньше данного числа Решение задач по теории вероятностей не менее, чем Решение задач по теории вероятностей

Решение задач по теории вероятностей

Теорема Чебышева. Пусть даны Решение задач по теории вероятностей, которые попарно независимы, имеют математические ожидания Решение задач по теории вероятностей и дисперсии, ограниченные одним и тем же числом Решение задач по теории вероятностей. Тогда для любого числа Решение задач по теории вероятностей выполняется неравенство

Решение задач по теории вероятностей

Если Решение задач по теории вероятностей имеют одно и то же математическое ожидание Решение задач по теории вероятностей, то неравенство (10.2) примет вид

Решение задач по теории вероятностей

Переходя в неравенство (10.3) к пределу при Решение задач по теории вероятностей, получим

Решение задач по теории вероятностей

В этом случае говорят, что при Решение задач по теории вероятностей последовательность Решение задач по теории вероятностей сходится по вероятности к своему математическому ожиданию Решение задач по теории вероятностей.

Теорема Бернулли. Если в каждом из Решение задач по теории вероятностей независимых испытаний вероятность Решение задач по теории вероятностей появления события Решение задач по теории вероятностей постоянна, то вероятность того, что отклонение относительной частоты Решение задач по теории вероятностей от вероятности Решение задач по теории вероятностей по модулю не превзойдет положительного числа Решение задач по теории вероятностей больше чем разность Решение задач по теории вероятностей

Решение задач по теории вероятностей

Переходя в неравенство (10.5) к пределу при Решение задач по теории вероятностей, получим

Решение задач по теории вероятностей

При большом числе испытаний относительная частота Решение задач по теории вероятностей события Решение задач по теории вероятностей сходится по вероятности к вероятности Решение задач по теории вероятностей появления события в отдельном испытании.

Центральная предельная теорема Ляпунова. Пусть Решение задач по теории вероятностей последовательность независимых Решение задач по теории вероятностей для каждой из которых существует математическое ожидание Решение задач по теории вероятностей и дисперсия Решение задач по теории вероятностей, центральный момент третьего порядка

Решение задач по теории вероятностей

и выполняется условие Ляпунова

Решение задач по теории вероятностей

Тогда при Решение задач по теории вероятностей распределение Решение задач по теории вероятностей стремится к нормальному закону с функцией распределения

Решение задач по теории вероятностей

Задача №25

Средняя длина детали равна 50 см, а дисперсия длины равна 0,1. Оценить вероятность того, что изготовленная деталь окажется по своей длине не менее 49,5 см. и не более 50,5 см.

Решение:

По условию задачи

Решение задач по теории вероятностей

Так как Решение задач по теории вероятностей непрерывна, то

Решение задач по теории вероятностей

Применяя неравенство (10.1), получим

Решение задач по теории вероятностей

Задача №26

При штамповке деталей брак составляет 3%. Найти вероятность того, что при проверке партии из 1000 деталей выявится отклонение от установленного процента брака меньше, чем на 1%.

Решение:

По условию задачи

Решение задач по теории вероятностей

Воспользуемся неравенством (10.5)

Решение задач по теории вероятностей

Задача №27

Складываются 48 попарно независимых Решение задач по теории вероятностей, распределенных по равномерному закону на интервале (0; 1). Записать приближенно функцию распределения суммы этих Решение задач по теории вероятностей. Найти вероятность того, что эта сумма будет заключена в пределах от 26 до 28.

Решение:

Решение задач по теории вероятностей

Обозначим

Решение задач по теории вероятностей
Решение задач по теории вероятностей

Тогда

Решение задач по теории вероятностей

и функция распределения Решение задач по теории вероятностей имеет вид

Решение задач по теории вероятностей

Найдем вероятность попадания Решение задач по теории вероятностей в интервал (26; 28).

Решение задач по теории вероятностей

Двумерные случайные величины. Законы распределения. Условные законы распределения

Двумерной Решение задач по теории вероятностей называется совокупность двух случайных величин Решение задач по теории вероятностей, описывающих тот или иной случайный эксперимент. Решение задач по теории вероятностей и Решение задач по теории вероятностей называются составляющими.

Если составляющие двумерной Решение задач по теории вероятностей являются дискретными Решение задач по теории вероятностей. то двумерная Решение задач по теории вероятностей называется дискретной, если составляющие являются непрерывными Решение задач по теории вероятностей. то двумерная Решение задач по теории вероятностей называется непрерывной. Если одна из составляющих является дискретной, а вторая — непрерывной, то двумерная величина называется сметанной.

Законом распределения дискретной двумерной Решение задач по теории вероятностей называется соответствие между всевозможными парами Решение задач по теории вероятностей и вероятностями их появления Решение задач по теории вероятностей. Закон распределения дискретных двумерных Решение задач по теории вероятностей задается в виде таблицы

Решение задач по теории вероятностей

Если известен закон распределения двумерной дискретной Решение задач по теории вероятностей, то законы распределения составляющих находятся следующим образом

Решение задач по теории вероятностей

Функцией распределения двумерной Решение задач по теории вероятностей называется вероятность события

Решение задач по теории вероятностей

Функция распределения вероятностей двумерной Решение задач по теории вероятностей обладает следующими свойствами:

  1. Решение задач по теории вероятностей
  2. Решение задач по теории вероятностей
  3. Решение задач по теории вероятностей
  4. Решение задач по теории вероятностей, где Решение задач по теории вероятностей — функции распределения составляющих Решение задач по теории вероятностей и Решение задач по теории вероятностей.
  5. Функция распределения Решение задач по теории вероятностей является не убывающей функцией по каждому из своих аргументов.

Функция Решение задач по теории вероятностей называется плотностью распределения вероятностей двумерной Решение задач по теории вероятностей, если она удовлетворяет соотношению

Решение задач по теории вероятностей

Плотность распределения вероятностей двумерной Решение задач по теории вероятностей обладает следующими свойствами:

Решение задач по теории вероятностей

Чтобы задать закон распределения непрерывной двумерной Решение задач по теории вероятностей, достаточно задать либо функцию распределения, либо плотность распределения.

Условным законом распределения называется закон распределения одной из составляющих при условии, что вторая составляющая приняла определенное значение. Для дискретных двумерных Решение задач по теории вероятностей условные вероятности определяются по формулам:

Решение задач по теории вероятностей

Условные плотности распределения находятся по формулам:

Решение задач по теории вероятностей

где Решение задач по теории вероятностей — плотности распределения составляющих Решение задач по теории вероятностей.

Составляющие Решение задач по теории вероятностей и Решение задач по теории вероятностей двумерной Решение задач по теории вероятностей называются независимыми, если

Решение задач по теории вероятностей

Задача №28

Двумерная дискретная Решение задач по теории вероятностей задана законом распределения

Решение задач по теории вероятностей

Требуется найти законы распределения составляющих и условный закон распределения составляющей Решение задач по теории вероятностей при условии, что Решение задач по теории вероятностей = 1.

Решение:

Законы распределения составляющих Решение задач по теории вероятностей и Решение задач по теории вероятностей найдем с использованием формул (11.1).

Решение задач по теории вероятностей

Тогда закон распределения составляющих Решение задач по теории вероятностей имеет вид

Решение задач по теории вероятностей

Аналогично находится закон распределения составляющей Решение задач по теории вероятностей.

Решение задач по теории вероятностей

Условный закон распределения составляющей Решение задач по теории вероятностей при условии, что Решение задач по теории вероятностей = 1, найдем с использованием формул (11.3).

Решение задач по теории вероятностей

Задача №29

Дана функция распределении двумерной Решение задач по теории вероятностей

Решение задач по теории вероятностей

Требуется найти плотность распределения Решение задач по теории вероятностей и условные плотности распределения

Решение задач по теории вероятностей

Решение:

Плотность распределения найдем, используя свойство 3 плотности распределения

Решение задач по теории вероятностей

Плотности распределения составляющих найдем, используя свойство 5 плотности распределения

Решение задач по теории вероятностей

Условные плотности распределения составляющих найдем с использованием формул (11.4)

Решение задач по теории вероятностей

Так как условные плотности распределения вероятностей совпадают с плотностями распределения составляющих, то составляющие являются независимыми Решение задач по теории вероятностей.

Числовые характеристики двумерных случайных величин. Коэффициент корреляции

Начальным моментом порядка Решение задач по теории вероятностей двумерной Решение задач по теории вероятностей называется математическое ожидание произведения Решение задач по теории вероятностей

Решение задач по теории вероятностей

Для непрерывных Решение задач по теории вероятностей

Решение задач по теории вероятностей

для дискретных

Решение задач по теории вероятностей

Центральным моментом порядка Решение задач по теории вероятностей двумерной Решение задач по теории вероятностей называется математическое ожидание произведения

Решение задач по теории вероятностей
Решение задач по теории вероятностей

Для непрерывной двумерной Решение задач по теории вероятностей центральный момент порядка Решение задач по теории вероятностей вычисляется по формуле

Решение задач по теории вероятностей

для дискретных Решение задач по теории вероятностей

Решение задач по теории вероятностей

Корреляционным моментом двумерной Решение задач по теории вероятностей называется центральный момент Решение задач по теории вероятностей. Для непрерывной Решение задач по теории вероятностей корреляционный момент вычисляется по формуле

Решение задач по теории вероятностей

для дискретных

Решение задач по теории вероятностей

Корреляционный момент характеризует тесноту связи между составляющими Решение задач по теории вероятностей и Решение задач по теории вероятностей. Коэффициентом корреляции Решение задач по теории вероятностей Решение задач по теории вероятностей и Решение задач по теории вероятностей называется отношение корреляционного момента к произведению средних квадратических отклонений составляющих

Решение задач по теории вероятностей

Коэффициент корреляции обладает следующими свойствами:

Решение задач по теории вероятностей

Если зависимость между Решение задач по теории вероятностей и Решение задач по теории вероятностей отсутствует, то Решение задач по теории вероятностей. Если Решение задач по теории вероятностей, то зависимость между Решение задач по теории вероятностей и Решение задач по теории вероятностей линейная. Решение задач по теории вероятностейи Решение задач по теории вероятностей, для которых Решение задач по теории вероятностей называются некоррелированными. Очевидно, что независимые Решение задач по теории вероятностей не коррелированы. Обратное утверждение верно лишь при условии нормального распределения двумерной Решение задач по теории вероятностей. Коэффициент корреляции вычисляется по формуле

Решение задач по теории вероятностей

Задача №30

Двумерная Решение задач по теории вероятностей задана таблицей

Решение задач по теории вероятностей

Вычислить коэффициент корреляции.

Решение:

Составим законы распределения составляющих

Решение задач по теории вероятностей

Вычислим математические ожидания и средние квадратические отклонения составляющих

Решение задач по теории вероятностей

Решение задач по теории вероятностей

Вычислим коэффициент корреляции по формуле (12.9)

Решение задач по теории вероятностей

Составляющие Решение задач по теории вероятностей и Решение задач по теории вероятностей являются некоррелированными Решение задач по теории вероятностей. Очевидно, что независимые Решение задач по теории вероятностей не коррелированы. Обратное утверждение верно лишь при условии нормального распределения двумерной Решение задач по теории вероятностей.

Задача №31

Непрерывная двумерная Решение задач по теории вероятностей задана плотностью распределения

Решение задач по теории вероятностей

Найти коэффициент корреляции.

Решение:

Найдем математические ожидания составляющих

Решение задач по теории вероятностей

Найдем дисперсии

Решение задач по теории вероятностей

Вычислим корреляционный момент

Решение задач по теории вероятностей

Коэффициент корреляции вычислим по формуле (12.8)

Решение задач по теории вероятностей

Статистическое распределение. Эмпирическая функция распределения и ее свойства. Полигон и гистограмма. Числовые характеристики выборки

Генеральной совокупностью называется совокупность элементов, объединенных по некоторому признаку, из которых производится выборка.

Выборочной совокупностью или выборкой называется совокупность объектов, случайно выбранных для исследования.

Объемом выборки называется количество объектов, входящих в выборку.

Пусть из совокупности извлечена выборка объемом п.

Выборочная совокупность, расположенная по возрастанию или убыванию значения признака, называется вариационным рядом, а сс объекты — вариантами.

Если значения вариант совпадают или отличаются незначительно, то их можно сгруппировать, придав частоту каждой варианте.

В результате получим сгруппированный вариационный ряд.

Частостью или относительной частотой варианты называется отношение частоты варианты к объему выборки

Решение задач по теории вероятностей

Статистическим распределением называется соответствие, по которому каждому возможному значению варианты ставится в соответствие частота (относительная частота) се появления. Статистическое распределение записывается в виде таблицы, в которой в первой строке перечислены все значения вариант, а во второй частоты или частости, которые соответствуют вариантам

Решение задач по теории вероятностей

Для построения интервального статистического ряда разбивают множество вариант на полуинтервалы Решение задач по теории вероятностей. т.е. производят группировку. Рекомендуется число интервалов Решение задач по теории вероятностей определять по формуле

Решение задач по теории вероятностей

Длина интервала равна

Решение задач по теории вероятностей

Для наглядности используются графические изображения вариционных рядов в виде полигона и гистограммы.

Полигоном частот или частостей называется ломаная линия, соединяющая точки с координатами

Решение задач по теории вероятностей

Гистограммой частот или частостей называют ступенчатую фигуру, составленную из прямоугольников с основанием Решение задач по теории вероятностей и высотой

Решение задач по теории вероятностей

Эмпирической функцией распределения называют функцию Решение задач по теории вероятностей, определяющую для каждого значения Решение задач по теории вероятностей относительную частоту события Решение задач по теории вероятностей:

Решение задач по теории вероятностей

где Решение задач по теории вероятностей — число вариант (с учетом их кратностей) меньших Решение задач по теории вероятностей — объем выборки. Эмпирическая функция распределения обладает следующими свойствами:

  1. Значения эмпирической функции принадлежат отрезку [0; l],
  2. Эмпирическая функция является неубывающей функцией.
  3. Если Решение задач по теории вероятностей наименьшее значение варианты, а Решение задач по теории вероятностей наибольшее значение варианты, то
Решение задач по теории вероятностей

Для описания выборки применяются такие числовые характеристики, как выборочная средняя, выборочная дисперсия, выборочное среднее квадратическое отклонение.

Выборочной средней называется среднее значение варианты, вычисленное по данным выборки

Решение задач по теории вероятностей

где Решение задач по теории вероятностей — частота варианты Решение задач по теории вероятностей.

Выборочной дисперсией называется дисперсия, вычисленная по данным выборки

Решение задач по теории вероятностей

Выборочная дисперсия равна разности между средним значением квадрата вариант и квадратом выборочного среднего

Решение задач по теории вероятностей

Выборочным средним квадратическим отклонением называется корень квадратный из выборочной дисперсии

Решение задач по теории вероятностей

Задача №32

По данному распределению выборки найти эмпирическую функцию распределения и построить полигон частот

Решение задач по теории вероятностей

.

Решение:

Определим объем выборки

Решение задач по теории вероятностей

Определим относительные частоты вариант

Решение задач по теории вероятностей

Решение задач по теории вероятностей

Запишем эмпирическую функцию распределения

Решение задач по теории вероятностей

Построим полигон частот

Решение задач по теории вероятностей

Задача №33

Построить гистограмму частостей по данным выборки объема 100 и вычислить числовые характеристики выборки.

Решение задач по теории вероятностей

Решение:

Вычислим относительные частоты по формуле

Решение задач по теории вероятностей

и найдем высоты прямоугольников по формуле

Решение задач по теории вероятностей

Вычисления сведем в таблицу

Решение задач по теории вероятностей

Построим гистограмму частостей

Решение задач по теории вероятностей

Вычислим числовые характеристики выборки

Решение задач по теории вероятностей

Вычислим

Решение задач по теории вероятностей
Решение задач по теории вероятностей

Точенные оценки неизвестных параметров распределения

Пусть изучается Решение задач по теории вероятностей с законом распределения, зависящим от одного или нескольких параметров. Требуется по выборке, полученной в результате Решение задач по теории вероятностей испытаний оценить неизвестный параметр Решение задач по теории вероятностей.

Точечной оценкой неизвестного параметра Решение задач по теории вероятностей теоретического распределения называется его приближенное значение, зависящее от данных выборки

Решение задач по теории вероятностей

Точечная оценка должна удовлетворять следующим требованиям:

  • оценка должна быть несмещенной, т.е.
Решение задач по теории вероятностей

оценка должна быть состоятельной, т.е. она должна сходиться по вероятности к оцениваемому параметру: для

Решение задач по теории вероятностей
  • оценка должна быть эффективной: если неизвестный параметр имеет несколько оценок, то в качестве оценки нужно брать оценку с наименьшей дисперсией.

Выборочная средняя Решение задач по теории вероятностей является несмещенной и состоятельной оценкой для математического ожидания генеральной совокупности.

Несмещенной и состоятельной оценкой для дисперсии генеральной совокупности является исправленная выборочная дисперсия

Решение задач по теории вероятностей

Исправленным средним квадратическим отклонением называется корень квадратный из исправленной дисперсии

Решение задач по теории вероятностей

Для вычисления Решение задач по теории вероятностей и Решение задач по теории вероятностей разработано много методов. Одним из наиболее распространенных методов является метод произведений. При вычислении выборочного среднего и выборочной дисперсии поступают следующим образом: выбираем «ложный нуль» Решение задач по теории вероятностей. В качестве «ложного нуля» берется варианта стоящая посредине вариационного ряда или варианта, имеющая максимальную частоту;

  • вычисляем условные моменты 1 -ого и 2-ого порядков
Решение задач по теории вероятностей
Решение задач по теории вероятностей

Задача №34

Методом произведений вычислить выборочную среднюю и выборочную дисперсию по данным выборки

Решение задач по теории вероятностей

Решение:

В качестве «ложного нуля» возьмем варианту 75, Решение задач по теории вероятностей = 75. Перейдем к условным вариантам по формуле Решение задач по теории вероятностей. Результаты вычислений сведем в таблицу.

Решение задач по теории вероятностей

Результаты вычислений можно проверить равенством

Решение задач по теории вероятностей

Равенство выполняется, следовательно, таблица заполнена верно. Вычислим условные моменты

Решение задач по теории вероятностей

Вычислим выборочную среднюю и выборочную дисперсию

Решение задач по теории вероятностей

Интервальные оценки

Пусть Решение задач по теории вероятностей — функция выборки. Это есть случайная величина, называемая статистикой.

Интервальной называют оценку, которая определяется случайным интервалом

Решение задач по теории вероятностей

В качестве интервальной оценки используются доверительные интервалы.

Доверительным интервалом для неизвестного параметра Решение задач по теории вероятностей, называется случайный интервал Решение задач по теории вероятностей, который с заданной вероятностью Решение задач по теории вероятностей (надежностью) накрывает неизвестный параметр, Решение задач по теории вероятностей.

Если исследуемая Решение задач по теории вероятностей распределена по нормальному закону с известным средним квадратическим отклонением Решение задач по теории вероятностей, то доверительный интервал для математического ожидания определяется неравенством

Решение задач по теории вероятностей

где Решение задач по теории вероятностей — точность оценки, Решение задач по теории вероятностей — объем выборки, Решение задач по теории вероятностей — значение аргумента функции Лапласа, при котором

Решение задач по теории вероятностей

Если среднее квадратическое отклонение неизвестно, то доверительный интервал для математического ожидания исследуемой Решение задач по теории вероятностей определяется неравенством

Решение задач по теории вероятностей

Значения Решение задач по теории вероятностей находят по таблице приложения 5 по заданным Решение задач по теории вероятностей и Решение задач по теории вероятностей. Число

Решение задач по теории вероятностей

называют точностью оценки математического ожидания.

Доверительный интервал для среднего квадратического отклонения исследуемой Решение задач по теории вероятностей определяется неравенством

Решение задач по теории вероятностей

Значения Решение задач по теории вероятностей и Решение задач по теории вероятностей находятся по таблице приложения 6 по заданным Решение задач по теории вероятностей и Решение задач по теории вероятностей.

Задача №35

Найти доверительный интервал для оценки с надежностью Решение задач по теории вероятностей неизвестного математического ожидания нормально распределенного признака Решение задач по теории вероятностей, если известно Решение задач по теории вероятностей, а по данным выборки объемом 100 вычислено Решение задач по теории вероятностей.

Решение:

Так как известно среднее квадратическое отклонение Решение задач по теории вероятностей то для определения доверительного интервала для математического ожидания воспользуемся неравенством (3.1). Определим значение

Решение задач по теории вероятностей

Подставим в неравенство (3.1)

Решение задач по теории вероятностей

Задача №36

Для исследования нормально распределенной Решение задач по теории вероятностей извлечена выборка объемом 25.

Решение задач по теории вероятностей

Найти с надежностью Решение задач по теории вероятностей доверительные интервалы для математического ожидания и среднего кадратического отклонения исследуемой Решение задач по теории вероятностей.

Решение:

По данным выборки методом произведений определим Решение задач по теории вероятностей и Решение задач по теории вероятностей

Решение задач по теории вероятностей

Проверка:

Решение задач по теории вероятностей

Решение задач по теории вероятностей

Для определения доверительного интервала для математического ожидания воспользуемся неравенством (3.2);

Решение задач по теории вероятностей

Для определения доверительного интервала для среднего квадратического отклонения воспользуемся неравенством (3.3):

Решение задач по теории вероятностей

Статистическая проверка гипотез. Критерии согласия Пирсона и Колмогорова

Статистической называется гипотеза о предполагаемом виде неизвестного распределения Решение задач по теории вероятностей или о значениях параметров известного вида распределения. Пулевой гипотезой Решение задач по теории вероятностей называется выдвинутая гипотеза. Конкурирующей (альтернативной) называется гипотеза, которая противоречит нулевой гипотезе. При проверке статистической гипотезы могут быть допущены ошибки двух родов. Ошибка первого рода — будет отклонена верная гипотеза. Ошибка второго рода — будет принята неверная гипотеза.

Вероятность допустить ошибку первого рода называется уровнем значимости. Для проверки статистической гипотезы используют специальную статистику, которая называется критерием.

По рассчитанному значению критерия определяют принимать или отвергать нулевую гипотезу.

Критерий согласия — это проверка гипотезы о виде распределения Решение задач по теории вероятностей.

Основными критериями согласия являются критерии Пирсона Решение задач по теории вероятностей и Колмохорова. При проверке гипотезы с помощью критерия Пирсона поступают следующим образом:

из генеральной совокупности извлекают выборку объемом Решение задач по теории вероятностей; по выборке вычисляют Решение задач по теории вероятностей и Решение задач по теории вероятностей:

переходят к нормированной Решение задач по теории вероятностей по формуле

Решение задач по теории вероятностей

находят вероятности попадания в интервал

Решение задач по теории вероятностей

вычисляют теоретические частоты

Решение задач по теории вероятностей

вычисляют статистику Пирсона

Решение задач по теории вероятностей

из таблицы критических точек распределения Пирсона (приложение 3) по уровню значимости Решение задач по теории вероятностей и числу степеней свободы

Решение задач по теории вероятностей

определяют Решение задач по теории вероятностей, где Решение задач по теории вероятностей — число интервалов в вариационном ряде, Решение задач по теории вероятностей — количество параметров закона распределения, которые оцениваются по выборке (для нормального закона Решение задач по теории вероятностей=2);

• если Решение задач по теории вероятностей то нет необходимости отвергать нулевую гипотезу, т.е. эмпирические и теоретические частоты согласуются;

• если Решение задач по теории вероятностей то гипотеза отвергается, т.е. расхождение между теоретическими и эмпирическими частотами существенно;

• если исследуется дискретная Решение задач по теории вероятностей, распределенная по нормальному закону, то теоретические вероятности определяются по формуле

Решение задач по теории вероятностей

где Решение задач по теории вероятностей — шаг,

Решение задач по теории вероятностей

Задача №37

Пользуясь критерием Пирсона, при уровне значимости Решение задач по теории вероятностей проверить, согласуется ли гипотеза о нормальном распределении генеральной совокупности с данными выборки

Решение задач по теории вероятностей

Решение:

По данным выборки методом произведений вычислим Решение задач по теории вероятностей и Решение задач по теории вероятностей.

Решение задач по теории вероятностей

Проверка:

Решение задач по теории вероятностей

Вычислим вероятности попадания в интервалы

Решение задач по теории вероятностей

Вычислим Решение задач по теории вероятностей

Решение задач по теории вероятностей

Определим число степеней свободы

Решение задач по теории вероятностей

По уровню значимости Решение задач по теории вероятностей и числу степеней свободы Решение задач по теории вероятностей найдем критическую точку правосторонней критической области распределения Пирсона (приложение 3)

Решение задач по теории вероятностей

Так как Решение задач по теории вероятностей, гипотеза о нормальном распределении совокупности отвергается.

Критерий согласия Колмогорова применяется для проверки гипотезы о законе распределения непрерывной Решение задач по теории вероятностей. Для статистической проверки гипотезы с помощью критерия согласия Колмогорова поступают следующим образом:

Решение задач по теории вероятностей

вычисляют статистику Колмогорова

Решение задач по теории вероятностей

где Решение задач по теории вероятностей объем выборки. Решение задач по теории вероятностей имеет функцию распределения

Решение задач по теории вероятностей

которая называется функцией Колмогорова;

находим по уровню значимости Решение задач по теории вероятностей (приложение 7);

Рассмотрим применение критерия Колмогорова на примере.

Задача №38

Проверить по критерию Колмогорова гипотезу о нормальном распределении Решение задач по теории вероятностей но данным выборки при уровне значимости Решение задач по теории вероятностей.

Решение задач по теории вероятностей

Решение:

Вычислим выборочную среднюю Решение задач по теории вероятностей и исправленное среднее квадратическое отклонение Решение задач по теории вероятностей.

Решение задач по теории вероятностей

Тогда теоретическая функция распределения в предположении, что Решение задач по теории вероятностей распределена по нормальному закону, имеет вид

Решение задач по теории вероятностей

где Решение задач по теории вероятностей — функция Лапласа.

Эмпирическую функцию распределения определим по формуле

Решение задач по теории вероятностей

где Решение задач по теории вероятностей сумма частот вариант меньших Решение задач по теории вероятностей.

Решение задач по теории вероятностей

Вычислим величину

Решение задач по теории вероятностей

Решение задач по теории вероятностей

Вычислим статистику Колмогорова

Решение задач по теории вероятностей

По уровню значимости Решение задач по теории вероятностей найдем по таблице (приложение 7) Решение задач по теории вероятностей. Т.к. Решение задач по теории вероятностей, то нет оснований отвергать гипотезу о нормальном распределении.

Выборочный коэффициент корреляции и его свойства

Проверка гипотезы о равенстве нулю коэффициента корреляции Краткие теоретические сведения

Для вычисления выборочного коэффициента корреляции данные представляются в виде корреляционной таблицы. Корреляционная таблица представляет собой таблицу следующего вида: в первой строке записаны наблюдаемые значения Решение задач по теории вероятностей, в первом столбце записаны наблюдаемые значения Решение задач по теории вероятностей, на пересечении Решение задач по теории вероятностей-той строки и Решение задач по теории вероятностей-го столбца записывается частота Решение задач по теории вероятностей появления пары Решение задач по теории вероятностей. В последнем столбце записывается частота появления варианты Решение задач по теории вероятностей, в последней строке — частота появления варианты Решение задач по теории вероятностей на пересечении последней строки и последнего столбца записывается суммарное количество наблюдений. Корреляционная таблица имеет вид

Решение задач по теории вероятностей

Основной оценкой тесноты связи между случайными величинами Решение задач по теории вероятностей и Решение задач по теории вероятностей служит выборочный коэффициент корреляции Решение задач по теории вероятностей который определяется так

Решение задач по теории вероятностей

где Решение задач по теории вероятностей — среднее арифметическое произведений значений Решение задач по теории вероятностей.

Свойства выборочного коэффициента корреляции аналогичны свойствам коэффициента корреляции между Решение задач по теории вероятностей:

  1. Решение задач по теории вероятностей;
  2. если переменные Решение задач по теории вероятностей и Решение задач по теории вероятностей умножить на одно и то же число, то коэффициент корреляции не изменится;
  3. если Решение задач по теории вероятностей, то корреляционная связь между значениями Решение задач по теории вероятностей и Решение задач по теории вероятностей представляет собой линейную функциональную зависимость.

Для вычисления выборочного коэффициента корреляции применяется формула

Решение задач по теории вероятностей

Если Решение задач по теории вероятностей, то между наблюдаемыми значениями Решение задач по теории вероятностей и Решение задач по теории вероятностей корреляционная зависимость отсутствует, чем ближе к единице приближается модуль коэффициента корреляции, тем теснее связь между переменными Решение задач по теории вероятностей и Решение задач по теории вероятностей. Т.к. выборочный коэффициент корреляции вычисляется по данным выборки, то в отличие от коэффициента корреляции генеральной совокупности Решение задач по теории вероятностей является случайной величиной. Если Решение задач по теории вероятностей то возникает вопрос, объясняется ли это действительно существующей связью между Решение задач по теории вероятностей и Решение задач по теории вероятностей или вызвано случайными факторами. Для выяснения этого вопроса проверяется гипотеза Решение задач по теории вероятностей о равенстве нулю коэффициента корреляции Решение задач по теории вероятностей генеральной совокупности.

Для того, чтобы при уровне значимости Решение задач по теории вероятностей проверить нулевую гипотезу о равенстве нулю коэффициента корреляции генеральной двумерной нормальной совокупности, вычисляют статистику

Решение задач по теории вероятностей

и по таблице критических точек распределения Стьюдента (приложение 4) по уровню значимости а и числу степеней свободы Решение задач по теории вероятностей находят

Решение задач по теории вероятностей

критическую точку двусторонней критической области. Если

Решение задач по теории вероятностей

нет оснований отвергать нулевую гипотезу, т.е. Решение задач по теории вероятностей; если

Решение задач по теории вероятностей

нулевую гипотезу отвергают, т.е. Решение задач по теории вероятностей. Рассмотрим вычисление выборочною коэффициента корреляции и проверку гипотезы о равенстве нулю коэффициента корреляции генеральной совокупности на примере.

Задача №39

По данной корреляционной таблице вычислить выборочный коэффициент корреляции и при уровне значимости Решение задач по теории вероятностей проверить гипотезу о равенстве нулю коэффициента корреляции генеральной совокупности.

Решение задач по теории вероятностей

Решение:

Вычислим компоненты, входящие в формулу (5.1), для вычисления Решение задач по теории вероятностей

Решение задач по теории вероятностей

Вычислим выборочный коэффициент корреляции

Решение задач по теории вероятностей

Проверим гипотезу о равенстве нулю коэффициента корреляции генеральной совокупности. Вычислим

Решение задач по теории вероятностей

По таблице критических точек распределения Стыодента (приложение 4) по уровню значимости Решение задач по теории вероятностей и числу степеней свободы Решение задач по теории вероятностей найдем

Решение задач по теории вероятностей

Так

Решение задач по теории вероятностей

то гипотеза о равенстве нулю коэффициента корреляции генеральной совокупности отвергается, т.е. выбранный коэффициент корреляции значим.

Кстати готовые задачи на продажу по предмету теория вероятности тут.

Линейная регрессия. Определение параметров линейной регрессии

Если обе линии регрессии Решение задач по теории вероятностей на Решение задач по теории вероятностей и Решение задач по теории вероятностей на Решение задач по теории вероятностей являются прямыми, то в этом случае корреляцию называют линейной. Выборочное уравнение прямой линии регрессии Решение задач по теории вероятностей на Решение задач по теории вероятностей имеет вид

Решение задач по теории вероятностей

Уравнение прямой регрессии Решение задач по теории вероятностей на Решение задач по теории вероятностей имеет вид

Решение задач по теории вероятностей

Здесь Решение задач по теории вероятностей — значения Решение задач по теории вероятностей — их выборочные средние.

Коэффициент уравнений (6.1)-(6.2) можно также определить по формулам, полученным методом наименьших квадратов. Например, если уравнение (6.1) взять в виде Решение задач по теории вероятностей, то параметры Решение задач по теории вероятностей и Решение задач по теории вероятностей линейной регрессии имеют вид:

Решение задач по теории вероятностей

Задача №41

Распределение 40 заводов отрасли по количеству слесарей Решение задач по теории вероятностей и числу станкосмен Решение задач по теории вероятностей задано корреляционной таблицей.

Решение задач по теории вероятностей

Составить уравнение прямой регрессии Решение задач по теории вероятностей на Решение задач по теории вероятностей.

Решение:

По корреляционной таблице вычислим

Решение задач по теории вероятностей

Подставим вычисленные значения в уравнение (6.1)

Решение задач по теории вероятностей

Задача №42

При эталонировании медного термометра изучалась зависимость электрического сопротивления Решение задач по теории вероятностей от температуры Решение задач по теории вероятностей. Были получены следующие результаты

Решение задач по теории вероятностей

Оценить параметры уравнения регрессии с помощью метода наименьших квадратов и записать уравнение регрессии Решение задач по теории вероятностей на Решение задач по теории вероятностей.

Решение:

Сведем результаты вычисления в таблицу.

Решение задач по теории вероятностей

Параметры линейной регрессии определим по формулам (6.3)

Решение задач по теории вероятностей

Эмпирическое уравнение регрессии Решение задач по теории вероятностей на Решение задач по теории вероятностей примет вид

Решение задач по теории вероятностей

Решение задач по теории вероятностей

Решение задач по теории вероятностей

Решение задач по теории вероятностей

Решение задач по теории вероятностей

Решение задач по теории вероятностей

Решение задач по теории вероятностей

Решение задач по теории вероятностей

Решение задач по теории вероятностей

Возможно эти страницы вам будут полезны:

  • Помощь по теории вероятности
  • Заказать работу по теории вероятности
  • Контрольная работа по теории вероятности
  • Курсовая работа по теории вероятности
  • Решение задач по математической статистике
  • Помощь по математической статистике
  • Заказать работу по математической статистике
  • Контрольная работа по математической статистике
  • Курсовая работа по математической статистике
  • Теория вероятностей краткий курс для школьников и студентов (заочников) 

Примеры решения задач по всем темам теории вероятностей

В различных разделах науки и техники нередко возникают ситуации, когда результат каждого из многих проводимых опытов заранее предугадать невозможно, однако можно исследовать закономерности, возникающие при проведении серии опытов. Нельзя, например, точно сказать, какая сторона монеты окажется сверху при данном броске: герб или цифра – но при большом количестве бросков число выпадений герба приближается к половине количества бросков; нельзя заранее предсказать результат одного выстрела из данного орудия по данной цели, но при большом числе выстрелов частота попадания приближается к некоторому постоянному числу. Исследование вероятностных закономерностей массовых однородных явлений составляет предмет теории вероятностей.

Основным интуитивным понятием классической теории вероятностей является случайное событие.

События, которые могут произойти в результате опыта, можно подразделить на три вида:

  • а) достоверное событие – событие, которое всегда происходит при проведении опыта;
  • б) невозможное событие – событие, которое в результате опыта произойти не может;
  • в) случайное событие – событие, которое может либо произойти, либо не произойти.

Теория вероятностей изучает закономерности, возникающие в случайных экспериментах, раскрывает объективные закономерности, присущие массовым явлениям.

Развитие как науки теории вероятностей берет свое начало с переписки Паскаля и Ферма (1654 г.). Но и до этого многих ученых интересовали задачи, относящиеся к азартным играм, теоретико-вероятностные задачи, имеющие прикладное значение (Кардано, Галилей).

Кроме задач азартных игр появлялся интерес к построению таблиц смертности и вопросам страхования (Граунт, Ван Худде, Ван де Витт).

Факты устойчивости частот случайных событий в задачах обработки демографических данных были известны еще в Древнем Китае и Древнем Риме.

С течением времени объект изучения теории вероятностей менялся. Если вначале основной интерес вызывало исследование вероятностей случайных событий, то уже в XIX в. интерес вызывало исследование случайных величин.

Теория вероятностей тесно связана с прикладными исследованиями различной природы. Она применима как в задачах экономики, производства, так и задачах лингвистики и истории. Сейчас без применения понятия доверительного интервала, корреляции, уровня значимости, нормального закона распределения случайной величины сложно представить обширное исследование в педагогике, физике, механике и других науках.

В основе квантовой механики лежат принципы теории вероятностей. В случае радиоактивного распада нет закона природы, позволяющего определить точное время деления ядра. Существуют только законы, согласно которым можно говорить о вероятности рассада ядра за определенный промежуток времени.

Элементарная теория вероятностей

Во многих областях человеческой деятельности существуют ситуации, когда определенные явления могут повторяться неограниченное число раз в одинаковых условиях. Подбрасывание монеты, кости, выброс из колоды карт и т.д.

Заметим, что представляется возможным предсказать исход последующего эксперимента по результатам предыдущих, как бы ни было велико число проведенных испытаний.

Во-вторых, относительная частота определенных исходов по мере роста числа испытаний стабилизируется, приближаясь к определенному числу.

Рассмотрим эксперимент по подбрасыванию монеты. Его результат представлен в таблице 1.

Примеры решения задач по теории вероятности

Примеры решения задач по теории вероятности — номер испытания, Примеры решения задач по теории вероятности — количество подбрасываний, в таблице указывается количество выпадений герба.

Наблюдалась стабилизация частот

Примеры решения задач по теории вероятности

Обнаруженные закономерности, распространенные на испытания с произвольным числом исходов, позволяют построить простейшую математическую модель случайного эксперимента.

Под опытом, или экспериментом, или испытанием понимают осуществление конкретного комплекса условий. Опыт называется случайным, если его результат нельзя точно предсказать до его осуществления.

Например, если опыт заключается в подбрасывании монеты, то результат его -выпадение герба (Г) или решетки (Р) — нельзя предсказать заранее. Точно также при стрельбе по мишени нельзя заранее предсказать, будет ли точное попадание в цель или промах.

Построение математической модели эксперимента начинается с описания множества Примеры решения задач по теории вероятности всевозможных исходов, которые могут произойти в результате каждого испытания.

Пространство Примеры решения задач по теории вероятности называют пространством элементарных исходов, элемент этого пространства Примеры решения задач по теории вероятности — элементарный исход (элементарное событие).

Событием является любое подмножество Примеры решения задач по теории вероятности.

Событие называется достоверным, если оно обязательно произойдет в условиях данного опыта. Например, выбор одной годной детали из партии Примеры решения задач по теории вероятности годных деталей есть событие достоверное. Так как достоверное событие является совокупностью всех элементарных событий из Примеры решения задач по теории вероятности, то оно совпадает с пространством Примеры решения задач по теории вероятности и также обозначается Примеры решения задач по теории вероятности.

Невозможным называется событие, которое в условиях данного опыта не может произойти. Невозможное событие в пространстве не имеет точек в Примеры решения задач по теории вероятности и обозначается Примеры решения задач по теории вероятности. Например, невозможно поразить одну и ту же мишень три раза при двух выстрелах.

Если ограничиться рассмотрением пространства элементарных исходов, состоящих из не более, чем счетного числа элементов, то построение вероятностной модели по существу состоит в задании распределения вероятностей на пространстве Примеры решения задач по теории вероятности в соответствие с которым каждому элементарному исходу Примеры решения задач по теории вероятности ставится в соответствие число Примеры решения задач по теории вероятности, называемое вероятностью элементарного события Примеры решения задач по теории вероятности.

Примеры решения задач по теории вероятности

Различают элементарные и составные события. События, которые невозможно разложить на более простые, называются элементарными. Все остальные события называются составными. Например, пусть событие состоит в том, что сумма очков, выпавших при бросании двух игральных костей, равна шести. Это событие состоит из пяти возможных элементарных событий — выпадение на гранях костей следующих пар цифр: (1,5), (2,4), (3,3), (4,2), (5,1) соответственно.

Вероятность любого составного события Примеры решения задач по теории вероятности:

Примеры решения задач по теории вероятности

Число Примеры решения задач по теории вероятности интерпретируется как относительная частота появления события Примеры решения задач по теории вероятности в статистическом эксперименте.

События называются несовместными, если появление одного из них исключает появление других событий в условиях одного и того же опыта.

Два или несколько событий называются равновозможными, если нет оснований утверждать, что одно из них имеет больше данных появиться в итоге опыта по сравнению с другими. Например, извлечение туза, валета, короля или дамы из колоды карт.

Событие Примеры решения задач по теории вероятности, которое обязательно произойдет, если не произойдет событие Примеры решения задач по теории вероятности, называется противоположным событию Примеры решения задач по теории вероятности. Например, выигрыш и проигрыш в лотерее — противоположные события.

Если в задаче дана вероятность Примеры решения задач по теории вероятности, тогда чтобы найти вероятность противоположного события, необходимо воспользоваться следующей формулой:

Примеры решения задач по теории вероятности

где Примеры решения задач по теории вероятности — вероятность противоположного события.

Говорят, что несколько событий в условиях данного опыта образуют полную группу событий, если в результате опыта обязательно произойдет хотя бы одно из них. Например, события «извлечение белого шара», «извлечение красного шара», «извлечение голубого шара» образуют полную группу событий в опыте извлечения шара из урны, в которой находятся белые, красные и голубые шары.

Пример №1

  • Подбрасывается монета и регистрируется сторона монеты, которая обращена к наблюдателю после падения. Найти пространство элементарных исходов.

Решение:

Пусть событие Г = {выпал герб}, Р = {выпала решка}.

Тогда Примеры решения задач по теории вероятности.

Пример №2

  • Бросается игральная кость и регистрируется число выпавших очков. Найти пространство элементарных исходов. Найти событие, состоящее в выпадении четного числа очков.

Решение:

Примеры решения задач по теории вероятности

Пример №3

  • Бросаются две игральные кости. Описать событие, состоящее в том, что сумма очков больше 10.

Решение:

Примеры решения задач по теории вероятности

Вероятностное пространство

Пусть Примеры решения задач по теории вероятности — множество элементарных исходов.

Подмножество пространства Примеры решения задач по теории вероятности называется событием Примеры решения задач по теории вероятности, если статистический эксперимент закончился элементарным исходом Примеры решения задач по теории вероятности.

Рассмотрим теоретико-множественные операции в данном пространстве, которые представлены в следующей таблице.

Примеры решения задач по теории вероятности

Примеры решения задач по теории вероятности

Пусть Примеры решения задач по теории вероятности и Примеры решения задач по теории вероятности — обозначают события выпадения при бросании игральной кости соответственно нечетного числа очков и числа очков, кратного трем. Тогда

Примеры решения задач по теории вероятности

и,значит,

Примеры решения задач по теории вероятности

Булева алгебра и понятие вероятности

Булевой алгеброй называют такой класс Примеры решения задач по теории вероятности подмножеств Примеры решения задач по теории вероятности, что:

Примеры решения задач по теории вероятности

Вероятностью Примеры решения задач по теории вероятности на булевой алгебре Примеры решения задач по теории вероятности подмножеств Примеры решения задач по теории вероятности называется отображение Примеры решения задач по теории вероятности в отрезок [0, 1], обладающее следующими свойствами:

1) Примеры решения задач по теории вероятности.

2) Если события Примеры решения задач по теории вероятности несовместны, то Примеры решения задач по теории вероятности.

3) Если Примеры решения задач по теории вероятности — монотонно убывающая последовательность элементов из Примеры решения задач по теории вероятности и Примеры решения задач по теории вероятности, то Примеры решения задач по теории вероятности. Это может быть записано, как Примеры решения задач по теории вероятности.

Замечание. Вероятность Примеры решения задач по теории вероятности на Примеры решения задач по теории вероятности обладает свойствами:

Примеры решения задач по теории вероятности

Пара Примеры решения задач по теории вероятности, состоящая из пространства элементарных исходов Примеры решения задач по теории вероятности и булевой Примеры решения задач по теории вероятности-алгебры Примеры решения задач по теории вероятности его подмножеств, называется измеримым пространством. Только элементы Примеры решения задач по теории вероятности называются событиями.

Тройка Примеры решения задач по теории вероятности, где Примеры решения задач по теории вероятности — вероятность на Примеры решения задач по теории вероятности — алгебре Примеры решения задач по теории вероятности, называется вероятностным пространством.

Элементы комбинаторики

Комбинаторика — раздел математики, изучающий комбинации конечных множеств элементов различной природы.

Пусть все элементы рассматриваемых множеств различны. Будем изучать комбинации этих элементов, различающихся количеством и/или порядком.

Дано конечное число Примеры решения задач по теории вероятности объектов произвольной природы, которые назовем элементами.

Из них по определенному правилу можно образовать некоторые группы. Подсчетом числа таких возможных групп и занимается комбинаторика.

Будем рассматривать такие множества, в которых каждый элемент входит не более одного раза (соединения без повторений).

Перестановкой из Примеры решения задач по теории вероятности элементов называется конечное множество элементов, в котором установлен порядок. Так, например, из букв Примеры решения задач по теории вероятности можно составить следующие перестановки:

Примеры решения задач по теории вероятности

Число возможных перестановок из Примеры решения задач по теории вероятности элементов равно:

Примеры решения задач по теории вероятности

Множество, для которого указан порядок расположения элементов, называется упорядоченным. Упорядоченные конечные подмножества некоторого множества называются размещениями.

Число всех возможных размещений, содержащих по Примеры решения задач по теории вероятности элементов из множества, содержащего Примеры решения задач по теории вероятности элементов Примеры решения задач по теории вероятности, определяется по формуле:

Примеры решения задач по теории вероятности

Всякое конечное подмножество, состоящее из Примеры решения задач по теории вероятности элементов данного множества из Примеры решения задач по теории вероятности элементов, называется сочетанием Примеры решения задач по теории вероятности элементов из Примеры решения задач по теории вероятности, если каждое подмножества из Примеры решения задач по теории вероятности элементов отличается одно от другого хотя бы одним элементом.

Число всех возможных сочетаний обозначается:

Примеры решения задач по теории вероятности

Пример №4

  • В группе 10 юношей и 7 девушек. Из группы случайным образом отбирается 5 студентов. Найти вероятность того, что среди них окажется 4 девушки?

Решение:

Пусть событие Примеры решения задач по теории вероятности состоит в том, что из 5 случайно отобранных студентов окажутся 4 девушки. Общее число исходов будет равно количеству способов, сколькими из 17 студентов можно отобрать по 5 студентов Примеры решения задач по теории вероятности. Благоприятствовать событию Примеры решения задач по теории вероятности будут те исходы, в которых будет 4 девушки и 1 юноша

Примеры решения задач по теории вероятности

Тогда

Примеры решения задач по теории вероятности

Пример №5

  • Сколько способов существует для выбора команды участников субботника, если известно, что в команде должно быть 5 человек, а в студенческой группе 25 человек?

Решение:

Поскольку порядок следования элементов в подгруппе не имеет значения, значит речь идет о количестве сочетаний

Примеры решения задач по теории вероятности

Гипергеометрическое распределение

Большой класс задач, которые интерпретируются в рамках урновой схемы. Типовая задача: Пусть в эксперименте рассматриваются: Примеры решения задач по теории вероятности — черных шаров, Примеры решения задач по теории вероятности — белых шаров.

Отбирается Примеры решения задач по теории вероятности шаров из урны. Какова вероятность, что выборка содержит Примеры решения задач по теории вероятности черных шаров?

Нахождение вероятности в рамках данной схемы осуществляется по следующей формуле:

Примеры решения задач по теории вероятности

Пример №6

  • Автомат с 30 мягкими игрушками, содержит фигурки зверей и супергероев в пропорции 2:1 соответственно. В случае победы автомат выдает случайным образом две игрушки. Какова вероятность, что это окажутся супергерои?

Решение:

Поскольку в эксперименте есть два ярко выделенных признака, по которым объект можно отнести либо к первому типу (мягкая игрушка), либо ко второму типу (супергерой), речь идет о гипергеометрическом распределении. Примеры решения задач по теории вероятностиПримеры решения задач по теории вероятности (количество супергероев), Примеры решения задач по теории вероятности (количество зверей). Тогда общее количество Примеры решения задач по теории вероятности, выбирают Примеры решения задач по теории вероятности игрушек, Примеры решения задач по теории вероятности (среди тех, которые выбрали, оба оказались супергероями). Тогда по формуле гипергеометрического распределения:

Примеры решения задач по теории вероятности

Пример №7

  • На складе обоев 10 трубок первой партии и 7 трубок второй партии. Продавец случайным образом выбирает 3 трубки, какова вероятность, что все трубки окажутся одной партии?

Решение:

По вопросу задачи можно сделать вывод, что исходами, благоприятствующими наступлению события Примеры решения задач по теории вероятности = { все три трубки окажутся одной партии}, являются следующие: {три трубки первой партии}, {три трубки второй партии}. Тогда вероятность может быть найдена по следующей формуле:

Примеры решения задач по теории вероятности

Примеры вероятностных пространств

Рассмотрим в таблице примеры вероятностных пространств.

Примеры решения задач по теории вероятности

Примеры решения задач по теории вероятности

Разбиение на группы: перестановки, сочетания и размещения с повторениями

Пусть Примеры решения задач по теории вероятности — целые неотрицательные числа, причем Примеры решения задач по теории вероятности. Число способов, которыми можно представить множество Примеры решения задач по теории вероятности из Примеры решения задач по теории вероятности элементов в виде суммы Примеры решения задач по теории вероятности множеств Примеры решения задач по теории вероятности, число элементов которых составляет соответственно Примеры решения задач по теории вероятности равно:

Примеры решения задач по теории вероятности

Сочетаниями из Примеры решения задач по теории вероятности элементов по Примеры решения задач по теории вероятности элементов с повторениями называются группы, содержащие Примеры решения задач по теории вероятности элементов, причем каждый элемент принадлежит одному из Примеры решения задач по теории вероятности типов.

Число различных сочетаний из Примеры решения задач по теории вероятности типов по Примеры решения задач по теории вероятности объектов с повторениями равно:

Примеры решения задач по теории вероятности

Отображение множества Примеры решения задач по теории вероятности первых натуральных чисел 1, 2, 3, …, Примеры решения задач по теории вероятности в данное множество Примеры решения задач по теории вероятности называется размещением с повторением, составленным из данных Примеры решения задач по теории вероятности элементов (количество типов) по Примеры решения задач по теории вероятности. Количество размещений с повторениями находится по следующей формуле:

Примеры решения задач по теории вероятности

Пример №8

  • Найдем число различных слов, которые можно получить, переставляя буквы в слове «Математика».

Решение:

Примеры решения задач по теории вероятности

Пример №9

  • Найти число способов, которыми можно выбрать три буквы из АААТТТГГГЦЦЦ.

Решение:

Примеры решения задач по теории вероятности

Пример №10

  • Найти количество всевозможных размещений с повторениями из букв Примеры решения задач по теории вероятности по две буквы.

Решение:

Примеры решения задач по теории вероятности

Независимость. Условные вероятности

Зная распределения вероятностей, мы в состоянии оптимизировать свое поведение при игре, производя ставки на те события из Примеры решения задач по теории вероятности, которые обладают наибольшей вероятностью.

Дальнейшая оптимизация такой игры обычно осуществляется за счет дополнительной информации, которой может располагать игрок, и учет такой информации осуществляется в терминах так называемой условной вероятности.

Рассмотрим два случайных события Примеры решения задач по теории вероятности и Примеры решения задач по теории вероятности. Пусть известно, что событие Примеры решения задач по теории вероятности наступило, но неизвестно, какое конкретно из элементарных событий Примеры решения задач по теории вероятности, составляющих событие Примеры решения задач по теории вероятности, наступило. Что можно сказать в этом случае о вероятности наступления события Примеры решения задач по теории вероятности?

Пусть вероятность события Примеры решения задач по теории вероятности — положительная величина. Условной вероятностью события Примеры решения задач по теории вероятности при условии, что произошло событие Примеры решения задач по теории вероятности, называют число:

Примеры решения задач по теории вероятности

Теорема умножения. Пусть

Примеры решения задач по теории вероятности

Тогда

Примеры решения задач по теории вероятности

Теорема.

Примеры решения задач по теории вероятности

Тогда

Примеры решения задач по теории вероятности

Задача. Студент знает 20 вопросов из 30. Экзаменатор задает три вопроса. Какова вероятность того, что студент ответит на все вопросы?

Два события называются независимыми, если вероятность появления одного из них не влияет на вероятность наступления другого. Говорят, что событие Примеры решения задач по теории вероятности не зависит от события Примеры решения задач по теории вероятности, если Примеры решения задач по теории вероятности, т.к. его вероятность не зависит от того, произошло ли событие В или нет. Независимость двух событий — свойство симметричное.

События Примеры решения задач по теории вероятности и Примеры решения задач по теории вероятности называются независимыми, если

Примеры решения задач по теории вероятности

Случайные события Примеры решения задач по теории вероятности называются попарно независимыми, если для любых

Примеры решения задач по теории вероятности

Случайные события Примеры решения задач по теории вероятности называются независимыми в совокупности, если для любого подмножества индексов:

Примеры решения задач по теории вероятности

Задача (Пример Бернштейна). На плоскость бросают тетраэдр, три грани которого окрашены соответственно в красный, зеленый и синий цвета, а на четвертой грани есть все цвета. Рассмотреть вероятности событий «выпала грань, которая содержит красный цвет», «выпала грань, которая содержит синий цвет», «выпала грань, которая содержит зеленый цвет». Будут ли эти события попарно независимыми и независимыми в совокупности?

Пример №11

  • В тире девушке и юноше выдали по одному патрону для попадания в цель и получения плюшевого медведя. Вероятность того, что попадет в цель девушка, равна 0,01. Вероятность того, что попадет юноша, равна 0,95. Каждый сделал по одному выстрелу. Какова вероятность, что мишка будет выигран?

Решение:

Исходы, благоприятствующие наступлению этого события:

{юноша попал и девушка попала},{юноша не попал и девушка попала},{юноша попал и девушка не попала}.

Примеры решения задач по теории вероятности

Пример №12

  • В вазе стоит 5 роз и 4 гвоздики. Случайным образом выбирается один цветок. После этого выбирается еще один. Какова вероятность того, что второй цветок — роза?

Решение:

Первым выбранным цветком могла оказаться роза, тогда после ее изъятия в вазе останется только 4 розы. Первой могла оказаться гвоздика, тогда после первого изъятия цветка останется 5 роз. Вероятность того, что второй выбранный цветок роза, вычисляется следующим образом:

Примеры решения задач по теории вероятности

Формула полной вероятности. Формулы Байеса

Конечное или счетное число случайных событий Примеры решения задач по теории вероятности,… образует полную группу событий (разбиение) если:

Примеры решения задач по теории вероятности

Теорема (Формула полной вероятности). Пусть случайные события Примеры решения задач по теории вероятности образует полную группу событий. Тогда для произвольного события В, рассматриваемого на том же вероятностном пространстве выполняется следующее:

Примеры решения задач по теории вероятности

Пусть до опыта об исследуемом случайном явлении имеются гипотезы Примеры решения задач по теории вероятности. После опыта становится известной информация о результатах этого

явления, но не полная. Результаты наблюдений показывают, не какой конкретно элементарный исход Примеры решения задач по теории вероятности произошел, а что наступило некоторое событие Примеры решения задач по теории вероятности. Считая, что до опыта были известны (априорные) вероятности Примеры решения задач по теории вероятности и условные вероятности Примеры решения задач по теории вероятности, необходимо определить апостериорные вероятности Примеры решения задач по теории вероятности. Решение поставленной задачи дают формулы Байеса.

Теорема (Формулы Байеса). Пусть случайные события Примеры решения задач по теории вероятности образуют полную группу событий. Пусть для произвольного события Примеры решения задач по теории вероятности. Тогда для любых значений Примеры решения задач по теории вероятности имеют место формулы:

Примеры решения задач по теории вероятности

Пример №13

  • Студент выучил 20 билетов из 25 и идет отвечать вторым. Какова вероятность, что он вытянет «удачный билет»?

Решение:

Рассмотрим следующие события:

Примеры решения задач по теории вероятности

Тогда

Примеры решения задач по теории вероятности

Пример №14

  • Соотношение грузовых автомобилей, проезжающих по шоссе, на котором стоит бензоколонка, к числу легковых машин, проезжающих по тому же шоссе, равно 2:3. Вероятность того, что будет заправляться грузовая автомашина равна 0,1; для легковой машины эта вероятность равна 0,3. К бензоколонке подъехала для заправки автомашина. Найти вероятность того, что это грузовая автомашина.

Решение:

Пусть событие Примеры решения задач по теории вероятности — к бензоколонке подъехала для заправки автомашина; Примеры решения задач по теории вероятности — подъехала грузовая автомашина; Примеры решения задач по теории вероятности — подъехала легковая автомашина. Тогда

Примеры решения задач по теории вероятности

Пример №15

  • При лечении больному необходимо принять лекарства двух видов одинаковой дозировки. Вероятность того, что больному станет легче от первого лекарства равна 0,9; от второго — 0,97. Больному стало легче. Какова вероятность того, что на его состояние повлияло первое лекарство?

Решение:

Рассмотрим равновероятные гипотезы Примеры решения задач по теории вероятности={больной принимает первое лекарство}, Примеры решения задач по теории вероятности= {больной принимает второе лекарство}.

Примеры решения задач по теории вероятности

Также рассмотрим событие Примеры решения задач по теории вероятности = {больному стало легче}. Условные вероятности:

Примеры решения задач по теории вероятности

Поскольку известно событие, которое наступило, необходимо использовать формулы Байеса. Вероятность того, что на состояние больного повлияло первое лекарство, будет найдена по формуле:

Примеры решения задач по теории вероятности

Пример №16

  • На огороде посажены семена гороха и перца в одинаковых пропорциях. Всхожесть гороха равна 0,06. Всхожесть перца составляет 0,15. Растение проросло, какова вероятность, что это взошел перец?

Решение:

Рассмотрим взаимоисключающие гипотезы Примеры решения задач по теории вероятности={посажено семя гороха}, Примеры решения задач по теории вероятности={посажено семя перца}.

Примеры решения задач по теории вероятности

Также рассмотрим событие Примеры решения задач по теории вероятности = {всхожесть семени}.

Примеры решения задач по теории вероятности
Примеры решения задач по теории вероятности

Поскольку известно событие, которое наступило (растение проросло), необходимо использовать формулы Байеса. Вероятность того, что взошел перец, будет найдена по формуле:

Примеры решения задач по теории вероятности

Схема Бернулли

Под испытанием следует понимать эксперимент со случайным исходом.

Пусть производятся Примеры решения задач по теории вероятности независимых испытаний. Известно, что в каждом испытании возможны два исхода: либо происходит событие Примеры решения задач по теории вероятности (успех), либо событие Примеры решения задач по теории вероятности не происходит (неудача). Данная схема называется схемой Бернулли. При том предполагается, что вероятность Примеры решения задач по теории вероятности успеха и Примеры решения задач по теории вероятности неудачи не изменяются при переходе от испытания к испытанию.

Примеры решения задач по теории вероятности

Задача. Известно, что левши составляют 1% от жителей Земли. Найти вероятность того, что среди 200 человек найдется хотя бы 3 левши.

Наивероятнейшее число появления события Примеры решения задач по теории вероятности в Примеры решения задач по теории вероятности независимых испытаниях Примеры решения задач по теории вероятности — число испытаний, при котором достигается максимальная вероятность в Примеры решения задач по теории вероятности независимых испытаниях:

Примеры решения задач по теории вероятности

Пример №17

  • Прибор состоит из четырех узлов. Вероятность безотказной работы в течение смены для каждого узла равна 0,85. Узлы выходят из строя независимо друг от друга. Найти вероятность того, что в течение смены откажут ровно два узла.

Решение:

Из условия задачи

Примеры решения задач по теории вероятности

Используя формулу Бернулли, получим:

Примеры решения задач по теории вероятности

Пример №18

  • Определить вероятность того, что в семье, имеющей пять детей, будет три девочки и два мальчика. Вероятности рождения мальчика и девочки предполагаются одинаковыми.

Решение:

Из условия задачи

Примеры решения задач по теории вероятности

Используя формулу Бернулли, получим:

Примеры решения задач по теории вероятности

Пример №19

  • В условиях предыдущей задачи найти вероятность того, что среди детей будет не больше трех девочек.

Решение:

Примеры решения задач по теории вероятности

Пример №20

  • Вероятность попадания в цель стрелком равна 0,75. Сделано 20 выстрелов. Определить наивероятнейшее число попаданий в цель.

Решение:

Здесь

Примеры решения задач по теории вероятности

Следовательно, применим формулу

Примеры решения задач по теории вероятности

Получим:

Примеры решения задач по теории вероятности

т.е.

Примеры решения задач по теории вероятности

Наивероятнейшее число попаданий в цель равно 15.

Предельные теоремы в схеме Бернулли

Схема независимых испытаний служит вероятностной моделью многих реальных явлений, поэтому представляет значительный интерес задача подсчета вероятности Примеры решения задач по теории вероятности. При больших значениях Примеры решения задач по теории вероятности и Примеры решения задач по теории вероятности есть трудности в получении численного значения этих вероятностей.

Естественным образом возникает задача нахождения асимптотических форм, позволяющих приближенно вычислять вероятности Примеры решения задач по теории вероятности для достаточно больших Примеры решения задач по теории вероятности и малых Примеры решения задач по теории вероятности.

Теорема (Локальная предельная теорема Пуассона). Если Примеры решения задач по теории вероятности, так что Примеры решения задач по теории вероятности то

Примеры решения задач по теории вероятности

Теорема (Интегральная предельная теорема Пуассона). В схеме Бернулли для любого натурального числа Примеры решения задач по теории вероятности, любого Примеры решения задач по теории вероятности и для любого числового множества Примеры решения задач по теории вероятности справедливо неравенство:

Примеры решения задач по теории вероятности

Теперь рассмотрим асимптотическую формулу для вероятности не близкой к нулю.

Теорема (Локальная предельная теорема Муавра-Лапласа). Если в схеме Бернулли Примеры решения задач по теории вероятности, то для любого положительного с равномерно по всем Примеры решения задач по теории вероятности таких:

Примеры решения задач по теории вероятности

справедливо соотношение:

Примеры решения задач по теории вероятности

где Примеры решения задач по теории вероятности — бесконечно малая величина при Примеры решения задач по теории вероятности.

Теорема (Интегральная предельная теорема Муавра-Лапласа). При выполнении условий предыдущей теоремы равномерно Примеры решения задач по теории вероятности выполнено предельное соотношение:

Примеры решения задач по теории вероятности

Заметим, что при использовании интегральной формулы Муавра-Лапласа формула обеспечивает достаточную точность уже при Примеры решения задач по теории вероятности.

По полученным теоремам составим таблицу.

Примеры решения задач по теории вероятности

Пример №21

Решение:

Применим формулу Бернулли:

Примеры решения задач по теории вероятности

Пример №22

Решение:

Применим локальную теорему Муавра-Лапласа:

Примеры решения задач по теории вероятности

Пример №23

  • Фабрика выпускает 70% продукции I сорта. Чему равна вероятность того, что в партии из 1000 изделий число изделий I сорта будет в диапазоне [652, 760]?

Решение:

Применим интегральную теорему Муавра-Лапласа.

Примеры решения задач по теории вероятности

Отклонение относительной частоты от постоянной вероятности в независимых испытаниях

Вероятность того, что в Примеры решения задач по теории вероятности независимых испытаниях, в каждом из которых вероятность появления события равна Примеры решения задач по теории вероятности, абсолютная величина отклонения относительной частоты появления события не превысит положительного числа Примеры решения задач по теории вероятности, приближенно равна удвоенной функции Лапласа при Примеры решения задач по теории вероятности.

Примеры решения задач по теории вероятности

Относительная частота события Примеры решения задач по теории вероятности определяется равенством Примеры решения задач по теории вероятности, где Примеры решения задач по теории вероятности — число испытаний, в которых Примеры решения задач по теории вероятности наступило, Примеры решения задач по теории вероятности — общее число произвольных испытаний.

Пример №24

  • Вероятность появления события в каждом из независимых испытаний равна 0,5. Найти число испытаний Примеры решения задач по теории вероятности, при котором с вероятностью 0,7698 можно ожидать, что относительная частота появления события отклонится от его вероятности по абсолютной величине не более чем на 0,02.

Решение:

Их рассмотренной формулы:

Примеры решения задач по теории вероятности

получим, что

Примеры решения задач по теории вероятности

Пример №25

  • Вероятность выигрыша на турнире по баскетболу равна 0,58. Найти количество турниров Примеры решения задач по теории вероятности, при котором с вероятностью приблизительно равной 0,9 можно ожидать, что относительная частота побед отклонится от вероятности по абсолютной величине не более чем на 0,1.

Решение:

Примеры решения задач по теории вероятности
Примеры решения задач по теории вероятности

Случайные величины и их распределения

В азартных играх интерес играющих вызывает не наступление случайного исхода, а связанный с ним выигрыш или проигрыш, т.е. определенная числовая величина, которая соответствует исходу.

Примером случайной величины может быть число очков, выпавших при подбрасывании кубика, число бракованных изделий среди общего числа изделий.

Случайная величина Примеры решения задач по теории вероятности есть число, которое ставится в соответствие каждому возможному исходу эксперимента, т.е. ее можно рассматривать как функцию Примеры решения задач по теории вероятности на пространстве элементарных событий Примеры решения задач по теории вероятности.

Пусть Примеры решения задач по теории вероятности — произвольное вероятностное пространство. Случайной величиной называется функция Примеры решения задач по теории вероятности, такая что для любого Примеры решения задач по теории вероятности выполняется следующее:

Примеры решения задач по теории вероятности

Определим функцию распределения случайной величины, которая несет всю информацию, заложенную в случайной величине.

Функцией распределения случайной величины Примеры решения задач по теории вероятности называется функция

Примеры решения задач по теории вероятности

такая, что для любого действительного Примеры решения задач по теории вероятности выполняется:

Примеры решения задач по теории вероятности

Любая функция распределения обладает следующими свойствами:

1) Примеры решения задач по теории вероятности

2) существуют пределы Примеры решения задач по теории вероятности.

3) функция непрерывна слева, т.е. Примеры решения задач по теории вероятности.

4) Примеры решения задач по теории вероятности

5) Примеры решения задач по теории вероятности

Классификация дискретных случайных величин

Дискретная случайная величина — это случайная величина, которая принимает не более чем счетное число значений.

Пусть ее значения Примеры решения задач по теории вероятности… такие, что Примеры решения задач по теории вероятности….

Тогда

Примеры решения задач по теории вероятности

Совокупность значений Примеры решения задач по теории вероятности и соответствующих вероятностей Примеры решения задач по теории вероятности называется распределением дискретной случайной величины.

Закон распределения такой величины может быть таблично следующим образом:

Примеры решения задач по теории вероятности

Примеры решения задач по теории вероятности

Закон распределения дискретной случайной величины можно изобразить графически, для чего в прямоугольной системе координат строят точки Примеры решения задач по теории вероятностиПримеры решения задач по теории вероятности, где Примеры решения задач по теории вероятности — возможные значения Примеры решения задач по теории вероятности — соответствующие вероятности; и соединяют их отрезками прямых. Полученную фигуру называют многоугольником распределения (полигоном).

Пример №26

  • Найти функцию распределения случайной величины, которая представлена таблицей:

Примеры решения задач по теории вероятности

Решение:

Запишем функцию распределения в виде сложной функции:

Примеры решения задач по теории вероятности

  • Два шахматиста Миша и Коля делают по одному ходу. Вероятность удачного хода Мишей равна 0,7, а для Коли эта вероятность равна 0,76. Найти ряд распределения суммарного числа удачных ходов шахматистами.
Примеры решения задач по теории вероятности

Пример №26.7

  • Партия изделий содержит 10% нестандартных. Пусть случайная величина Примеры решения задач по теории вероятности — число стандартных изделий в партии из пяти изделий. Требуется составить закон распределения случайной величины и записать функцию распределения.

Решение:

Случайная величина Примеры решения задач по теории вероятности может принимать значения Примеры решения задач по теории вероятности.

ВероятностьПримеры решения задач по теории вероятности найдем по формуле Бернулли:

Примеры решения задач по теории вероятности

По условию задачи

Примеры решения задач по теории вероятности
Примеры решения задач по теории вероятности

Запишем закон распределения случайной величины:

Примеры решения задач по теории вероятности

Найдем функцию распределения. По определению:

Примеры решения задач по теории вероятности
Примеры решения задач по теории вероятности

Окончательно получим:

Примеры решения задач по теории вероятности

Классификация абсолютно непрерывных случайных величин

Если случайная величина Примеры решения задач по теории вероятности принимает любые значения из некоторых интервалов или отрезков числовой оси, то она называется непрерывной случайной величиной. Примерами такой величины являются дальность полета снаряда, время безотказной работ прибора.

Плотностью распределения вероятностей случайной величины Примеры решения задач по теории вероятности в точке Примеры решения задач по теории вероятности Примеры решения задач по теории вероятности называется предел:

Примеры решения задач по теории вероятности

Теорема. Для того, чтобы случайная величина Примеры решения задач по теории вероятности была абсолютно непрерывной, необходимо и достаточно, чтобы:

Примеры решения задач по теории вероятности

Распределение случайной величины Примеры решения задач по теории вероятности называется непрерывным, а сама случайная величина — абсолютно непрерывной случайной величиной, если

Примеры решения задач по теории вероятности

где Примеры решения задач по теории вероятности — минимальная Примеры решения задач по теории вероятности — алгебра.

Свойства плотности распределения:

Примеры решения задач по теории вероятности

Эти три свойства выполняются для любой точки непрерывности функции.

Примеры решения задач по теории вероятности

Примеры решения задач по теории вероятности

Примеры решения задач по теории вероятности

Примеры решения задач по теории вероятности

Пример №27

  • Плотность вероятности случайных амплитуд боковой качки корабля подчиняется закону Рэлея с параметрами Примеры решения задач по теории вероятности. Найти вероятность того, что значение случайной амплитуды будет находиться в диапазоне 0,1 до 0,6.

Решение:

Примеры решения задач по теории вероятности

Пример №28

  • Непрерывная случайная величина задана плотностью распределения:
Примеры решения задач по теории вероятности

Требуется найти значение параметра с и записать функцию распределения.

Решение:

Значение параметра с определим, используя свойство плотности распределения:

Примеры решения задач по теории вероятности

Примеры решения задач по теории вероятности

Функцию распределения определим из условия:

Примеры решения задач по теории вероятности

Пример №29

  • Дана функция распределения случайной величины. Найти ее плотность распределения.
Примеры решения задач по теории вероятности

Решение:

Плотность распределения определим из свойства плотности распределения:

Примеры решения задач по теории вероятности
Примеры решения задач по теории вероятности

Некоторые законы распределения случайных величин

Примеры решения задач по теории вероятности

Примеры решения задач по теории вероятности

Пример №30

  • Автобусы некоторого маршрута ходят строго по расписанию. Интервал движения 5 мин. Найти вероятность того, что пассажир, подошедший к остановке, будет ожидать очередной автобус менее 3 минут.

Решение:

Случайная величина Примеры решения задач по теории вероятности — время прихода пассажира на остановку, распределена равномерно на [0; 5]. Плотность распределения вероятностей имеет вид:

Примеры решения задач по теории вероятности

Пассажир будет ожидать автобус менее 3 минут, если он подойдет к остановке в интервале времени от 2 до 5 минут после отправления автобуса.

Примеры решения задач по теории вероятности

Пример №31

  • Телефонная станция обслуживает 400 абонентов. Вероятность того, что в течение часа абонент позвонит на станцию, равна 0,01 и постоянна для всех абонентов. Найти вероятность того, что на станцию в течение часа позвонят не более двух абонентов.

Решение:

Дискретная случайная величина распределена по закону Пуассона. Воспользуемся формулой Пуассона:

Примеры решения задач по теории вероятности

По условию задачи

Примеры решения задач по теории вероятности
Примеры решения задач по теории вероятности

Пример №32

  • Время Примеры решения задач по теории вероятности безотказной работы двигателя автомобиля распределено по показательному закону. Известно, что среднее время наработки двигателя на отказ между техническим обслуживанием — 100 ч. Определить вероятность безотказной работы двигателя в течение 80 ч.

Решение:

По условию задачи математическое ожидание случайной величины Примеры решения задач по теории вероятности равно 100 ч. Следовательно,

Примеры решения задач по теории вероятности

Тогда плотность распределения времени безотказной работы двигателя имеет вид:

Примеры решения задач по теории вероятности

Функция распределения случайной величины Примеры решения задач по теории вероятности принимает вид:

Примеры решения задач по теории вероятности

и определяет вероятность отказа двигателя за время продолжительностью Примеры решения задач по теории вероятности. Тогда вероятность безотказной работы двигателя за это время будет равна:

Примеры решения задач по теории вероятности

Функцию Примеры решения задач по теории вероятности называют функцией надежности. Для нашего случая

Примеры решения задач по теории вероятности

Основные числовые характеристики случайных величии

Примеры решения задач по теории вероятности

Свойства математического ожидания:
1) Математическое ожидание числа появлений события Примеры решения задач по теории вероятности в одном испытании равно вероятности Примеры решения задач по теории вероятности наступления этого испытания.

Примеры решения задач по теории вероятности

Заметим, что математическое ожидание случайной величины Примеры решения задач по теории вероятности можно трактовать как вероятностное среднее этой величины.

Для любой случайной величины Примеры решения задач по теории вероятности случайная величина Примеры решения задач по теории вероятности называется центрированной случайной величиной или отклонением.

Пусть случайная величина Примеры решения задач по теории вероятности определена на вероятностном пространстве Примеры решения задач по теории вероятности. Для Примеры решения задач по теории вероятности величина Примеры решения задач по теории вероятности, если она определена, называется моментом Примеры решения задач по теории вероятности-го порядка случайной величины

Величина Примеры решения задач по теории вероятности называется абсолютным моментом Примеры решения задач по теории вероятности-го порядка случайной величины

Моменты случайной величины Примеры решения задач по теории вероятности называются центральными моментами случайной величины Примеры решения задач по теории вероятности.

Центральные моменты четного порядка случайной величины Примеры решения задач по теории вероятности характеризуют степень разброса значений относительно ее среднего значения.

Дисперсией случайной величины Примеры решения задач по теории вероятности называется число Примеры решения задач по теории вероятности, число Примеры решения задач по теории вероятности называется среднеквадратическим отклонением случайной величины Примеры решения задач по теории вероятности.

Примеры решения задач по теории вероятности

Свойства дисперсии случайной величины:

Примеры решения задач по теории вероятности

Формулы вычисления математического ожидания и дисперсии для некоторых случайных величин:

Примеры решения задач по теории вероятности

Примеры решения задач по теории вероятности

Ковариацией случайной величины Примеры решения задач по теории вероятности и Примеры решения задач по теории вероятности называется число:

Примеры решения задач по теории вероятности

Если математическое ожидание случайной величины Примеры решения задач по теории вероятности является характеристикой ее положения, средним значением, около которого группируются значения случайной величины, то дисперсия и среднеквадратического отклонение являются характеристиками рассеяния случайной величины около математического ожидания.

Пример №33

  • Найти математическое ожидание, дисперсию и среднеквадратическое отклонение случайной величины Примеры решения задач по теории вероятности

Примеры решения задач по теории вероятности

Решение:

Примеры решения задач по теории вероятности

Пример №34

  • Найти математическое ожидание, дисперсию и среднеквадратическое отклонение непрерывной случайной величины Примеры решения задач по теории вероятности, заданной плотностью распределения:
Примеры решения задач по теории вероятности

Решение:

Примеры решения задач по теории вероятности

Другие характеристики случайных величин

Примеры решения задач по теории вероятности

Кроме рассмотренных выше числовых характеристик случайной величины, в приложениях используются так называемые квантили.

Квантилью уровня Примеры решения задач по теории вероятности случайной величины Примеры решения задач по теории вероятности называется решение уравнения:

Примеры решения задач по теории вероятности

Квантили Примеры решения задач по теории вероятности имеют названия нижняя квартиль, медиана, верхняя квартиль. Они делят числовую прямую на четыре части, вероятности попадания в которые равны 0,25.

Пример №35

  • Найти моду случайной величины Примеры решения задач по теории вероятности, заданной распределением:
Примеры решения задач по теории вероятности

Решение:

Поскольку для моды выполняется равенство:

Примеры решения задач по теории вероятности

Наибольшая вероятность достигается при

Примеры решения задач по теории вероятности

Пример №36

  • Найти эксцесс случайной величины Примеры решения задач по теории вероятности:

Примеры решения задач по теории вероятности

Решение:

Найдем начальные моменты случайной величины Примеры решения задач по теории вероятности первых четырех порядков:

Примеры решения задач по теории вероятности

Найдем центральные моменты случайной величины Примеры решения задач по теории вероятности первых четырех порядков:

Примеры решения задач по теории вероятности

Тогда среднеквадратическое отклонение случайной величины Примеры решения задач по теории вероятности:

Примеры решения задач по теории вероятности

Эксцесс случайной величины Примеры решения задач по теории вероятности найдем по формуле:

Примеры решения задач по теории вероятности

Свойства нормальной случайной величины:

1) Примеры решения задач по теории вероятности, график функции расположен выше оси Примеры решения задач по теории вероятности.

2) Ось Примеры решения задач по теории вероятности служит асимптотой графика функции Примеры решения задач по теории вероятности, т.к. Примеры решения задач по теории вероятности.

3) Функция Примеры решения задач по теории вероятности имеет один максимум при Примеры решения задач по теории вероятности равный Примеры решения задач по теории вероятности.

4) График функции Примеры решения задач по теории вероятности симметричен относительно прямой Примеры решения задач по теории вероятности.

5) Точки

Примеры решения задач по теории вероятности

являются точками перегиба графика функции Примеры решения задач по теории вероятности.

Вероятность попадания случайной величины

Примеры решения задач по теории вероятности

на заданный участок Примеры решения задач по теории вероятности:

Примеры решения задач по теории вероятности

Случайная величина Примеры решения задач по теории вероятности принимает свое значения в промежутке Примеры решения задач по теории вероятности с вероятностью 0,9973.

Вероятность отклонения нормально распределенной случайной величины от ее математического ожидания по абсолютной величине определяется по формуле:

Примеры решения задач по теории вероятности

Вероятность отклонения относительной частоты Примеры решения задач по теории вероятности от вероятности наступления события Примеры решения задач по теории вероятности в серии из Примеры решения задач по теории вероятности независимых испытаний выражается формулой:

Примеры решения задач по теории вероятности

Пример №37

  • Суточное потребление электроэнергии исправной печью является случайной величиной, распределенной по нормальному закону со средним 1000 кВт и среднеквадратичным отклонением 50. Если суточное потребление превысит 1100 кВт, то по инструкции печь отключают и ремонтируют. Найти вероятность ремонта печи.

Решение:

Случайная величина Примеры решения задач по теории вероятности есть суточное потребление электроэнергии печью.

Примеры решения задач по теории вероятности

Найдем вероятность попадания случайной величины Примеры решения задач по теории вероятности в интервал (0; 1100). Для этого воспользуемся формулой:

Примеры решения задач по теории вероятности

Тогда вероятность ремонта печи равна 1-0,9544 = 0,0456.

Пример №38

  • Рост мальчиков возрастной группы 15 лет есть нормально распределённая случайная величина Примеры решения задач по теории вероятности с параметрами
Примеры решения задач по теории вероятности

Какую долю костюмов для мальчиков, имеющих рост от 152 до 158 см, нужно предусмотреть в объёме производства для данной возрастной группы.

Решение:

Примеры решения задач по теории вероятности

Найдем вероятность попадания случайной величины Примеры решения задач по теории вероятности в интервал (152; 158). Для этого воспользуемся формулой:

Примеры решения задач по теории вероятности

Пример №39

  • Текущая оценка ценной бумаги представляет собой нормально распределенную случайную величину со средним значением 100 у. е. и дисперсией 9. Найти вероятность того, что цена актива (ценной бумаги) будет находиться в пределах от 91 до 109 у. е.

Решение:

Так как

Примеры решения задач по теории вероятности

тогда

Примеры решения задач по теории вероятности

Неравенство Чебышева

Необходимо рассмотреть условия, при выполнении которых совокупное действие очень многих случайных причин приводит к результату, почти не зависящему от случая, так как позволяет предвидеть ход явлений. Эти условия и указываются в теоремах, носящих общее название закона больших чисел. К ним относятся теоремы Чебышева и Бернулли.

Теорема Чебышева является наиболее общим законом больших чисел, теорема Бернулли — простейшим.

Теорема (Неравенство Чебышева). Вероятность того, что отклонение случайной величины Примеры решения задач по теории вероятности от ее математического ожидания по абсолютной величине меньше положительного числа Примеры решения задач по теории вероятности, не меньше чем

Примеры решения задач по теории вероятности
Примеры решения задач по теории вероятности

Теорема Чебышева. Если последовательность попарно независимых случайных величин Примеры решения задач по теории вероятности… имеет конечное математическое ожидание и дисперсии этих величин равномерно ограничены (не превышают постоянного числа С), то среднее арифметическое случайной величины сходится по вероятности к среднему арифметическому их математического ожидания, т.е. если Примеры решения задач по теории вероятности — любое положительное число, то

Примеры решения задач по теории вероятности

В этом случае среднее арифметическое достаточно большого числа независимых случайных величин (дисперсии которых равномерно ограничены) утрачивает характер случайной величины.

Если проводится серия измерений какой-либо физической величины, причем:

1) результат каждого измерения не зависит от результатов остальных (измерения попарно независимы);

2) измерения производятся без систематических ошибок, (имеют одно и то же математическое ожидание);

3) обеспечена определенная точность измерений, (дисперсии их ограничены)

то при достаточно большом числе измерений их среднее арифметическое окажется сколь угодно близким к истинному значению измеряемой величины.

Теорема Бернулли. Если в каждом из независимых опытов вероятность появления события Примеры решения задач по теории вероятности постоянна, то при достаточно большом числе испытаний вероятность того, что модуль отклонения относительной частоты появлений Примеры решения задач по теории вероятности в Примеры решения задач по теории вероятности опытах от Примеры решения задач по теории вероятности будет сколь угодно малым, как угодно близка к 1.

Закон больших чисел не исследует вид предельного закона распределения суммы случайных величин. Этот вопрос рассмотрен в группе теорем, называемых центральной предельной теоремой. Они утверждают, что закон распределения суммы случайных величин, каждая из которых может иметь различные распределения, приближается к нормальному при достаточно большом числе слагаемых. Этим объясняется важность нормального закона для практических приложений.

Центральная предельная теорема. Если Примеры решения задач по теории вероятности — независимые случайные величины с одинаковым законом распределения, математическим ожиданием Примеры решения задач по теории вероятности и дисперсией Примеры решения задач по теории вероятности, то при неограниченном увеличении Примеры решения задач по теории вероятности закон распределения суммы Примеры решения задач по теории вероятности неограниченно приближается к нормальному.

Пример №40

Примеры решения задач по теории вероятности

Решение:

По неравенству Чебышева:

Примеры решения задач по теории вероятности

Найдем математическое ожидание случайной величины Примеры решения задач по теории вероятности.

Примеры решения задач по теории вероятности

Пример №41

Примеры решения задач по теории вероятности

Решение:

Математическое ожидание случайной величины принимает значение:

Примеры решения задач по теории вероятности

Дисперсия равна

Примеры решения задач по теории вероятности

Пример №42

  • В ящике 20 деталей, из которых 4 бракованные.

Найти вероятность того, что наугад взятая из ящика деталь окажется бракованной.

Решение:

Так как каждая из имеющихся деталей может быть из ящика, то число всех равновозможных элементарных исходов Примеры решения задач по теории вероятности. Число исходов, благоприятствующих появлению бракованной детали, Примеры решения задач по теории вероятности. Если событие Примеры решения задач по теории вероятности означает, что взятая деталь бракованная, то

Примеры решения задач по теории вероятности

Пример №43

  • Для сигнализации об аварии установлены два независимо работающих сигнализатора. Вероятность того, что при аварии сигнализатор сработает, равна 0,95 для первого сигнализатора и 0,9 для второго.

Найти, вероятность того, что при аварии сработает только один сигнализатор.

Решение:

Вероятности появления каждого из двух независимых событий Примеры решения задач по теории вероятности и Примеры решения задач по теории вероятности соответственно равны Примеры решения задач по теории вероятности и Примеры решения задач по теории вероятности.

Примеры решения задач по теории вероятности — появилось только событие Примеры решения задач по теории вероятности;

Примеры решения задач по теории вероятности — появилось только событие Примеры решения задач по теории вероятности.

Появление события Примеры решения задач по теории вероятности равносильно появлению события Примеры решения задач по теории вероятности (появилось первое событие и не появилось второе), т.е. Примеры решения задач по теории вероятности.

Появление события Примеры решения задач по теории вероятности равносильно появлению события Примеры решения задач по теории вероятности (появилось второе событие и не появилось первое). Примеры решения задач по теории вероятности.

Таким образом, чтобы найти вероятность появления одного из событий Примеры решения задач по теории вероятности и Примеры решения задач по теории вероятности достаточно найти вероятность появления одного, безразлично какого, из событий Примеры решения задач по теории вероятности и Примеры решения задач по теории вероятности. События Примеры решения задач по теории вероятности и Примеры решения задач по теории вероятности несовместны, поэтому применима теорема сложения:

Примеры решения задач по теории вероятности

События Примеры решения задач по теории вероятности и Примеры решения задач по теории вероятности — независимы, следовательно, независимы события Примеры решения задач по теории вероятности и Примеры решения задач по теории вероятности, а также Примеры решения задач по теории вероятности и Примеры решения задач по теории вероятности, поэтому применима теорема умножения:

Примеры решения задач по теории вероятности

Подставив эти вероятности, найдем искомую вероятность появления только одного из событий Примеры решения задач по теории вероятности и Примеры решения задач по теории вероятности:

Примеры решения задач по теории вероятности

Пример №44

  • В вычислительной лаборатории имеется 6 компьютеров одного типа и 4 другого. Вероятность того, что на время выполнения некоторого расчета компьютер I типа не выйдет из строя, равна 0,95; для компьютера другого типа — 0,8. Студент производит расчет на наудачу взятом компьютере.

Найти вероятность того, что до окончания расчета компьютер не выйдет из строя.

Решение:

Обозначим через Примеры решения задач по теории вероятности — компьютер не выйдет из строя до окончания расчета. Возможны следующие предположения (гипотезы) о первоначальном качестве компьютеров; всего компьютеров в лаборатории — 10, из них:

Примеры решения задач по теории вероятности

В сумме гипотезы всегда равны 1, проверим:

Примеры решения задач по теории вероятности

Условная вероятность того, что студент воспользуется компьютером 1-го типа, равна Примеры решения задач по теории вероятности; 2-го типа — Примеры решения задач по теории вероятности.

Искомая вероятность того, что до окончания расчета компьютер любого типа не выйдет из строя, равна:

Примеры решения задач по теории вероятности

Пример №45

  • Два автомата производят одинаковые детали, которые сбрасываются на общий конвейер. Производительность первого автомата вдвое больше производительности второго. Первый автомат производит в среднем 78% деталей отличного качества, а второй — 86%.

Найти вероятность того, что эта деталь произведена первым автоматом.

Решение:

Обозначим событие Примеры решения задач по теории вероятности — деталь отличного качества. Можно сделать два предположения (гипотезы): Примеры решения задач по теории вероятности — деталь произведена первым автоматом;

Примеры решения задач по теории вероятности — деталь произведена вторым автоматом; Примеры решения задач по теории вероятности.

Условная вероятность того, что деталь будет отличного качества, если она произведена первым автоматом, равна

Примеры решения задач по теории вероятности

Условная вероятность того, что деталь будет отличного качества, если она произведена вторым автоматом, равна

Примеры решения задач по теории вероятности

Вероятность того, что наудачу взятая деталь окажется отличного качества, по формуле полной вероятности равна

Примеры решения задач по теории вероятности

Искомая вероятность того, что взятая отличная деталь произведена первым автоматом, по формуле Байсса равна

Примеры решения задач по теории вероятности

Пример №46

  • При измерении роста у 18 студентов установлено, что у трех рост — 188 см; у четверых — 182 см; у пятерых — 180 см; у шестерых 178 см. Примеры решения задач по теории вероятности — рост студента.

Записать закон распределения Примеры решения задач по теории вероятности. Вычислить математическое ожидание Примеры решения задач по теории вероятности, дисперсию Примеры решения задач по теории вероятности и среднеквадратическое отклонение Примеры решения задач по теории вероятности.

Решение:

Вероятность обнаружения среди 18 студентов троих с ростом 188 см:

Примеры решения задач по теории вероятности

Аналогично вероятность обнаружения среди 18 студентов четверых с ростом 182 см:

Примеры решения задач по теории вероятности

Получаем закон распределения в виде следующей таблицы:

Примеры решения задач по теории вероятности

Далее находим:

Примеры решения задач по теории вероятности

Пример №47

  • Устройство состоит из четырех независимо работающих элементов. Вероятность отказа каждого элемента равна 0,2.

Записать закон распределения числа отказавших элементов устройства, найти математическое ожидание и дисперсию.

Решение:

Дискретная случайная величина Примеры решения задач по теории вероятности может принимать значения Примеры решения задач по теории вероятности Примеры решения задач по теории вероятности. Так как Примеры решения задач по теории вероятности то по формуле Бернулли находим:

Примеры решения задач по теории вероятности

Математическое ожидание равно

Примеры решения задач по теории вероятности

Элементы теории вероятностей

Событие и вероятность

Под событием мы будем понимать всякое явление, которое происходит или не происходит. Легко понять, что эта фраза отнюдь не может служить точным определением в том смысле, как мы понимаем математическое определение, однако мы вынуждены ею ограни­читься.

Для большей ясности приведем некоторые примеры. Так, например, событием является выпадение герба при бросании монеты, выпадение того или иного числа очков (например, шестерки) при бросании игральной кости, попадание в цель при выстреле, нахож­дение молекулы газа в заранее выделенном объеме, опоздание в шко­лу, приход в школу вовремя, прочтение (или непрочтение) этой книги (или «Евгения Онегина»)…

Различные события мы будем обозначать буквами A, B, C, … .

Событие называют достоверным, если оно непременно должно произойти. Так, достоверным является выпадение не более шести очков при бросании обычной игральной кости, появление белого шара при извлечении из урны, содержащей только белые ша­ры, и т. п.

Наоборот, событие называют невозможным, если оно заведомо не наступит. Примерами невозможных событий являются извлечение более четырех тузов из обычной карточной колоды, появление красного шара из урны, содержащей лишь белые и черные шары, и т. п.

Пусть А — некоторое событие. Под событием, противоположным ему, будем понимать событие, состоящее в том, что А не наступило. Его обозначают через Теория вероятностей решение задач, заданий  и примеры Если, скажем, событие A состоит в появле­нии красной масти при вытаскивании карты из колоды, то Теория вероятностей решение задач, заданий  и примеры озна­чает появление черной.

События А и В называют несовместными, если наступление одного из них исключает возможность наступления другого. Так, появление любого возможного числа очков при бросании игральной кости (событие А) несовместно с появлением иного числа (событие В). Выпадение четного числа очков несовместно с выпадением нечетного числа. Наоборот, выпадение четного числа очков (событие А) и числа очков, кратного трем (событие В), не будут несовместными, ибо выпадение шести очков означает наступление и события А , и события В у так что наступление одного из них не исключает наступ­ления другого. Легко понять, что события А и Теория вероятностей решение задач, заданий  и примеры всегда будут не­ совместными.

Рассмотрим некоторую совокупность событий А , В, …, L. Эти события принято называть единственно возможными, если в резуль­тате каждого испытания хотя бы одно из них наверное наступит. Говорят также, что рассматриваемые события образуют полную группу событий. Так, например, при бросании игральной кости полную группу образуют события, состоящие в выпадении одного, двух, трех, четырех, пяти и шести очков.

Теперь мы можем перейти к рассмотрению важнейшего понятия вероятности события.

Рассмотрим систему конечного числа событий Теория вероятностей решение задач, заданий  и примеры относительно которой сделаем следующие предположения:

1. Эти события попарно несовместны; иначе говоря, для любых двух событий Теория вероятностей решение задач, заданий  и примеры появление одного из них исключает появление другого.

2. События Теория вероятностей решение задач, заданий  и примеры единственно возможны, то есть какое-либо одно из них непременно должно наступить.

3. События Теория вероятностей решение задач, заданий  и примеры равновозможны. Это означает, что не существует никаких объективных причин, вследствие которых одно из них могло бы наступать чаще, чем какое-либо другое.

Пусть имеется событие А , которое наступает при появлении некоторых из наших «элементарных» событий Теория вероятностей решение задач, заданий  и примеры и не на­ступает при появлении других. Мы будем говорить в таком случае, что те из «элементарных» событий Теория вероятностей решение задач, заданий  и примеры при наступлении которых наступает также событие A, благоприятствуют событию A.

Допустим, что из общего числа n рассматриваемых событий Теория вероятностей решение задач, заданий  и примерыТеория вероятностей решение задач, заданий  и примеры событию A благоприятствует m из них. Тогда вероятностью события А называется отношение числа событий, благо­приятствующих событию А , к общему числу всех равновозможных событий. Если, как это принято, обозначить вероятность события A через Р (A), то мы получаем по определению

Теория вероятностей решение задач, заданий  и примеры

Поясним приведенное нами определение примером. Рассмотрим бросание игральной кости и обозначим через Теория вероятностей решение задач, заданий  и примеры

события, состоящие в выпадении соответственно одного, двух шести оч­ков. Легко проверить, что эти события удовлетворяют всем сделан­ным выше предположениям.

Отсюда следует, что

Теория вероятностей решение задач, заданий  и примеры

потому что каждому из этих событий благоприятствует только оно само, так что здесь m = 1, а n = 6.

Если событие А означает появление четного числа очков, то ему благоприятствуют события Теория вероятностей решение задач, заданий  и примеры состоящие в появлении двух, четырех и шести очков. Поэтому для события А имеем m = 3, так что Теория вероятностей решение задач, заданий  и примеры

Пусть событие В состоит в появлении числа очков, кратного трем. Тогда событию B благоприятствуют «элементарные» события Теория вероятностей решение задач, заданий  и примерыоткуда следует, что для события В имеем m = 2. ПоэтомуТеория вероятностей решение задач, заданий  и примеры

Легко заметить, что для любого события А число благоприятствующих событий m удовлетворяет неравенствам Теория вероятностей решение задач, заданий  и примеры Поэтому вероятность любого события А подчинена условиям

Теория вероятностей решение задач, заданий  и примеры

Далее, если обозначить через Е некоторое достоверное событие, то ему, очевидно, должны благоприятствовать все элементарные» события Теория вероятностей решение задач, заданий  и примеры так что для него должно быть m = n. Поэтому вероятность достоверного события равна единице:

Теория вероятностей решение задач, заданий  и примеры

Если, наоборот, UI — невозможное событие, то из самого определения следует, что здесь m — 0, так что вероятность невозможно­го события равна нулю:

Теория вероятностей решение задач, заданий  и примеры

Рассмотрим несколько примеров, разъясняющих введенное нами понятие вероятности.

Пример:

В урне находятся три синих, восемь красных и девять белых шаров одинакового размера и веса, неразличимых наощупь. Шары тщательно перемешаны. Какова вероятность по­явления синего, красного и белого шаров при одном вынимании шара из урны.

Решение:

Так как появление любого шара можно считать равновозможным, то мы имеем всего n = 3 + 8 + 9 = 20 элементарных событий. Если через A, В, С обозначить события, состоя­щие в появлении соответственно синего, красного и белого шаров, а через Теория вероятностей решение задач, заданий  и примеры — число благоприятствующих этим событиям случаев, то ясно, что Теория вероятностей решение задач, заданий  и примеры Поэтому

Теория вероятностей решение задач, заданий  и примеры

Пример:

Одновременно брошены две монеты. Какова вероятность появления m гербов (m = 0, 1, 2)?

Решение:

Рассмотрим возможные при бросании двух монет исходы. Очевидно, их можно описать схемой

Теория вероятностей решение задач, заданий  и примеры

где Г означает выпадение герба, а Р — надписи. Таким образом, возможны четыре элементарных события. Поскольку монеты пред­полагаются однородными и имеющими геометрически правильную форму, то нет никаких оснований предполагать, что одна из сторон какой-либо монеты выпадает чаще других. Поэтому все четыре слу­чая следует считать равновозможными. Но тогда, обозначив через Теория вероятностей решение задач, заданий  и примеры вероятность выпадения m гербов, легко получим:

Теория вероятностей решение задач, заданий  и примеры

Пример:

Одновременно бросаются две игральные кости, на гранях которых нанесены очки 1, 2, 3, 4, 5, 6. Какова вероят­ность того, что сумма очков, выпавших на двух костях, равна восьми?

Решение:

Так как любое из возможного числа очков на од­ной кости может сочетаться с любым числом очков на другой, то общее число различных случаев равно n = 6 • 6 = 36. Легко убе­диться в том, что все эти случаи попарно несовместны, равновозмож­ны и образуют полную группу событий. Для ответа на вопрос следует подсчитать, в каком числе случаев сумма очков равна восьми. Это будет, если число очков на брошенных костях равно

Теория вероятностей решение задач, заданий  и примеры

причем первое слагаемое означает число очков на первой, а второе — на второй кости. Отсюда видно, что событию A, состоящему в том, что сумма очков, выпавших на двух костях, равна восьми, благо­приятствует m=5 случаев. Поэтому

Теория вероятностей решение задач, заданий  и примеры

Сложные вероятности. Теоремы сложения и умножения. Условные вероятности

Непосредственный подсчет случаев, благоприятствующих дан­ному событию, может оказаться затруднительным. Поэтому для определения вероятности события бывает выгодно представить дан­ное событие в виде комбинации некоторых других, более простых событий. При этом, однако, надо знать правила, которым подчи­няются вероятности при комбинации событий. Именно к этим пра­вилам и относятся упомянутые в названии параграфа теоремы.

Первая из них относится к подсчету вероятности того, что осуществится хотя бы одно из нескольких событий.

Теорема сложения:

Пусть А и В — два несовместных события. Тогда вероятность того, что осуществится хотя бы одно из этих двух событий, равна сумме их вероятностей:

Теория вероятностей решение задач, заданий  и примеры

Доказательство:

Пусть Теория вероятностей решение задач, заданий  и примеры — полная группа попарно несовместных событий. Если Теория вероятностей решение задач, заданий  и примеры, Теория вероятностей решение задач, заданий  и примеры то среди этих n элементарных событий имеется ровно Теория вероятностей решение задач, заданий  и примеры событий, благоприятствующих A, и ровно Теория вероятностей решение задач, заданий  и примеры событий, благоприятствующих В. Так как события А и В несовместны, то никакое из событий не может благоприятствовать обоим этим собы­тиям. Событию (А или В), состоящему в том, что наступает хотя бы одно из этих двух событий, благоприятствует, очевидно, как каждое из событий Теория вероятностей решение задач, заданий  и примеры благоприятствующих А, так и каждое из событий Теория вероятностей решение задач, заданий  и примеры благоприятствующих В . Поэтому общее число событий, благо­приятствующих событию (А или В), равно сумме Теория вероятностей решение задач, заданий  и примеры откуда следует:

Теория вероятностей решение задач, заданий  и примеры

что и требовалось доказать .

Нетрудно видеть, что теорема сложения, сформулированная выше для случая двух событий, легко переносится на случай любого конечного числа их. Именно если A, В, С, …L— попарно не­ cовместные события, то

Теория вероятностей решение задач, заданий  и примеры

Для случая трех событий, например, можно написать

Теория вероятностей решение задач, заданий  и примеры

откуда уже вытекает наше утверждение. Далее следует воспользоваться методом математической индукции.

Важным следствием теоремы сложения является утверж­дение: если события Теория вероятностей решение задач, заданий  и примеры попарно несовместны и единствен­но возможны, то

Теория вероятностей решение задач, заданий  и примеры

Действительно, событие (Теория вероятностей решение задач, заданий  и примеры или Теория вероятностей решение задач, заданий  и примеры или … или Теория вероятностей решение задач, заданий  и примеры) по предполо­жению достоверно и его вероятность, как было указано в § 1, равна единице. В частности, если A и Теория вероятностей решение задач, заданий  и примеры означают два взаимно противо­положных события, то

Теория вероятностей решение задач, заданий  и примеры

Проиллюстрируем теорему сложения примерами.

Пример:

При стрельбе по мишени вероятность сделать отличный выстрел равна 0,3, а вероятность сделать выстрел на оценку «хорошо» равна 0,4. Какова вероятность получить за сделанный выстрел оценку не ниже «хорошо»?

Решение:

Если событие A означает получение оценки «от­лично», а событие В — получение оценки «хорошо», то

Теория вероятностей решение задач, заданий  и примеры

Пример:

В урне, содержащей n шаров белого, красного и черного цвета, находятся к белых шаров и l красных. Какова вероятность вынуть шар не черного цвета?

Решение:

Если событие Л состоит в появлении белого, а событие В — красного шара, то появление шара не черного цвета означает появление либо белого, либо красного шара. Так как по определению вероятности

Теория вероятностей решение задач, заданий  и примеры

то по теореме сложения вероятность появления шара не черного цвета равна;

Теория вероятностей решение задач, заданий  и примеры

Эту задачу можно решить и так. Пусть событие С состоит в появлении черного шара. Число черных шаров равно n — (k + l), так что Теория вероятностей решение задач, заданий  и примеры Появление шара не черного цвета является противоположным событием Теория вероятностей решение задач, заданий  и примеры поэтому на основании указанного выше следствия из теоремы сложения имеем:

Теория вероятностей решение задач, заданий  и примеры

как и раньше.

Пример:

В денежно-вещевой лотерее на серию в 1000 би­летов приходится 120 денежных и 80 вещевых выигрышей. Какова вероятность какого-либо выигрыша на один лотерейный билет?

Решение:

Если обозначить через А событие, состоящее в выпадении денежного выигрыша и через В — вещевого, то из опре­деления вероятности следует

Теория вероятностей решение задач, заданий  и примеры

Интересующее нас событие представляет (А или В), поэтому из теоремы сложения вытекает

Теория вероятностей решение задач, заданий  и примеры

Таким образом, вероятность какого-либо выигрыша равна 0,2.

Прежде чем перейти к следующей теореме, необходимо ознакомиться с новым важным понятием — понятием условной вероятно­сти. Для этой цели мы начнем с рассмотрения следующего примера.

Пусть на складе имеется 400 электрических лампочек, изготовленных на двух различных заводах, причем на первом изготовлено 75% всех лампочек, а на втором — 25%. Допустим, что среди лампо­чек, изготовленных первым заводом, 83% удовлетворяют условиям определенного стандарта, а для продукции второго завода этот про­цент равен 63. Определим вероятность того, что случайно взятая со склада лампочка окажется удовлетворяющей условиям стандарта.

Заметим, что общее число имеющихся стандартных лампочек состоит из 400 • 0,75 • 0,83 = 249 лампочек, изготовленных первым заводом, и 63 лампочек, изготовленных вторым заводом, то есть равно 312. Так как выбор любой лампочки следует считать равновоз­можным, то мы имеем 312 благоприятствующих случаев из 400, так что

Теория вероятностей решение задач, заданий  и примеры

где событие В состоит в том, что выбранная нами лампочка стандартна.

При этом подсчете не делалось никаких предположений о том, к продукции какого завода принадлежит выбранная нами лампочка. Если же какие-либо предположения такого рода сделать, то очевидно, что интересующая нас вероятность может измениться. Так, например, если известно, что выбранная лампочка изготовлена на первом заводе (событие А), то вероятность того, что она стандартна, будет уже не 0,78, а 0,83.

Такого рода вероятность, то есть вероятность события В при условии, что имеет место событие А , называют условной вероятностью события В при условии наступления события А и обозначают Теория вероятностей решение задач, заданий  и примеры

Если мы в предыдущем примере обозначим через А событие, состоящее в том, что выбранная лампочка изготовлена на первом заводе, то мы можем написать Теория вероятностей решение задач, заданий  и примеры

Теперь мы можем сформулировать важную теорему, относящую­ся к подсчету вероятности совмещения событий.

Теорема умножения:

Вероятность совмещения событий А и В равна произведению вероятности одного из событий на условную вероятность другого в предположении, что первое имело место:

Теория вероятностей решение задач, заданий  и примеры

При этом под совмещением событий А и В понимается наступление каждого из них, то есть наступление как события А , так и события В .

Доказательство:

Рассмотрим полную группу из п рав­новозможных попарно несовместных событий Теория вероятностей решение задач, заданий  и примеры каж­дое из которых может быть благоприятствующим или неблагоприятствующим как для события А , так и для события В.

Разобьем все эти события на четыре различные группы следующим образом. К первой группе отнесем те из событий Теория вероятностей решение задач, заданий  и примеры которые благоприятствуют и событию А , и событию В; ко второй и третьей группам отнесем такие события Теория вероятностей решение задач, заданий  и примеры которые благоприятст­вуют одному из двух интересующих нас событий и не благоприятст­вуют другому, например ко второй группе — те, которые благо­приятствуют А , но не благоприятствуют В, а к третьей — те, кото­рые благоприятствуют В , но не благоприятствуют А; наконец, к четвертой группе отнесем те из событий Теория вероятностей решение задач, заданий  и примеры которые не благоприят­ствуют ни A, ни В.

Так как нумерация событий Теория вероятностей решение задач, заданий  и примеры не играет роли, то можно предположить, что это разбиение на четыре группы выглядит так: I группа: Теория вероятностей решение задач, заданий  и примеры

II группа: Теория вероятностей решение задач, заданий  и примеры

Теория вероятностей решение задач, заданий  и примеры

III группа: Теория вероятностей решение задач, заданий  и примеры

IV группа: Теория вероятностей решение задач, заданий  и примеры

Таким образом, среди n равновозможных и попарно несовместных событий Теория вероятностей решение задач, заданий  и примеры имеется k событий, благоприятствую­щих и событию A, и событию В, l событий, благоприятствующих событию A, но не благоприятствующих событию В, m событий, бла­гоприятствующих В, но не благоприятствующих A, и, наконец, n — (k + I + m) событий, не благоприятствующих ни A, ни В.

Заметим, между прочим, что какая-либо из рассмотренных нами четырех групп (и даже не одна) может не содержать ни одного события. В этом случае соответствующее число, означающее количество событий в такой группе, будет равно нулю.

Произведенная нами разбивка на группы позволяет сразу на­ писать

Теория вероятностей решение задач, заданий  и примеры

ибо совмещению событий А и В благоприятствуют события первой группы и только они. Общее число событий, благоприятствующих А, равно общему числу событий в первой и второй группах, а благо­приятствующих В — общему числу событий в первой и третьей группах.

Подсчитаем теперь вероятность Теория вероятностей решение задач, заданий  и примеры то есть вероятность события В при условии, что событие А имело место. Теперь события, вхо­дящие в третью и четвертую группы, отпадают, так как их появле­ние противоречило бы наступлению события A, и число возможных случаев оказывается равным уже не n, а k + l. Из них событию В благоприятствуют лишь события первой группы, так что мы получаем:

Теория вероятностей решение задач, заданий  и примеры

Для доказательства теоремы достаточно теперь написать очевид­ное тождество:

Теория вероятностей решение задач, заданий  и примеры

и заменить в нем все три дроби вычисленными выше вероятностями. Мы придем к утверждавшемуся в теореме равенству:

Теория вероятностей решение задач, заданий  и примеры

Ясно, что написанное нами выше тождество имеет смысл лишь при Теория вероятностей решение задач, заданий  и примеры что справедливо всегда, если только А не есть невоз­можное событие.

Так как события А и В равноправны, то, поменяв их местами, получим другую форму теоремы умножения:

Теория вероятностей решение задач, заданий  и примеры

Впрочем, это равенство можно получить тем же путем, что и предыдущее, если заметить, что Теория вероятностей решение задач, заданий  и примеры и воспользоваться тождеством Теория вероятностей решение задач, заданий  и примеры

Сравнивая правые части двух выражений для вероятности Р (А и В), получим полезное равенство:

Теория вероятностей решение задач, заданий  и примеры

Рассмотрим теперь примеры, иллюстрирующие теорему умножения.

Пример:

В продукции некоторого предприятия признаются годными (событие А) 96% изделий. К первому сорту (событие В) оказываются принадлежащими 75 изделий из каждой сотни годных. Определить вероятность того, что произвольно взятое изделие бу­дет годным и принадлежит к первому сорту.

Решение:

Искомая вероятность есть вероятность совмещения событий А и В. По условию имеем: Теория вероятностей решение задач, заданий  и примеры Поэтому теорема умножения дает

Теория вероятностей решение задач, заданий  и примеры

Пример:

Вероятность попадания в цель при отдельном выстреле (событие А) равна 0,2. Какова вероятность поразить цель, если 2% взрывателей дают отказы (т. е. в 2% случаев выстре­ла не произойдет)?

Решение:

Пусть событие В состоит в том, что выстрел произойдет, а Теория вероятностей решение задач, заданий  и примеры означает противоположное событие. Тогда по условию Теория вероятностей решение задач, заданий  и примеры и согласно следствию из теоремы сложения Р(В)=1- Р(В) = 0,98. Далее, по условию Теория вероятностей решение задач, заданий  и примеры

Поражение цели означает совмещение событий А а В (выстрел произойдет и даст попадание), поэтому по теореме умножения

Теория вероятностей решение задач, заданий  и примеры

Важный частный случай теоремы умножения можно получить, если воспользоваться понятием независимости событий.

Два события называются независимыми, если вероятность одного из них не изменяется в результате того, наступило или не наступи­ло другое.

Примерами независимых событий являются выпадение различного числа очков при повторном бросании игральной кости или той или иной стороны монет при повторном бросании монеты, так как очевидно, что вероятность выпадения герба при втором бросании равна Теория вероятностей решение задач, заданий  и примеры независимо от того, выпал или не выпал герб в первом.

Аналогично, вероятность вынуть во второй раз белый шар из урны с белыми и черными шарами, если вынутый первым шар пред­варительно возвращен, не зависит от того, белый или черный шар был вынут в первый раз. Поэтому результаты первого и второго вынимания независимы между собой. Наоборот, если шар, вынутый первым, не возвращается в урну, то результат второго вынимания зависит от первого, ибо состав шаров, находящихся в урне после первого вынимания, меняется в зависимости от его исхода. Здесь мы имеем пример зависимых событий.

Пользуясь обозначениями, принятыми для условных вероятно­стей, можно записать условие независимости событий А в В в виде

Теория вероятностей решение задач, заданий  и примеры

или

Теория вероятностей решение задач, заданий  и примеры

Воспользовавшись этими равенствами, мы можем привести теорему умножения для независимых событий к следующей форме.

Если события А и В независимы, то вероятность их совмещения равна произведению вероятностей этих событий:

Теория вероятностей решение задач, заданий  и примеры

Действительно, достаточно в первоначальном выражении теоремы умножения положить Теория вероятностей решение задач, заданий  и примеры что вытекает из независи­мости событий, и мы получим требуемое равенство.

Рассмотрим теперь несколько событий: А, В, …, L. Будем называть их независимыми в совокупности, если вероятность появле­ния любого из них не зависит от того, произошли ли какие-либо другие рассматриваемые события или нет.

В случае событий, независимых в совокупности, теорема умножения может быть распространена на любое конечное число их, благодаря чему ее можно сформулировать так:

Вероятность совмещения событий A, В , …, L, независимых в совокупности, равна произведению вероятностей этих событий:

Теория вероятностей решение задач, заданий  и примеры

Пример:

Рабочий обслуживает три автоматических станка, к каждому из которых нужно подойти для устранения неисправно­сти, если станок остановится. Вероятность того, что первый станок не остановится в течение часа, равна 0,9. Та же вероятность для вто­рого станка равна 0,8 и для третьего — 0,7. Определить вероят­ность того, что в течение часа рабочему не потребуется подойти ни к одному из обслуживаемых им станков.

Решение:

Если считать станки работающими независимо друг от друга, то в силу теоремы умножения искомая вероятность совмещения трех событий равна произведению

Теория вероятностей решение задач, заданий  и примеры

Пример:

Вероятность сбить самолет винтовочным выстрелом р = 0,004. Какова вероятность уничтожения неприятельского самолета при одновременной стрельбе из 250 винтовок?

Решение:

Вероятность того, что при одиночном выстреле самолет не будет сбит, по теореме сложения равна 1 — р = 0,996. Тогда можно подсчитать с помощью теоремы умножения вероятность того, что самолет н е будет сбит при 250 выстре­лах, как вероятность совмещения событий. Она равна Теория вероятностей решение задач, заданий  и примеры После этого мы можем снова воспользоваться теоремой сложения и найти вероятность того, что самолет будет сбит, как вероят­ность противоположного события

Теория вероятностей решение задач, заданий  и примеры

Отсюда видно, что, хотя вероятность сбить самолет одиночным винтовочным выстрелом ничтожно мала, тем не менее при стрельбе из 250 винтовок вероятность сбить самолет оказывается уже весьма ощутимой. Она существенно возрастает, если число винтовок уве­личить. Так, при стрельбе из 500 винтовок вероятность сбить само­лет, как легко подсчитать, равна Теория вероятностей решение задач, заданий  и примеры а при стрель­бе из 1000 винтовок — даже Теория вероятностей решение задач, заданий  и примеры

Доказанная выше теорема умножения позволяет несколько расширить теорему сложения, распространив ее на случай совмести­мых событий. Ясно, что если события А и В совместимы, то вероят­ность наступления хотя бы одного из них не равна сумме их вероят­ностей. Например, если событие А означает выпадение четного числа очков при бросании игральной кости, а событие В — вы­падение числа очков, кратного трем, то событию (А или В) благо­приятствует выпадение 2, 3, 4 и б очков, то есть

Теория вероятностей решение задач, заданий  и примеры

С другой стороны, Теория вероятностей решение задач, заданий  и примеры и Теория вероятностей решение задач, заданий  и примеры то есть Теория вероятностей решение задач, заданий  и примерыТеория вероятностей решение задач, заданий  и примеры Таким образом, в этом случае

Теория вероятностей решение задач, заданий  и примеры

Отсюда видно, что в случае совместимых событий теорема сложения вероятностей должна быть изменена. Как мы сейчас уви­дим, ее можно сформулировать таким образом, чтобы она была справедлива и для совместимых, и для несовместных событий, так что ранее рассмотренная теорема сложения окажется частным слу­чаем новой.

Расширенная теорема сложения:

Пусть А и В — произволь­ные события. Вероятность того, что осуществится хотя бы одно из этих двух событий, равна сумме их вероятностей без вероятнос­ти их совмещения:

Теория вероятностей решение задач, заданий  и примеры

Доказательство:

Пусть Теория вероятностей решение задач, заданий  и примеры — полная группа n попарно несовместных событий. Если Теория вероятностей решение задач, заданий  и примеры, то событию А благоприятствует Теория вероятностей решение задач, заданий  и примеры из n элементарных событий. Допу­стим, что среди них есть k событий, благоприятствующих также и событию В, а Теория вероятностей решение задач, заданий  и примеры ему не благоприятствуют. Тогда сре­ди n элементарных событий имеется ровно k событий, благоприятствующих и A и В. Поэтому если Теория вероятностей решение задач, заданий  и примеры, то среди Теория вероятностей решение задач, заданий  и примеры событий, благоприятствующих В, имеется k событий, благоприятствующих A, и Теория вероятностей решение задач, заданий  и примеры событий, которые A не благоприятст­вуют.

Все элементарные события, которые благоприятствуют событию (A или В), должны благоприятствовать либо только A, либо толь­ко В, либо и A и В. Таким образом, общее число таких событий равно

Теория вероятностей решение задач, заданий  и примеры

а вероятность

Теория вероятностей решение задач, заданий  и примеры

что и требовалось доказать.

Применяя формулу (9) к рассмотренному выше примеру выпа­дения числа очков при бросании игральной кости, получим:

Теория вероятностей решение задач, заданий  и примеры

что совпадает с результатом непосредственного подсчета.

Очевидно, что формула (1) является частным случаем (9). Действительно, если события А и В несовместны, то k = 0 и вероятность совмещения Р (А и В) = 0.

Пример:

В электрическую цепь включены последователь­но два предохранителя. Вероятность выхода из строя первого предохранителя равна 0,6, а второго 0,2. Определим вероятность пре­кращения питания в результате выхода из строя хотя бы одного из этих предохранителей.

Решение:

Так как события A и В, состоящие в выходе из строя первого и второго из предохранителей, совместимы, то искомая вероятность определится по формуле (9):

Теория вероятностей решение задач, заданий  и примеры

Примеры вычисления вероятностей

В этом параграфе мы рассмотрим ряд примеров вычисления вероятностей. При этом будет использоваться непосредственный подсчет общего числа равновозможных случаев и числа благоприятствующих случаев на основе комбинаторных задач, а также теоре­мы сложения и умножения вероятностей, рассмотренные в начале настоящей главы.

Пример:

Бросаются две игральные кости (кубики). Найти вероятность того, что на обеих костях окажется: а) одинаковое число очков; б) различное число очков.

Решение:

Подсчитаем сначала общее число возможных результатов. Каждый результат бросания двух костей можно описать в виде некоторого размещения из шести элементов (шесть возмож­ностей для числа выпавших очков) по два (бросаются две кости) с повторениями (может выпасть одно и то же число очков на обеих костях). Поэтому общее число элементарных событий есть

Теория вероятностей решение задач, заданий  и примеры

Очевидно, все элементарные события следует считать равновероятными.

Число случаев, благоприятствующих появлению одинакового числа очков, равно 6. Отсюда следует, что ответом для задачи а) является вероятность

Теория вероятностей решение задач, заданий  и примеры

Событие, указанное в задаче б), является противоположным первоначальному, и его вероятность Теория вероятностей решение задач, заданий  и примеры удовлетворяет усло­вию

Теория вероятностей решение задач, заданий  и примеры

откуда

Теория вероятностей решение задач, заданий  и примеры

Пример:

В городе имеется 10 ООО велосипедов, занумеро­ванных различными номерами от 0000 до 9999. Какова вероятность того, что номер первого встречного велосипеда будет содер­жать хотя бы одну цифру 8?

Решение:

Найдем сначала вероятность того, что ни одна цифра случайно встреченного номера не будет восьмеркой.

Для первой цифры вероятность не быть восьмеркой равна 0,9, так как всех равновероятных возможностей — различных цифр — десять, а отличных от восьмерки — девять. Значения цифр в раз­личных разрядах независимы. Тогда вероятность того, что все четы­ре цифры отличны от 8, можно определить как вероятность совме­щения событий. Она равна Теория вероятностей решение задач, заданий  и примеры

Искомая вероятность есть вероятность противоположного собы­тия, а потому равна 1 — 0,6561 = 0,3439.

Пример:

Абонент, забывший одну цифру нужного ему номера телефона, набирает эту цифру наудачу. Какова вероят­ность, что ему придется звонить не более двух раз?

Решение:

Представим для удобства рассуждений, что або­нент всегда звонит дважды, независимо от результата первой попытки. Общее число равновозможных случаев представляет здесь число размещений из 10 цифр по две без повторений, по­скольку два раза звонить по одному телефону не имеет смысла. Следовательно, это будет Теория вероятностей решение задач, заданий  и примеры Благоприятствующими будут те случаи, когда нужная цифра встретится на первом или втором месте в комбинации с любой другой. Ясно, что таких случаев 9 + 9=18. Искомая вероятность равна, следовательно, р = 0,2.

Пример:

В отделении 12 солдат. В наряд назначаются два человека наугад. Какова вероятность попасть в наряд для каждого данного солдата?

Решение:

Общее число различных парных нарядов в этом случае мы уже подсчитывали в примере 4 из § 1 предыдущей главы. Оно разно Теория вероятностей решение задач, заданий  и примеры Число парных нарядов, не содержащих данного солдата, по тем же соображениям равно Теория вероятностей решение задач, заданий  и примерыПоэтому вероятность не попасть в наряд равна Теория вероятностей решение задач, заданий  и примеры, а искомая вероятность попасть в наряд Теория вероятностей решение задач, заданий  и примеры

Пример:

В некоторой партии изделий число бракованных составляет 4%. Из числа годных изделий 75% являются первосортными. Какова вероятность того, что случайно выбранное изделие будет первосортным?

Решение:

Пусть событие А означает, что изделие является годным, а событие В — что изделие относится к первому сорту. Тогда по условию

Теория вероятностей решение задач, заданий  и примеры

Искомая вероятность есть вероятность совмещения событий и по теореме умножения равна:

Теория вероятностей решение задач, заданий  и примеры

Пример:

В некоторой лотерее имеется всего n билетов, из которых m являются выигрышными. Определить вероятность хотя бы одного выигрыша для лица, обладающего k билетами.

Решение:

Общее число равновозможных случаев выбора k билетов из имеющихся n равно числу сочетаний Теория вероятностей решение задач, заданий  и примеры Так как не­выигрышных билетов имеется n — m, то число элементарных событий, благоприятствующих событию «не выиграть ни на один би­ лет», равно Теория вероятностей решение задач, заданий  и примеры Следовательно, вероятность не выиграть ни на один билет равна Теория вероятностей решение задач, заданий  и примерыТребуемое событие выиграть хотя бы на один билет является противоположным, и его вероятность равна

Теория вероятностей решение задач, заданий  и примеры

Пример:

Из карточной колоды с 36 картами извлекается наугад одна карта. Какова вероятность извлечь картинку (короля, даму или валета) любой масти или карту пиковой масти?

Решение:

Так как в колоде всего 12 картинок, то вероятность извлечь картинку равна Теория вероятностей решение задач, заданий  и примеры. Вероятность извлечь карту пиковой масти равна Теория вероятностей решение задач, заданий  и примеры. Остается воспользоваться теоремой сложения.

Однако необходимо учесть, что рассматриваемые события совместимы, так что следует воспользоваться расширенной теоремой сложения (см. формулу (9) из § 2), которая дает:

Теория вероятностей решение задач, заданий  и примеры

Это и есть искомая вероятность.

Пример:

Шесть пассажиров садятся на остановке в трам­вайный поезд, состоящий из трех трамвайных вагонов. Какова вероятность того, что: а) все пассажиры сядут в один вагон; б) хотя бы в один вагон не сядет ни один пассажир; в) в каждый вагон сядут по два пассажира?

Решение:

Число различных способов, которыми пассажиры могут разместиться в вагонах, подсчитывалось в примере 6 из § 5 предыдущей главы. Так как нас заведомо интересует лишь число пассажиров в каждом вагоне, то это число различных способов есть число сочетаний с повторениями Теория вероятностей решение задач, заданий  и примеры Число благоприятствующих событий подсчитывается непосредст­венно. а) Благоприятствующих событий три — все пассажиры сели в первый вагон, или во второй, или в третий. Искомая вероятностьТеория вероятностей решение задач, заданий  и примеры

б) Благоприятствующих событий 6: в трех случаях свободным остается один вагон и в трех случаях — два вагона. Искомая вероятность Теория вероятностей решение задач, заданий  и примеры

в) Благоприятствующих событий одно. Вероятность равна Теория вероятностей решение задач, заданий  и примерыТеория вероятностей решение задач, заданий  и примеры

Полная вероятность. Формула Бейеса

При вычислении вероятностей сложных событий часто приходит­ся одновременно применять теоремы сложения и умножения. Рассмотрим следующий пример.

Пример:

Имеется три одинаковых на вид урны с различным составом белых и черных шаров. Пусть в первой урне находит­ся Теория вероятностей решение задач, заданий  и примеры белых и Теория вероятностей решение задач, заданий  и примеры черных шаров, во второй урне соответственно Теория вероятностей решение задач, заданий  и примеры белых и Теория вероятностей решение задач, заданий  и примеры черных и, наконец, в третьей — Теория вероятностей решение задач, заданий  и примеры белых и Теория вероятностей решение задач, заданий  и примеры черных шара. Выбирается наугад одна из урн, и из нее вынимается один шар. Требуется определить вероятность того, что вынутый шар ока­жется белым.

Решение:

Сделаем сначала предположение, что шар вынут из первой урны. Можно сказать, что это предположение означает наступление события Теория вероятностей решение задач, заданий  и примеры или осуществление гипоте­зы Теория вероятностей решение задач, заданий  и примеры Так как выбор любой урны равновероятен, то вероятность этой гипотезы равна Теория вероятностей решение задач, заданий  и примеры. Из предположения о составе шаров следует, что вероятность вынуть белый шар из первой урны (событие Теория вероятностей решение задач, заданий  и примеры) равна

Теория вероятностей решение задач, заданий  и примеры

Рассмотрим сложное событие, состоящее в том, что выбрана пер­вая урна и вынутый из нее шар оказался белым. Тогда вероятность такого события в силу теоремы умножения будет равна:

Теория вероятностей решение задач, заданий  и примеры

(см. формулу (4) предыдущего параграфа). Точно так же вероятность вынуть белый шар из второй урны есть вероятность сложного

события, состоящего в совмещении события Теория вероятностей решение задач, заданий  и примеры (выбрана вторая урна) и события Теория вероятностей решение задач, заданий  и примеры (из второй урны вынут белый шар), в результа­те чего эта вероятность равна

Теория вероятностей решение задач, заданий  и примеры

а для третьей урны

Теория вероятностей решение задач, заданий  и примеры

Пусть теперь событие А означает извлечение белого шара независимо от того, из какой именно урны он был вынут. Тогда, учиты­вая, что события Теория вероятностей решение задач, заданий  и примеры являются несовместными, ибо выби­рается лишь одна урна, мы можем воспользоваться для нахожде­ния вероятности события теоремой сложения, которая дает

Теория вероятностей решение задач, заданий  и примеры

Сформулируем теперь общую задачу. Пусть события Теория вероятностей решение задач, заданий  и примерыТеория вероятностей решение задач, заданий  и примеры образуют полную группу событий и при наступлении каждого из них, например Теория вероятностей решение задач, заданий  и примеры событие А может наступить с некоторой ус­ловной вероятностью Теория вероятностей решение задач, заданий  и примерыКакова вероятность наступления со­бытия A?

Воспользовавшись, как и в примере, с которого мы начали, тео­ремой умножения, найдем, что вероятность наступления A при условии наступления равна

Теория вероятностей решение задач, заданий  и примеры

Аналогично

Теория вероятностей решение задач, заданий  и примеры

Теперь для нахождения вероятности события A можно воспользоваться теоремой сложения, так как события Теория вероятностей решение задач, заданий  и примеры несов­местны. Складывая все равенства (1) и (2), приходим к формуле:

Теория вероятностей решение задач, заданий  и примеры

или, короче,

Теория вероятностей решение задач, заданий  и примеры

Формула (3) называется формулой полной вероятности.

События Теория вероятностей решение задач, заданий  и примеры обычно называют в таких случаях гипотезами . В рассмотренном выше примере 1 имелось три гипоте­зы (n = 3), которые были равновероятны между собой:

Теория вероятностей решение задач, заданий  и примеры

Пример:

При разрыве снаряда образуются крупные, средние и мелкие осколки, причем число крупных осколков составляет 0,1 их общего числа, а число средних и мелких — соответственно 0,3 и 0,6 общего числа осколков. При попадании в танк крупный осколок пробивает броню с вероятностью 0,9, средний — с вероятностью 0,3 и мелкий — с вероятностью 0,1. Какова вероятность того, что попавший в броню осколок пробьет ее?

Решение:

В нашем примере имеется три гипотезы, вероятности которых Теория вероятностей решение задач, заданий  и примеры Поль­зуясь формулой полной вероятности (3), находим:

Теория вероятностей решение задач, заданий  и примеры

Используя формулу полной вероятности, можно получить еще одну важную формулу, которая называется формулой Бейеса или формулой вероятностей гипотез.

Пусть мы имеем некоторую полную группу событий — гипотез Теория вероятностей решение задач, заданий  и примеры вероятность каждой из которых Теория вероятностей решение задач, заданий  и примеры до производства опыта имеет определенное значение. Пред­положим, что в результате опыта наступило некоторое событие А. Появление этого нового сведения — наступление события А — может повлечь за собой изменение первоначальных вероятностей ги­потез.

Поясним сказанное примером. Пусть урна содержит три шара белого и черного цвета, однако распределение числа шаров по цве­там неизвестно. До производства опыта о содержимом урны можно сделать четыре гипотезы:

1) 3 белых и 0 черных Теория вероятностей решение задач, заданий  и примеры

2) 2 белых и 1 черный Теория вероятностей решение задач, заданий  и примеры

3) 1 белый и 2 черных Теория вероятностей решение задач, заданий  и примеры

4) 0 белых и 3 черных Теория вероятностей решение задач, заданий  и примеры

которые мы будем считать равновероятными: Теория вероятностей решение задач, заданий  и примерыТеория вероятностей решение задач, заданий  и примерыДопустим, что в результате опыта был вынут белый шар (событие А). В таком случае вероятность гипотезы Теория вероятностей решение задач, заданий  и примеры делается равной нулю. Вероятности остальных трех гипотез также изменятся, причем их уже нельзя будет считать равновероятными; вероятность гипотезы Теория вероятностей решение задач, заданий  и примеры например, больше, чем вероят­ность гипотезы Теория вероятностей решение задач, заданий  и примеры

Поставим вопрос в общем виде: выяснить, каковы будут вероятности гипотез Теория вероятностей решение задач, заданий  и примеры после опыта в предположении, что в результате опыта наступило событие А.

Обозначим вероятности гипотез Теория вероятностей решение задач, заданий  и примеры до производства опыта соответственно через Теория вероятностей решение задач, заданий  и примеры

Теория вероятностей решение задач, заданий  и примеры

Вероятности тех же гипотез после опыта, в результате которого наступило событие A, обозначим через Теория вероятностей решение задач, заданий  и примеры

Теория вероятностей решение задач, заданий  и примеры

причем снова

Теория вероятностей решение задач, заданий  и примеры

так как события Теория вероятностей решение задач, заданий  и примеры по-прежнему несовместны и един­ственно возможны. Обозначим условную вероятность Теория вероятностей решение задач, заданий  и примеры через Теория вероятностей решение задач, заданий  и примеры и полную вероятность события А через Р .

Пользуясь равенством (6) предыдущего параграфа, которое сле­дует из теоремы умножения, напишем:

Теория вероятностей решение задач, заданий  и примеры

или с введенными обозначениями

Теория вероятностей решение задач, заданий  и примеры

Отсюда

Теория вероятностей решение задач, заданий  и примеры

Подставляя сюда выражение для полной вероятности Р из формулы (3), получим:

Теория вероятностей решение задач, заданий  и примеры

Нетрудно проверить, что сумма всех вероятностей Теория вероятностей решение задач, заданий  и примеры действитель­но равна единице.

Формула (4), дающая выражение вероятности гипотезы Теория вероятностей решение задач, заданий  и примеры пос­ле опыта, и есть нужная нам формула Бейеса.

Вернемся снова к нашему примеру. В соответствии с приняты­ ми обозначениями имеем:

Теория вероятностей решение задач, заданий  и примеры

Далее находим:

Теория вероятностей решение задач, заданий  и примеры

Окончательно получим:

Теория вероятностей решение задач, заданий  и примеры

Аналогично:

Теория вероятностей решение задач, заданий  и примеры

Повторение испытаний

Правила сложения и умножения вероятностей дают возмож­ность определять вероятности достаточно сложных комбинаций событий. Одной из наиболее простых и вместе с тем весьма распро­страненных ситуаций, с которой мы сейчас познакомимся, является схема повторения независимых испытаний.

Пусть при некотором испытании событие А может наступить или не наступить. Обозначим вероятность наступления события А через Р (А) = р и вероятность его ненаступления — через Теория вероятностей решение задач, заданий  и примеры

Рассмотрим возможные исходы двух последовательных независимых испытаний. Они описываются в т а б л. 1, в которой приведе­ны также вероятности различных исходов. Теперь нетрудно под­считать, что вероятность двукратного появления события А равна

Теория вероятностей решение задач, заданий  и примеры

Теория вероятностей решение задач, заданий  и примеры вероятность его однократного появления (безразлично, при каком испытании, то есть вероятность того, что при двух испытаниях один раз наступит А и один раз Теория вероятностей решение задач, заданий  и примерыравна 2pq, а вероятность то­го, что А не наступит ни разу, равна Теория вероятностей решение задач, заданий  и примеры Очевидно, что эти результаты единственно возможны, причем

Теория вероятностей решение задач, заданий  и примеры

Приведенное рассуждение без труда переносится на случай большего числа испытаний. Например, при трех испытаниях вероятность наступления события A три раза подряд равна Теория вероятностей решение задач, заданий  и примеры как вероятность совмещения событий. Чтобы найти вероятность наступления события А два раза, безразлично в каком порядке, заметим, что это возможно при следующих трех исходах: Теория вероятностей решение задач, заданий  и примеры вероятность каждого из которых Теория вероятностей решение задач, заданий  и примеры так что вероятность двукратного наступления события A при трех испытаниях Теория вероятностей решение задач, заданий  и примеры Аналогич­но подсчитывается вероятность однократного наступления Теория вероятностей решение задач, заданий  и примеры и вероятность того, что событие A не наступит ни разу, которая равна Теория вероятностей решение задач, заданий  и примеры Как и выше,

Теория вероятностей решение задач, заданий  и примеры

Мы можем теперь формулировать общую задачу. Производится серия из п независимых испытаний, причем вероятность наступления события А при каждом отдельном испытании равна р. Требуется определить вероятность Теория вероятностей решение задач, заданий  и примеры того, что событие наступит точно m раз.

Такая задача может встретиться, например, при подсчете вероятности т попаданий в цель при п выстрелах и во многих аналогичных случаях, которые будут рассмотрены ниже.

Заметим прежде всего, что два крайних частных случая

Теория вероятностей решение задач, заданий  и примеры

легко находятся по теореме умножения как вероятности совмещения событий.

Подсчитаем теперь вероятность того, что при n испытаниях событие A появится ровно m раз в определенном порядке, например, как в выражении

Теория вероятностей решение задач, заданий  и примеры

Ясно, что эта вероятность равна Теория вероятностей решение задач, заданий  и примеры. Очевидно, что вероятность появления события A также m раз, но в другом порядке, будет той же самой. Число всех возможных выражений из n элементов, в ко­торых m раз встречается A в различном порядке, равно числу со­четаний Теория вероятностей решение задач, заданий  и примеры

Поэтому, пользуясь теоремой сложения вероятностей, получаем:

Теория вероятностей решение задач, заданий  и примеры

Из этой формулы видно, что вероятности Теория вероятностей решение задач, заданий  и примеры представляют отдельные слагаемые в разложении бинома:

Теория вероятностей решение задач, заданий  и примеры

Поэтому формулу (1) называют биномиальной.

Итак, сформулированная выше задача полностью решена. Проиллюстрируем теперь полученную формулу двумя примерами.

Пример:

Бросается монета 6 раз. Какова вероятность вы­ падения герба 0,1, …, 6 раз?

Решение:

В данном случае Теория вероятностей решение задач, заданий  и примеры. Пользуясь полученной формулой, приходим к результатам:

Теория вероятностей решение задач, заданий  и примеры

Эти результаты можно изобразить графически, отложив по оси абсцисс значения m , а по оси ординат — значения Теория вероятностей решение задач, заданий  и примерыОчевидно, что наиболее вероятное число выпадений герба m = 3, однако вероятность эта невелика.

Пример:

Производится восемь выстрелов по резервуару с горючим, причем первое попадание вызывает течь, а второе — воспламенение горючего. Какова вероятность того, что резервуар будет подожжен, если вероятность попадания при отдельном выстреле равна р = 0,2?

Решение:

Найдем сначала вероятность противоположного события, т. е. вероятность того, что резервуар не будет подожжен. Это произойдет лишь тогда, когда число попаданий не превзойдет единицы. Вероятность этого равна:

Теория вероятностей решение задач, заданий  и примеры

Так как здесь р = 0,2 и q = 0,8, то

Теория вероятностей решение задач, заданий  и примеры

откуда следует, что вероятность того, что резервуар будет подожжен, равна:

Теория вероятностей решение задач, заданий  и примеры

Примеры вычисления вероятностей

Рассмотрим еще несколько примеров на вычисление вероятностей.

Пример:

При разрыве бронебойного снаряда крупные осколки составляют по весу 20% от общего веса снаряда, средние — 30% и мелкие — 50%. Вероятность того, что крупный осколок пробьет броню танка, равна 0,8. Для средних и мелких осколков та же вероятность равна соответственно 0,5 и 0,2. Подсчитаем вероятность того, что броня танка будет пробита.

Решение:

Здесь следует воспользоваться формулой полной вероятности. Приняв в качестве гипотез различные размеры оскол­ка, получим, что их вероятности равны соответственно 0,2, 0,3 и 0,5. Поэтому искомая вероятность равна

Теория вероятностей решение задач, заданий  и примеры

Пример:

В условиях предыдущего примера, если броня танка оказалась пробитой, какова вероятность того, что пробоина произошла от мелкого осколка?

Решение:

Здесь мы можем применить формулу Бейеса, которая дает:

Теория вероятностей решение задач, заданий  и примеры

Пример:

Для данного стрелка вероятность попадания в десятку равна 0,7, а в девятку — 0,3. Определить вероятность того, что этот стрелок при трех выстрелах выбьет не менее 29 очков.

Решение:

Чтобы набрать не менее 29 очков, необходимо либо три раза попасть в десятку, либо два раза в десятку и один раз в девятку. Вероятность попасть три раза подряд в десятку находится по теореме умножения как вероятность совмещения событий. Она равна:

Теория вероятностей решение задач, заданий  и примеры

Вероятность попадания два раза в десятку и один раз в девятку можно найти по биномиальной формуле:

Теория вероятностей решение задач, заданий  и примеры

Искомая вероятность находится по теореме сложения и равна:

Теория вероятностей решение задач, заданий  и примеры

Пример:

Что вероятнее выиграть у равносильного противника: а) три партии из четырех или пять партий из восьми; б) не менее трех партий из четырех или не менее пяти партий из восьми (считая, что ничейный исход партии исключен)?

Решение:

Указание на равносильность противника следует рассматривать как утверждение, что вероятность выигрыша партии равна Теория вероятностей решение задач, заданий  и примеры, так же как и вероятность проигрыша Теория вероятностей решение задач, заданий  и примерыТеория вероятностей решение задач, заданий  и примеры . Теперь мы можем воспользоваться формулой повторения испытаний. а) Вероятность выиграть три партии из четырех находится по формуле:

Теория вероятностей решение задач, заданий  и примеры

Аналогично для выигрыша пяти партий из восьми получаем:

Теория вероятностей решение задач, заданий  и примеры

Отсюда видно, что вероятность выиграть три партии из четырех больше, чем вероятность выиграть пять партий из восьми, хотя на первый взгляд может показаться, что это не так. б) Выигрыш не менее трех партий из четырех означает, что долж­ны быть выиграны три либо четыре партии. По теореме сложения и формуле повторения испытаний находим:

Теория вероятностей решение задач, заданий  и примеры

Точно так же

Теория вероятностей решение задач, заданий  и примеры

Сравнивая между собой полученные вероятности, замечаем, что вероятность выиграть не менее трех партий из четырех меньше, чем вероятность выиграть не менее пяти партий из восьми.

Пример:

Из колоды, содержащей 36 карт, извлекаются одна за другой четыре карты. Какова вероятность того, что среди вынутых карт окажется не более одного туза? Рассмотреть два раз­ личных случая: а) после проверки вынутой карты она снова возвращается в колоду; б) вынутая карта в колоду не возвращается.

Решение:

а) Задача решается очень просто с помощью биномиальной формулы. Действительно, так как карта возвращается после каждого вынимания, то вероятность вынуть туз каждый раз остается одной и той же и равна Теория вероятностей решение задач, заданий  и примеры( в колоде четыре туза). Среди вынутых карт окажется не более одного туза, если число вынутых тузов будет равно либо нулю, либо единице.

Таким образом, искомая вероятность равна:

Теория вероятностей решение задач, заданий  и примеры

б) В случае, когда вынутая карта в колоду не возвращается, дело обстоит сложнее, так как вероятность вынуть туз меняется от одного вынимания к другому и зависит от результатов предыдущего вынимания.

В первом вынимании вероятность вынуть туз равна Теория вероятностей решение задач, заданий  и примеры Что касается второго вынимания, то эта вероятность будет иной. Именно, если в первом случае был вынут туз, то вероятность вынуть туз во второй раз будет равнаТеория вероятностей решение задач, заданий  и примеры Если же в первом случае был вынут не туз, то вероятность вынуть туз во второй раз равна уже Теория вероятностей решение задач, заданий  и примеры

Можно таким же способом проследить, какова будет вероятность вынуть туз в третий и четвертый раз в зависимости от исхода предыдущих выниманий. Однако это чересчур сложно и громоздко. Гораз­до проще решать этот вопрос иначе в более общем виде. Для этой цели обратимся к следующему примеру.

Пример:

Имеется N предметов, из которых М обладают некоторым признаком. Из этого множества предметов выбираются наугад (то есть выбор каждого из N предметов равновозможен) n предметов. Какова вероятность того, что среди них ровно m будут обладать этим признаком?

Решение:

Найдем, прежде всего, общее число возможных комбинаций. Ясно, что оно равно числу сочетаний Теория вероятностей решение задач, заданий  и примеры По условию, извлечение каждой из этих комбинаций следует считать равновоз­можным. Подсчитаем теперь число благоприятствующих собы­тий.

Группу из m элементов, обладающих нужным признаком, из общего числа М таких элементов можно выбрать Теория вероятностей решение задач, заданий  и примеры различными спо­собами. Далее, оставшиеся n — m элементов, нужным признаком не обладающие, могут быть выбраны Теория вероятностей решение задач, заданий  и примерыразличными спосо­бами, поскольку общее число таких элементов есть N — М. Так как любая группа элементов, обладающих нужным признаком, может комбинироваться с любой группой элементов, им не обладающих, то общее число благоприятствующих событий равно произведению Теория вероятностей решение задач, заданий  и примеры

Окончательно находим, что искомая вероятность равна:

Теория вероятностей решение задач, заданий  и примеры

Теперь мы можем возвратиться к решению задачи б) из предыдущего примера. Здесь у нас N = 36, М = 4, n = 4. Как и в а), нас интересуют случаи m = 0 (ни одного туза) и m = 1 (ровно один туз). Эти события несовместны, и по теореме сложения для искомой вероятности находим:

Теория вероятностей решение задач, заданий  и примеры

Теория вероятностей — основные формулы и примеры с вычислением

Испытания и события

Предметом теории вероятностей является изучение законов, управляющих случайными событиями (явлениями). К основным понятиям теории вероятностей относятся испытание и событие.
Под испытанием (опытом) понимают реализацию данного комплекса условий, в результате которого непременно произойдет какое-либо событие

Пример:

Брошена монета — испытание. Появление герба или цифры — события.

Пример:

Произведен выстрел по мишени — испытание. Попадание или промах — события.

Пример:

В урне имеются цветные шары. Из урны наудачу берут один шар. Извлечение шара из урны — испытание. Появление шара определенного цвета — событие.

Случайным событием называется событие, связанное с данным испытанием, которое при осуществлении этого испытания может произойти, а может и не произойти. Прилагательное «случайное» для краткости часто опускают и говорят просто «событие».

Пример:

Брошена игральная кость (кубик, на гранях которого отмечено от одного до шести очков). Выпадение четырех очков — случайное событие.

Пример:

В урне имеются белые и черные шары. Из урны наугад берут два шара. Оба шара белые — случайное событие.

Достоверным событием называется событие, которое в результате данного испытания непременно произойдет.

Пример:

Брошена игральная кость. Выпадение не более шести очков — достоверное событие.

Невозможным событием называется событие, которое заведомо не произойдет в результате данного испытания.

Пример:

Брошена игральная кость. Выпадение десяти очков — невозможное событие.

Пример:

Камень брошен вверх. Камень остается висеть в воздухе — невозможное событие.

Случайные события обозначаются большими буквами латинского алфавита A, B, С, … Например, событие А — попадание в мишени при стрельбе, событие В — появление герба при бросании монеты. Достоверное событие: будем обозначать буквой U, невозможное V.

Отметим, что всякое случайное событие является следствием очень многих причин. Например, выпадение герба или цифры при бросании монеты зависит от силы, с которой брошена монета, ее формы, сплава и многих других причин. Попадание или промах при стрельбе зависит от расстояния до мишени, веса пули (снаряда), от направления и силы ветра и других случайных причин. В связи с этим невозможно заранее предсказать, произойдет единичное событие или нет. Иначе обстоит дело при изучении многократно повторяющихся событий. Оказывается, что однородные случайные события при многократном повторении подчиняются определенным закономерностям. Изучением этих закономерностей и занимается теория вероятностей.

Виды случайных событий

Пусть произведено испытание, в результате которого возможны события Теория вероятностей События Теория вероятностей Называются несовместными, или осуществление одного из них исключает осуществление других

Пример:

В ящике имеются стандартные и нестандартные детали. Наудачу берут одну деталь. События Теория вероятностей — «появилась стандартная деталь» и Теория вероятностей — «появилась нестандартная деталь» являются несовместными событиями.

Пример:

Брошена игральная кость. Событие Теория вероятностей — «появление двух очков» и событие Теория вероятностей — «появление четного числа очков» совместны, так как появление одного из них не исключает появление другого.

События Теория вероятностей Называют равновозможными, если условия испытания обеспечивают одинаковую возможность осуществления каждого из них.

Пример:

Появление того или иного числа очков при бросании игральной кости есть события равновозможные, так как игральная кость изготовляется из однородного материалу и имеет строго симметричную форму.

Пример:

Появление герба и появление цифры при бросании симметричной монеты есть события равновозможные.

События Теория вероятностей образуют полную группу событий, если в результате одного испытания непременно произойдет хотя бы одно из них.

Пример:

В урне имеются три белых шара, перенумерованных цифрами 1, 2, 3, и пять черных шаров перенумерованных цифрами
1, 2, 3, 4, 5. Из урны наугад берут один шар. События:

Теория вероятностей — «появление шара с цифрой 1»,
Теория вероятностей — «появление шара с цифрой 2»,
Теория вероятностей — «появление шара с цифрой 3»,
Теория вероятностей — «появление шара с цифрой 4»,
Теория вероятностей — «появление шара с цифрой 5»

образуют полную группу. Важную роль играет полная группа несовместных событий, т. е. такая группа событий, что в результате
данного испытания непременно произойдет одно и притом только одно события данной группы.

Пример:

При бросании игральной кости возможны события:

Теория вероятностей — «появление одного очка»,
Теория вероятностей — «появление двух очков»,
Теория вероятностей — «появление трех очков»,
Теория вероятностей —«появление четырех очков»,
Теория вероятностей — «появление пяти очков»,
Теория вероятностей — «появление шести очков».

Эти события образуют полную группу несовместных событий.

Два случайных события называются противоположными, если одно из них происходит в той и только том случае, когда не происходит другое.

Событие, противоположное событию А обозначают через Теория вероятностей ( читаю «не А » ).

Пример:

Попадание и промах при выстреле по мишени противоположные события. Если А — попадание, то Теория вероятностей — промах.

Пример:

Появление четного числа очков при бросании игральной кости — событие, противоположное появлению нечетного числа очков.

Очевидно, что противоположные события образуют полную группу событий.

Отметим, что любое случайное событие может быть представлено в виде некоторого множества. Поясним сказанное на конкретном примере.

Пример:

При бросании игральной кости непременно произойдет одно из событий Теория вероятностей (см. пример 6). Каждое из этих событий назовем элементарным событием. Все элементарные события Теория вероятностей Теория вероятностей образуют множество элементарных событий Теория вероятностей

Очевидно, что: 1) событие В — «появление четного числа очков» может быть представлено в виде множества Теория вероятностей;
2) событие С — «появление числа очков, не большего трех», может быть представлено множеством Теория вероятностей 3) событие D
«появление числа очков, которое делится на 3», может быть
представлено множеством Теория вероятностей и т. д.

Нетрудно заметить, что множества В, С и D являются подмножествами множества элементарных событий A. Таким образом, любое случайное событие может быть представлено подмножеством множества всех элементарных событий данного испытания.

Операции над событиями

Прежде всего установим некоторые отношения между событиями. Рассмотрим события:

А — «появление трех очков при бросании игральной кости»,
В —«появление нечетного числа очков при бросании игральной кости».

Очевидно, что если произошло событие А, то непременно произошло и событие В. В этом случае говорят «А влечет за собой В» (или «В является следствием А») и записывают Теория вероятностей (или Теория вероятностей).

Если события А и В таковы, что Теория вероятностей то они называются равными (равносильными) при этом пишут Теория вероятностей

Пример:

Брошена симметричная монета. Событие A — «появление герба», событие В — «непоявление цифры». Очевидно, что Теория вероятностей и Теория вероятностей и, следовательно, Теория вероятностей.

Пример:

В урне имеются пять белых шаров, перенумерованных от 1 до 5, и семь черных шаров, перенумерованных от 6 до 12. Очевидно, что событие А — «появление шара с номером 8», влечет за собой событие В — «появление черного шара». Поэтому Теория вероятностей.

Так как события могут быть представлены в виде подмножеств множества элементарных событий, то действия над событиями выполняются аналогично действиям над множествами.

Сложение

Определение:

Сумой или объединением двух событий А и В называют событие С, состоящие в осуществлении хотя бы одного из событий А или В (безразлично, какого именно, или обоих, если это возможно).

Символически записывают так: Теория вероятностей или Теория вероятностей.

Сумма событий интерпретируется как объединение (сумма) множеств (подмножеств множества элементарных событии) — см. рис. 118.

Теория вероятностей

Суммой или объединением нескольких событий Теория вероятностей называется событие С, состоящее в осуществлении хотя бы одного из событий Теория вероятностей Символически:

Теория вероятностей

Пример:

Найти сумму событий: А — «появление одного очка при бросании игральной кости» и В — «появление двух очков при бросании игральной кости».

Суммой А + В является событие С — «появление не больше двух очков при бросании игральной кости», поэтому

Теория вероятностей

Если события А и В — несовместные, то сумма А + В является событием, состоящим в осуществлении одного из этих событий, безразлично какого (их совместное осуществление невозможно).

Непосредственно из определения суммы событий вытекают следующие свойства сложения:

Теория вероятностей (коммутативность); Теория вероятностей (ассоциативность);
Теория вероятностей

Умножение

Определение:

Произведением или пересечением двух событий А и В называется событие С, состоящее в одновременном осуществлении
А и В.

Символически произведение записывают так:

Теория вероятностей

Теоретико — множественная интерпретация произведения событий дана на рис. 119.

Теория вероятностей

Произведением или пересечением нескольких событий Теория вероятностей называется событие С, состоящее в одновременном осуществлении всех событий Теория вероятностей Символически:

Теория вероятностей

Пример:

Найти произведение событий A — «студенту попался экзаменационный билет с четным номером» и В — «студенту попался экзаменационный билет с номером, кратным пяти».

Решение:

Произведением является событие С — «студенту попался экзаменационный билет с номером, кратным десяти», поэтому AВ = С.

Если события А и В— несовместные, то AВ = V, т. е.. произведение — невозможное событие. Можно показать, что для умножения событий имеют место свойства:

1) AВ = ВA (коммутативность);
2) А () = ()С (ассоциативность);
3) А (В + С) = AВ + АС (дистрибутивность);
4) Теория вероятностей

Вероятность события

Известно, что случайное событие в результате испытания может произойти, а может и не произойти. Однако объективная возможность различных событий в одном и том же испытании может, вообще говоря, быть различной.

Рассмотрим пример. В урне 12 одинаковых, тщательно перемешанных шаров, причем 3 из них белые и 9 черные. Из урны наудачу вынимают один шар. Очевидно, что возможность появления черного шара «больше», чем возможность появления белого шара. В этом случае говорят: «вероятность появления черного шара
больше вероятности появления белого шара». Под вероятностью события понимают численную меру объективной возможности появления этого события.

Поставим своей задаче научиться находить эту численную меру объективной возможности события, т. е. находить вероятность события, причем ограничимся лишь вычислением вероятностей в классической модели.

Под классической моделью понимают такое множество элементарных событий, которое образует полную группу несовместных событий и все элементарные события равновозможны.

Например при бросании игральной кости множество элементарных событий:

Теория вероятностей — «появление одного очка»,
Теория вероятностей — «появление двух очков»,
Теория вероятностей — «появление трех очков»,
Теория вероятностей — «появление четырех очков»,
Теория вероятностей — «появление пяти очков»,
Теория вероятностей — «появление шести очков»

образует классическую модель. Вероятность каждого из этих элементарных событий Теория вероятностей считаем равной 1/6.

Рассмотрим теперь события: A — «появление четного числа очков»,
В — «появление не больше двух очков», Нетрудно заметить, что событие A произойдет, если произойдет по крайней мере одно из событий Теория вероятностей. В этом случае говорят, что событию А благоприятствуют события Теория вероятностей. Очевидно, что событию В
благоприятствуют события Теория вероятностей и Теория вероятностей.

То элементарное событие, при котором интересующее нас событие наступит, называется благоприятствующим этому событию.

При бросании игральной кости имеем 6 элементарных событий, из них 3 благоприятствуют событию А. Вероятность события А считаем равной Теория вероятностей . Аналогично вероятность события В равна Теория вероятностей Кратко это записывается так:

Теория вероятностей

Определение:

Вероятностью P(А) события А называется отношение числа m элементарных событий, благоприятствующих этому событию, к общему числу n равновозможных событий:

Теория вероятностей

Это определение носит название классического определения вероятности.

Из (1) следует, что

Теория вероятностей

т. е. вероятность достоверного события равна единице, а вероятность невозможного события равна нулю. Если Теория вероятностей

Итак, вероятность любого события A удовлетворяет неравенствам

Теория вероятностей

Рассмотрим ряд примеров непосредственного вычисления вероятностей.

Пример:

В урне 3 белых и 9 черных шаров. Из урны наугад вынимают один шар. Какова вероятность того, что вынутый шар окажется черным (событие А)?

Решение:

Имеем Теория вероятностей и поэтому

Теория вероятностей

Пример:

В урне 4 белых и 7 черных шаров. Из урны одновременно вынимают два шара. Какова вероятность того, что оба шара белые (событие А)?

Решение:

Здесь число элементарных событий

Теория вероятностей

Число случаев, благоприятствующих событию А:

Теория вероятностей

Следовательно,

Теория вероятностей

Пример:

В урне а белых и b черных шаров. Из урны наугад вынимают Теория вероятностей шаров. Найти вероятность того, что среди них будет Теория вероятностейбелых, а следовательно, Теория вероятностей черных Теория вероятностей.

Решение:

Число элементарных событий Теория вероятностей Подсчитаем число элементарных событий, благоприятствующих интересующему нас событию А — среди Теория вероятностей взятых шаров будет. Теория вероятностей белых и Теория вероятностейчерных. Очевидно, что число способов, которыми можно выбрать Теория вероятностей белых шаров из а, равно Теория вероятностей, а число способов, которыми можно к ним «добавить» Теория вероятностей черных шаров, равно Теория вероятностей. Каждая комбинация белых шаров может сочетаться с каждой комбинацией черных, поэтому Теория вероятностей Следовательно,

Теория вероятностей

Пример:

В партии из 12 деталей имеется 7 стандартных. Найти вероятность того, что среди шести взятых наугад деталей 4 стандартных.

Решение:

Нетрудно заметить сходство между этой и предыдущей задачами. Здесь в качестве «урны» фигурирует партия деталей, среди которых 7 стандартных («белые шары») и 5 нестандартных («черные шары»), а роль вынимаемых шаров играет контрольная партия из шести деталей. Поэтому искомую вероятность находим по формуле (2) для случая Теория вероятностей,Теория вероятностей

Теория вероятностей

Пример:

Десять различных книг расставляются наудачу на одной полке. Найти вероятность того, что три определенные книги окажутся поставленными рядом.

Решение:

Представим себе, что три определенные книги связаны вместе. Тогда число возможных способов расположения связки на полке равно числу перестановок из 8 элементов (связка плюс остальные 7 книг), т. е. Теория вероятностей Внутри связки 3 книги можно переставлять Теория вероятностей раз. При этом каждая комбинация внутри
связки может сочетаться с каждой из Теория вероятностей комбинацией. Поэтому число m благоприятствующих случаев равно Теория вероятностей
Число n возможных случаев, очевидно, равно Теория вероятностей. Таким образом, искомая вероятность

Теория вероятностей

Пример:

Первенство по футболу оспаривают 18 команд, среди которых 5 лидирующих. Путем жеребьевки команды распределяются на две группы по 9 команд в каждой. Какова вероятность попадания всех лидирующих команд в одну группу (событие A)? Какова вероятность попадания двух лидирующих команд в одну группу и трех — в другую (событие B)?

Решение:

Очевидно, что

Теория вероятностей

Событию А благоприятствуют столько событий, сколькими способами 5 лидирующих команд могут образовать девятки с четырьмя командами из числа остальных 13 команд. Поэтому обе группы могут быть образованы Теория вероятностейспособами. Следовательно, Теория вероятностей и

Теория вероятностей

Рассуждая аналогично, находим, что число Теория вероятностей событий, благоприятствующих событию В, равно

Теория вероятностей

Следовательно,

Теория вероятностей

Операции над вероятностями

Сложение

Теорема:

Вероятность суммы двух несовместных событий А и В равна сумме вероятностей этих событий, т. е.

Теория вероятностей

Доказательство:

Пусть n — общее число равновозможных несовместных элементарных событий испытания, в результате которого может произвести одно из событий A или В, Теория вероятностей — число элементарных событий, благоприятствующих событию A, Теория вероятностей — число элементарных событий, благоприятствующих событию В. Тогда, так как события A и В несовместны, имеем:

Теория вероятностей

что и требовалось доказать.

Из теоремы 1 вытекают некоторые следствия.

Следствие 1. Вероятность суммы нескольких несовместных событий Теория вероятностей равна сумме вероятностей этих событий, т. е.

Теория вероятностей

Это следствие получается из теоремы 1 применением метода математической индукции.

Следствие 2. Если события Теория вероятностей несовместны и образуют полную группу, то сумма их вероятностей равна единице:

Теория вероятностей

Предлагаем читателю самостоятельно доказать это следствие.

Следствие 3. Сумма вероятностей противоположных событий равна единице, т. е.

Теория вероятностей

Это непосредственно следует из формулы (3), так как противоположные события образуют полную группу»

Пример:

Военный летчик получил задание уничтожить два рядом расположенных склада боеприпасов противника. На борту самолета. осталась лишь одна бомба. Вероятность попадания в первый склад равна 0,225, во второй — 0,325. В результате-детонации любое
попадание взрывает оба склада. Какова вероятность того, что склады будут уничтожены?

Решение:

События А —«попадание в первый склад» и В — «попадание во второй склад» несовместны, поэтому вероятность попадания хотя бы в один из складов

Теория вероятностей

Пример:

На заочное отделение техникума поступают контрольные работы по математике из городов А, В и С. Вероятность поступления контрольной работы из города А равна 0,6, из города В — 0,1. Найти
вероятность того, что очередная контрольная работа поступит из города С.

Решение:

События «контрольная работа поступила из города А», «контрольная работа поступила из города В»* и «контрольная работа поступила из города С» образуют полную группу, поэтому сумма их вероятностей равна единице:

Теория вероятностей

Пример:

Вероятность того, что день будет ясным, Теория вероятностей. Найти вероятность q того, что день будет облачным.

Решение:

События «день ясный» и «день облачный» противоположные, поэтому

Теория вероятностей

Теорема:

Если события А и В совместны, то вероятность их суммы выражается формулой

Теория вероятностей

т. е. вероятность суммы двух совместных событий равна сумме вероятностей этих событий без вероятности их произведения (совместного осуществления).

Доказательство:

Пусть m — число равновозможных элементарных событий, благоприятствующих событию А, Теория вероятностей — число равновозможных элементарных событий, благоприятствующих событию В. Допустим, что среди m + Теория вероятностей элементарных событий содержится l таких, которые благоприятствуют как событию A, так и
событию В. Тогда, если n — общее число равновозможных элементарных событий,

Теория вероятностей

Так как событие A + В состоит в том, что произошло или событие A, или событие В, или и то и другое, то ему благоприятствуют Теория вероятностей элементарных событий. Поэтому

Теория вероятностей

или

Теория вероятностей

что и требовалось доказать.

Пример:

Найти вероятность того, что при бросании двух игральных костей хотя бы один раз выпадет 6 очков.

Решение:

Обозначим события:

А — «выпадение шести очков при бросании первой
игральной кости»;

В — «выпадение шести очков при бросании второй
игральной кости».

Так как события А и В совместны, то

Теория вероятностей

Но

Теория вероятностей

поэтому

Теория вероятностей

Умножение

Определение:

Два события А и В называются независимыми, если вероятность одного из них не зависит от того, произошло или не произошло другое.

Пример:

Игральная кость брошена два раза. Вероятность появления трех очков в первом испытании (событие А) не зависит от появления или непоявления трех очков во втором испытании (событие В). Аналогично, вероятность появления трех очков во втором испытании не зависит от результата первого испытания. Следовательно, события А и В — независимые.

Теорема:

Вероятность произведения двух независимых событий равна произведению вероятностей этих событий, т. е.

Теория вероятностей

Доказательство:

Пусть Теория вероятностей — число равновозможных элементарных событий испытания, в результате которого событие А может произойти или не произойти; Теория вероятностей —число элементарных событий, благоприятствующих событию Теория вероятностей, Теория вероятностей — число равновозможных элементарных событий испытания, в результате
которого может произойти событие В, Теория вероятностей — число элементарных событий, благоприятствующих событию Теория вероятностей

Нетрудно заметить, что общее число элементарных событий испытания, в результате которого может произойти (или не произойти) событие , равно Теория вероятностей Так как события А и В независимы, то число элементарных событий, благоприятствующих событию , равно Теория вероятностей. Поэтому

Теория вероятностей

что и требовалось доказать.

Если имеем n попарно независимых событий Теория вероятностей, то можно доказать, что

Теория вероятностей

Пример:

Два стрелка стреляют по одной и той же цели. Вероятность попадания в цель для первого стрелка равна 0,9, для второго — 0,8. Найти вероятность того, что оба стрелка попадут в цель.

Решение:

Обозначим события:

A — «попадание в цель первым стрелком»,
В — «попадание в цель вторым стрелком».
Так как события A и В независимы, то

Теория вероятностей

Пример:

Вероятность попадания в цель при стрельбе из первого орудия (событие A) равна Теория вероятностей, из второго орудия (событие В) равна Теория вероятностей. Найти вероятность попадания в цель хотя бы одним из орудий (событие A + В) при одновременной стрельбе из обоих орудий.

Решение:

Так как вероятность попадания в цель каждым из орудий не зависит от результата стрельбы из другого орудия, то события А и В независимы. Но

Теория вероятностей

Поэтому искомая вероятность

Теория вероятностей

Определение:

Два события А и В называются зависимыми, если вероятность одного из них зависит от того, произошло или не произошло другое.

Пример:

В ящике имеется 90 стандартных деталей и 10 нестандартных. Из ящика наудачу берут одну за другой две детали. Вероятность появления стандартной детали при первом испытании (событие А) равна Теория вероятностей Вероятность появления стандартной детали при втором испытании (событие В) зависит от
результата первого испытания: если в первом испытании событие А произошло, то Теория вероятностей если же событие В не произошло, то Теория вероятностей Следовательно, события А и В — зависимые.

Определение:

Вероятность события А, вычисленная при условии, что событие В произошло, называется условной вероятностью события А при условии В и обозначается Теория вероятностей.

Пример:

В урне а белых и b черных шаров. Из урны наудачу последовательно вынимают два шара. Найти вероятность того, что второй шар окажется черным при условии, что первый шар был черным.

Решение:

Обозначим события:

A — «первый шар черный»;
В — «второй шар черный».

Если произошло событие A, то в урне осталось всего Теория вероятностейшаров, из них b — 1 черных. Поэтому условная вероятность события В при условии, что произошло событие A, есть:

Теория вероятностей

Для зависимых событий справедлива следующая теорема, которую мы приводим без доказательства.

Теорема:

Вероятность произведения двух зависимых событий А и В равна произведению вероятности одного из этих событий на условную вероятность другого, вычисленную при условии, что первое событие произошло:

Теория вероятностей

В случае n произвольных событий Теория вероятностей справедлива формула

Теория вероятностей

где Теория вероятностей — вероятность события Теория вероятностей, вычисленная при условии, что произошли события Теория вероятностей

Пример:

В учебных мастерских техникума изготовляются детали на трех станках. Вероятность изготовления детали на первом станке равна 0,6. Вероятность появления годной детали на первом станке равна 0,8. Найти вероятность того, что годная деталь изготовлена на первом станке.

Решение:

Обозначим события:

A —«деталь изготовлена на первом станке»,
В — «деталь годная».

Имеем: Теория вероятностей По первой формуле (8) находим:

Теория вероятностей

Пример:

В ящике находится 7 деталей первого сорта, 5 второго сорта и 3 третьего сорта. Из ящика последовательно вынимают три детали. Найти вероятность того, что первая наугад вынутая деталь окажется первого сорта (событие Теория вероятностей ), вторая деталь — второго сорта (событие Теория вероятностей) и третья деталь — третьего сорта (событие Теория вероятностей).

Решение:

Очевидно, что

Теория вероятностей

По формуле (8) находим

Теория вероятностей

Формула полной вероятности

Операции над вероятностями представляют собой правила, служащие для вычисления вероятностей случайных событий через вероятности элементарных событий. При решении многих задач оказывается полезным одно следствие из этих правил, известное под
названием формулы полной вероятности. Выведем эту формулу.

Пусть событие А может произойти только с одним из событий Теория вероятностей образующих полную группу несовместных равновозможных событий. Тогда вероятность события A вычисляется по формуле полной вероятности:

Теория вероятностей

или

Теория вероятностей

В самом деле, так как событие A может произойти только с одним из событий Теория вероятностей образующих полную группу, то

Теория вероятностей

Из несовместности событий Теория вероятностей следует несовместность событий Теория вероятностей. Поэтому

Теория вероятностей

Применив к каждому слагаемому последнего равенства правило умножения вероятностей Теория вероятностей, получим формулу (1).

Пример:

В учебных мастерских на станках a, b и с изготовляют соответственно 25, 35 и 40% всех деталей. В их продукции брак составляет соответственно 15, 12 и 6%. Найти вероятность того, что наугад взятая деталь дефектна.

Решение:

Обозначим события:

Теория вероятностей — «наугад взятая деталь дефектна»,
Теория вероятностей — «деталь изготовлена на станке а»,
Теория вероятностей — «деталь изготовлена на станке 6»,
Теория вероятностей — «деталь изготовлена на станке с».

Очевидно, что события Теория вероятностейобразуют полную группу и Теория вероятностей Кроме того, числа Теория вероятностей являются условными вероятностями события А при выполнении событий (гипотез) Теория вероятностей соответственно, т. е.

Теория вероятностей

По формуле (1) находим

Теория вероятностей

Пример:

По цели произведено три последовательных выстрела. Вероятность попадания при первом выстрелеТеория вероятностей, при втором Теория вероятностей, при третьем Теория вероятностей. При одном попадании вероятность поражения цели равна 0,4, при двух — 0,7, при трех—1,0. Найти
вероятность поражения цели при трех выстрелах.

Решение:

Обозначим события:

Теория вероятностей — «поражение цели при трех выстрелах»,
Теория вероятностей — «одно попадание»,
Теория вероятностей — «два попадания»,
Теория вероятностей — «три попадания»,
Теория вероятностей — «ни одного попадания».

Согласно формуле полной вероятности

Теория вероятностей

Из условия задачи имеем:

Теория вероятностей

Вычислим вероятности событий Теория вероятностей Подчеркнем, что если Теория вероятностей — соответственно вероятности попаданий при первом, втором и третьем выстрелах, то Теория вероятностей — соответственно вероятности промахов при тех же выстрелах. Следовательно,

Теория вероятностей

Подставив полученные значения вероятностей в равенство (2), найдем

Теория вероятностей

С помощью формулы полной вероятности легко можно доказать так называемую формулу Бейеса

Теория вероятностей

В самом деле, по формулам (8) § 5 имеем: Заменив в последнем равенстве Теория вероятностей его значением из формулы (1), получаем формулу Бейеса (3).

Формула Бейеса позволяет переоценить вероятности гипотез, принятые до испытания, по результатам уже произведенного испытания.

Пример:

Имеются три одинаковые по виду урны. В первой урне 15 белых шаров, во второй— 10 белых я 5 черных, в третьей—15 черных шаров. Из выбранной наугад урны вынули белый шар. Найти вероятность того, что шар вынут из первой урны.

Решение:

Введем обозначения: событие А — «появление белого шара»,
гипотезы:

Теория вероятностей — «выбор первой урны»,
Теория вероятностей — «выбор второй урны»,
Теория вероятностей — «выбор третьей урны»;

имеем:

Теория вероятностей

Искомую вероятность Теория вероятностей находим по формуле (3):

Теория вероятностей

Пример:

Двадцать учащихся, уезжающих в студенческий строительный отряд, пришли сдавать экзамен по математике досрочно. Шестеро из них подготовились отлично, восемь хорошо, четыре удовлетворительно, а двое совсем не подготовились — понадеялись, что все помнят. В билетах 50 вопросов. Отлично
подготовившиеся учащиеся могут ответить на все 50 вопросов, хорошо — на 40, удовлетворительно — на 30 и не подготовившиеся — на 10 вопросов. Приглашенный учащийся ответил правильно на все три заданных ему вопроса. Найти вероятность того, что он отлично
подготовился к экзамену.

Решение:

Обозначим события:

Теория вероятностей — «приглашен учащийся, подготовившийся отлично»,
Теория вероятностей — «приглашен учащийся, подготовившийся хорошо»,
Теория вероятностей — «приглашен учащийся, подготовившийся удовлетворительно»,
Теория вероятностей — «приглашен учащийся, не подготовившийся к экзамену»,
Теория вероятностей — «приглашенный учащийся ответил на все три вопроса».

Имеем:

Теория вероятностей

Находим условные вероятности:

Теория вероятностей

Согласно условию задачи требуется найти Теория вероятностей Применив формулу Бейеса, получим:

Теория вероятностей

Искомая вероятность сравнительно невелика. Поэтому для уточнения оценки желательно предложить учащемуся дополнительные вопросы.

Повторение испытаний. Формула Бернулли

Пусть производится n независимых испытаний, в каждом из которых вероятность того, что произойдет событие A, равна p, а следовательно, вероятность того, что оно не произойдет, равна Теория вероятностей. Требуется найти вероятность того, что при n повторных испытаниях событие А произойдет m раз. Искомую вероятность обозначим Теория вероятностей.

Событие, состоящее в том, что событие А происходит при каждом из m первых испытаний и не происходит при остальных nm испытаниях, можно записать в виде

Теория вероятностей

Так как все n испытаний, по условию, независимы, то можно применить правило вычисления вероятности произведения независимых событий; получим

Теория вероятностей

Событие A может произойти m раз при n испытаниях, но jipn этом может получиться и другая последовательность чередований событий A и Теория вероятностей, однако каждый раз получим одну и ту же вероятностьТеория вероятностей . Очевидно, что число чередований событий A и Теория вероятностей равно числу сочетании Теория вероятностей из, n элементов по m, поэтому по теореме сложения вероятностей для несовместных событий искомая вероятность вычисляется по формуле

Теория вероятностей

Эта формула называется формулой Бернулли.

Пример:

В урне 20 шаров: 15 белых и 5 черных. Вынули подряд 5 шаров, причем каждый вынутый шар возвращается в урну и перед извлечением следующего шары в урне тщательно перемешиваются. Найти вероятность того, что из пяти вынутых шаров будет два белых.

Решение:

Вероятность появления белого шара в каждом испытании равна Теория вероятностей а вероятность непоявления белого шара равна Теория вероятностей По формуле Бернулли находим

Теория вероятностей

Пример:

Вероятность того, что расход электроэнергии в техникуме в течение одних суток не превысит установленной нормы, равна Теория вероятностей. Найти вероятность того, что в ближайшие 25 суток расход
электроэнергии в течение 20 суток не превысит нормы.

Решение:

Так как вероятность нормального расхода электроэнергии на протяжении каждых из 25 суток постоянна и равна Теория вероятностей, то вероятность перерасхода электроэнергии в каждые сутки также
постоянна и равна Теория вероятностей.

По формуле Бернулли находим искомую вероятность:

Теория вероятностей

Математическое ожидание дискретной случайной величины. Закон распределения

Определение:

Случайной величиной называется переменная X, которая в результате испытания может принять одно и только одно значение, не известное заранее и зависящее от исхода испытания.

Пример:

При бросании игральной кости случайной является величина X — число очков, которое выпадет на верхней грани. Возможными значениями величины X служат числа 1, 2, 3, 4, 5, 6.

Пример:

Число родившихся мальчиков среди ста новорожденных есть случайная величина X, возможными значениями которой являются числа 0, 1, 2, 3, ,…., 100.

Определение:

Величина X называется дискретной случайной величиной, если множество ее возможных значений представляет собой конечную или бесконечную последовательность чисел Теория вероятностей и если каждое соотношение Теория вероятностей Теория вероятностей является элементарным случайным событием и имеет определенную вероятность Теория вероятностей.

Мы будем рассматривать дискретные случайные величины лишь с конечными множествами значений.

Определение:

Законом распределения дискретной случайной величины X называется соответствие между возможными значениями Теория вероятностей и их вероятностями Теория вероятностей.

Закон распределения (как и всякую функцию) можно задать таблично, аналитически и графически. Если случайная величина X может принимать лишь конечное число различных значений Теория вероятностей, то элементарные события Теория вероятностей образуют полную группу и поэтому сумма их вероятностей равна единице, т. е.

Теория вероятностей

Закон распределения такой величины может быть представлен в виде таблицы:

Теория вероятностей

Вот, например, как выглядит таблица распределения вероятностей дискретной случайной величины X — числа очков, выпадающего при бросании правильной игральной кости:

Теория вероятностей

Математическое ожидание

Определение:

Математическим ожиданием M(X) дискретной случайной величины X называется сумма произведений всех ее возможных значений Теория вероятностей на их вероятности Теория вероятностей

Теория вероятностей

Пример:

Найти математическое ожидание случайной величины X зная закон ее распределения;

Теория вероятностей

Решение:

По формуле (1) находим

Теория вероятностей

Пусть при проведений n независимых испытаний дискретная случайная величина X может принимать Теория вероятностей раз значение Теория вероятностейраз значение Теория вероятностей раз значение Теория вероятностей. Тогда сумма всех значений величины X равна

Теория вероятностей

Найдем среднее арифметическое Теория вероятностей значений, принимаемых величиной X:

Теория вероятностей

поэтому

Теория вероятностей

Таким образом, Теория вероятностей, т. е. математическое ожидание дискретной случайной величины X равно среднему арифметическому полученных значений этой величины.

Математическое ожидание обладает следующими свойствами.

1) Математическое ожидание постоянной величины С равно самой постоянной:

Теория вероятностей

2) Математическое ожидание суммы случайных
величин равно сумме математических ожиданий
слагаемых
:

Теория вероятностей

3) Математическое ожидание произведения независимых случайных величин равно произведению математических ожиданий этих величин:

Теория вероятностей

4) Постоянный множитель можно выносить за знак
математического ожидания
:

Теория вероятностей

Докажем, например, второе свойство для случая случайных величин X и Y, заданных следующими законами распределения:

Теория вероятностей

Составим возможные значения величины X+Y, для чего к каждому возможному значению величины X прибавим каждое возможное значение величины Y:

Теория вероятностей

Обозначим вероятности этих значений соответственно через Теория вероятностей Согласно определению математического ожидания имеем

Теория вероятностей

или

Теория вероятностей

Так как событие, состоящее в том, что X принимает значение Теория вероятностей(вероятность этого события равна Теория вероятностей) влечет за собой событие, состоящее в том, что X + Y примет значение Теория вероятностей или значение Теория вероятностей (вероятность этого события по теореме сложения равна Теория вероятностей) и обратно, то Теория вероятностей.

Рассуждая аналогично, найдем

Теория вероятностей

Таким образом,

Теория вероятностей

Но Теория вероятностей Поэтому

Теория вероятностей

Доказательство остальных свойств аналогично.

Пример:

Найти математическое ожидание случайных величин
X и Y, зная законы их распределения:

Теория вероятностей

Решение:

По формуле (1) имеем:

Теория вероятностей

Мы получили любопытный результат: законы распределения величин X и Y разные, а их математические ожидания одинаковы.

Из рис. 120 видно, что значения величины Y сосредоточены около математического ожидания M(Y)

Теория вероятностей

(рис. 120, б), а значения величины X разбросаны (рассеяны) подальше от математического ожидания M(X) (рис. 120, а). Основной числовой характеристикой рассеяния возможных значений случайной величины X служит дисперсия D (X), которая определяется по формуле

Теория вероятностей

Величина Теория вероятностей называется средним квадратическим отклонением случайной величины X.

Преобразуем формулу (2) следующим образом:

Теория вероятностей

Здесь мы воспользовались свойствами математического ожидания и тем, что M(X) — величина постоянная. Таким образом,

Теория вероятностей

По формуле (3) вычислять значение дисперсии проще, чем по формуле (2). Пример 5 Дискретная случайная величина распределена по закону:

Теория вероятностей

Найти D(X).

Решение. Сначала находим

Теория вероятностей

а затем

Теория вероятностей

По формуле (3) имеем:

Теория вероятностей

Закон больших чисел

Основная особенность случайной величины состоит в том, что нельзя заранее предвидеть, какое из возможных значений она примет в результате испытания. Однако при достаточно большом числе испытаний суммарное поведение случайных величин почти утрачивает случайный характер и становится закономерным. Весьма важным при этом является знание условий возникновения закономерностей случайной величины. Эти условия составляют содержание ряда теорем, получивших общее название закона больших чисел. Впервые этот закон (в простейшей его форме) был сформулирован Яковом Бернулли в виде теоремы, устанавливающей
связь между вероятностью случайного события и его относительной частотой.

Относительной частотой W(A) случайного события A называют отношение числа Теория вероятностей испытаний, в результате которых событие произошло, к общему числу n проведенных испытаний:

Теория вероятностей

Оказывается, что при многократном повторении испытания относительная частота случайного события принимает значения, близкие к вероятности того, что оно произошло в результате одного испытания. Например, знаменитый статистик К. Пирсон бросил монету 24000 раз и получил при этом 12012 гербов, что дает
относительную частоту, очень близкую к вероятности, равной 1/2, появления герба в одном испытании.

Теорема Бернулли:

С вероятностью, сколь угодно близкой к единице, можно утверждать, что при достаточно большом числе независимых испытаний относительная частота случайного события как угодно мало отличается от его вероятности при отдельном испытании.

Наиболее общим законом больших чисел является Теорема П. Л. Чебышева, которую, как и теорему Бернулли, мы приводим без доказательства, пояснив лишь ее сущность.

Теорема Чебышева:

Если Теория вероятностей — независимые случайные величины, причем дисперсии их равномерно ограничены (не превышают постоянного числа С), то последовательность Теория вероятностей сходится по вероятности к нулю при Теория вероятностей , т. е.

Теория вероятностей

или

Теория вероятностей

где

Теория вероятностей

Отметим, что если все случайные величины Теория вероятностей имеют
одно и то же математическое ожидание а:

Теория вероятностей

то математическое ожидание среднего арифметического Теория вероятностей также совпадает с а:

Теория вероятностей

В этом случае соотношение (1) принимает вид (2)

Теория вероятностей

Сущность теоремы Чебышева состоит в том, что среднее арифметическое достаточно большого числа независимых случайных величин с равномерно ограниченными дисперсиями утрачивает характер случайной величины.

Основные понятия теории вероятностей

Теория вероятностей есть математическая наука, изучающая массовые закономерности в случайных явлениях независимо от их конкретной природы и дающая методы количественной оценки влияния случайных факторов на различные явления.

Случайные события

В теории вероятностей событием А называют все то, что может произойти, а может и не произойти при осуществлении некоторого комплекса условий G. Событие наступает в результате реализации различных процессов, которые называют опытами (экспериментами). Примеры событий:

Теория вероятностей — появление орла при бросании монеты;

Теория вероятностей — выпадение четного числа очков при игре в кости;

Теория вероятностей — выход из строя компьютера после пяти часов работы;

Теория вероятностей — замерзание воды при сильном морозе;

Теория вероятностей— в перечне месяцев года после января следует-апрель.

Все эти события отличаются в первую очередь тем, что возможность их появления различна. Одно событие (Теория вероятностей) происходит всегда, другое (Теория вероятностей) никогда не наступает, остальные могут произойти или не произойти в результате проведения одного опыта.

Если при реализации условий G событие А всегда происходит, то оно называется достоверным. Если же это событие при заданных условиях никогда не наступает, то его называют невозможным. Пример. При бросании одной игральной кости невозможно выпадение целого числа, большего шести. В то же время гарантировано выпадение натурального числа, меньшего семи.

Если в результате опыта при реализации определенного комплекса условий данное событие может наступить или не наступить, то оно называется случайным. Условия проведения такого опыта часто называют случайным опытом (экспериментом). Очевидно, что после бросания игральной кости число три может выпасть, но оно может и не выпасть. Через пять часов после включения компьютер может быть исправным, но может и выйти из строя. Стреляя в мишень можно попасть с первого выстрела, можно с третьего, но можно не попасть и никогда.

Элементарными событиями называют события, не разложимые на более простые. Пусть при данных условиях проводится случайный опыт, в результате которого обязательно наступает одно и только одно из возможных элементарных событий. Множество всех Теория вероятностейэлементарных событий Теория вероятностей образует пространство элементарных событий. Например, при бросании игральной кости множество элементарных событий —исходов образует пространство из шести элементарных исходов Теория вероятностей : выпало одно очко, выпало два очка и т. д.

Теория вероятностей

Наряду с элементарными рассматриваются так называемые составные или разложимые события. Событие В называется составным, если можно указать, по меньшей мере, два таких элементарных события Теория вероятностей, и Теория вероятностей что из существования каждого из них в отдельности следует существование события В. Этот факт записывается в виде: Теория вероятностей Используя введенную ранее терминологию, случайным событием А называют любое подмножество S пространства элементарных событий Теория вероятностей Содержательно это означает, что появление любого из элементарных событий, входящих в S, влечет за собой появление события А.

Например, при бросании игральной кости составное событие А = {число очков четное} можно записать так: А = {2,4,6}, подразумевая при этом, если выпадет число 2 или 4, или 6, то наступит событие А.

Таким образом, случайным событием А можно называть такой элемент подмножества всех элементарных событий F (А

Теория вероятностей про которое мы можем сказать, наступило это событие в результате опыта или нет.

Правила действий над событиями

Объединением (суммой) событий Теория вероятностей называется событие А, состоящее в наступлении хотя бы одного из этих событий. Обозначается одним из следующих способов

Теория вероятностей

Пример:

В урне 6 шаров, которые отличаются лишь номером Теория вероятностей Событие Теория вероятностей — наугад выбрать шар под номером i. Событие Теория вероятностей состоит в том, что будет выбран шар с номером 1 или 3, или 5, т. е. с нечетным номером.

Пересечением (произведением) событий Теория вероятностей называется событие В, состоящее в обязательном наступлении всех этих событий. Обозначается пересечение событий так

Теория вероятностей

Пример:

В урне 12 шаров, среди которых одна половина белых с номерами от 1 до 6, а другая — черных с такими же номерами. Пусть А — «вынуть белый шар», В — «вынуть шар с нечетным номером». Тогда событие Теория вероятностей означает « выбрать белый шар с нечетным номером».

Как следует из определения, пересечение событий не изменится, если поменять местами сомножители:

Теория вероятностей

Разностью событий А и В (обозначается А — В) называется событие, заключающееся в наступлении события А и одновременном ненаступлении события В. Для предыдущего примера событие

Теория вероятностей

означает «выбрать белый шар с четным номером».

Дополнение (противоположное к А) — это событие Теория вероятностей, состоящее в ненаступлении события А. Так, если А — «вынуть белый шар», то Теория вероятностей — «вынуть не белый шар».

Достоверным можно считать событие Теория вероятностей состоящее из всех элементарных событий, т. е. Теория вероятностей

Невозможным же событием считаем пустое событие Теория вероятностей т. е. событие, противоположное по отношению к достоверному.

Теория вероятностей

События Теория вероятностей называются несовместными, если в результате одного опыта никакие два из них не могут произойти одновременно. Это означает, что среди событий Теория вероятностей нельзя найти такую пару событий Теория вероятностей, в которой обнаружилось бы хотя бы по одному общему элементарному событию. Формально несовместность событий Теория вероятностей определяется следующим образом

Теория вероятностей

Например, при однократном бросании игральной кости выпадение четного и нечетного числа — несовместные события. Несовместными являются также промах и попадание при одном выстреле по мишени.

События Теория вероятностей называются равновозможными, если нет оснований считать, что одно событие встречается чаще, чем другое. Пример: выпадение герба или решки при бросании монеты.

Совокупность событий Теория вероятностей называется полной группой несовместных событий, если

Теория вероятностей

Примером полной группы несовместных событий является пространство элементарных событий. Другим характерным примером является пара двух противоположных событий (А, Теория вероятностей). Например, выпадение герба и цифры в результате подбрасывания монеты, работоспособность компьютера и его неисправность в данный момент времени, попадание и не попадание в мишень при одном выстреле и т. д.

События А и В называются независимыми, если появление одного из них не изменяет шансы появления другого. Напри-

мер, одновременно бросаются две игральные кости. Появление на одной из них трех очков ни коим образом не зависит от того,-какое количество очков появилось на верхней грани другой кости.

Если появление одного события влияет на появление другого, то такие события называются зависимыми. Рассмотрим пример. В урне два красных и два черных шара. Вынимается один шар, записывается его цвет, а шар откладывается в сторону, затем вынимается второй шар. Событие А — первый вынутый шар красный. Событие В — вынутый второй шар, тоже красный. Очевидно, что эти события зависимы: если первым вынули красный шар, то шанс вынуть красный шар и во втором опыте (событие А • В) будет меньше, чем, если бы первым был вынут черный шар (событие Теория вероятностей • В).

Понятие вероятности

Аксиоматический подход. Аксиоматическое определение вероятности.

Числовая функция Теория вероятностей называется вероятностью события А, если она удовлетворяет следующим аксиомам:

1.Вероятность Теория вероятностей есть неотрицательное число, заключенное между нулем и единицей: Теория вероятностей

2.Вероятность достоверного события равна единице;

3.Вероятность невозможного события равна нулю;

4.Вероятность суммы двух несовместных событий А и В равна сумме вероятностей этих событий

Теория вероятностей

Тройка Теория вероятностей где Теория вероятностей — множество элементарных событий, F — допустимое множество составных событий (множество подмножеств Теория вероятностей), Р — множество вероятностей случайных событий, называется вероятностным пространством.

Классическая вероятность

Классический подход. Классическая вероятность. Рассмотрим полную группу из n несовместных равновозможных событий. Примерами таких групп являются число очков при бросании игральных костей, число попаданий в мишень при выстрелах, проводимых в одинаковых условиях, появление шара с за-

данным номером при наличии в урне нескольких неразличимых на ощупь шаров.

Пусть среди всех n возможных исходов опыта, только m исходов, образующих m-подмножество в полной группе, влекут за собой наступление события А. Случаи, входящие в m подмножество будем называть благоприятными. Например, в урне два белых, три черных и пять красных одинаковых на ощупь шаров. Будем считать благоприятным выбор белого шара, таких случаев два. Появление же черного или красного шара — случай неблагоприятный, таких случаев восемь.

Вероятность события А вычисляется как отношение числа благоприятных событию А случаев к общему числу исходов опыта:

Теория вероятностей

Эта формула называется классической формулой для вычисления вероятностей. Схема применения классической формулы следующая:

1.Удостоверяются в том, что возможные исходы образуют полную группу несовместных равновозможных событий;

2.Выбирается интересующее нас случайное событие А;

3.Вычисляется число возможных исходов (n) и число благоприятных исходов (m);

4.Вычисляется искомая вероятность Теория вероятностей.

Априорно-частотный подход. Статистическая вероятность.

Рассмотрим пример: пусть в урне 10 одинаковых по размеру шаров, из них: два белых, три черных и пять красных. Случайный опыт заключается в том, что вынимается наугад один шар. При этом шар может оказаться белым или черным, или красным. Будем в каждом опыте вытаскивать шар, фиксировать его цвет, опускать вынутый шар обратно в урну и тщательно перемешивать там шары. В результате каждого проведенного опыта можно вытащить шар любого из трех возможных цветов (Теория вероятностей -вынут белый шар, Теория вероятностей — вынут черный шар, Теория вероятностей — красный шар).

Пусть проведено n рассматриваемых опытов, в результате которых белый шар вынимался Теория вероятностей раз, черный Теория вероятностей а красный —

Теория вероятностей раз. Частота появления каждого из возможных событий есть отношение

Теория вероятностей

В общем случае частотой появления события А в n опытах называется отношение числа появлений этого события (m) к числу проведенных опытов n

Теория вероятностей

Если проводить такой опыт значительное число раз (n — велико), то окажется, что примерно в половине случаев вынули красный шар, в двадцати процентах случаев — белый, а в тридцати — черный. По мере увеличения числа проведенных опытов уверенность в соотношении шансов возможных событий соответственно 5: 2 : 3 будет подтверждаться со все большей точностью. Поэтому статистическая вероятность вычисляется по формуле

Теория вероятностей

Введенная таким образом вероятность события носит название статистической или априорно-частотной. Она фиксирует связь между частотой события, как непосредственно измеряемой величиной, и вероятностью, являясь формальной характеристикой случайного события.

Непосредственной проверкой можно установить, что и классическая вероятность, и статистическая события удовлетворяет сформулированным выше аксиомам.

Замечание:

При использовании классического определения вероятности используют формулы комбинаторики.

Задача:

Имеется колода из 36 карт. Наугад вынимаются три карты. Найти вероятность того, что среди них окажется ровно один валет.

Решение:

В выбранных трех картах может быть три валета, два, один или ни одного валета. Всего в колоде четыре валета.

Поэтому реализовать благоприятный случай А — выбрать один из них (любой!) можно Теория вероятностей различными способами, две другие карты (не валеты) можно выбрать Теория вероятностей различными способами. Следовательно, Теория вероятностей (здесь логическое «и»). Общее же число исходов есть число сочетаний из 36 по 4, т. е. Теория вероятностей Искомая вероятность вычисляется по формуле

Теория вероятностей

Задача:

Замок имеет четырехзначный цифровой шифр. Наугад выбираются четыре цифры. Какова вероятность открыть при этом замок, если известно, что в коде все цифры различные?

Решение:

Поскольку в шифре замка важен не только набор цифр, но и их порядок, то число благоприятных событий равно

1.Общее же число возможных упорядоченных комбинаций из четырех различных цифр определяется по формуле Теория вероятностей Таким образом, искомая вероятность

Теория вероятностей

Геометрические вероятности

Пусть пространство элементарных событий является несчетным, но выполняются следующие условия:

1.Любые два элементарных события несовместны;

2.Все события являются равновозможными.

В таких опытах вероятности некоторых событий можно вычислить геометрически как отношение длин отрезков, площадей фигур, объемов соответствующих областей

Теория вероятностей

здесь Теория вероятностей — «благоприятная» площадь (длина, объем), а S — общая площадь (длина, объем).

Задача:

Поезда в метро идут с интервалом в три минуты. Чему равна вероятность того, что пассажир будет ждать поезда более двух минут?

Решение:

Считаем, что моменты появления пассажира в интервале между поездами равновероятны. Если время интервала принять за три единицы длины Теория вероятностей то время ожидания «более двух минут» равно одной единице длины Теория вероятностей(А), т. к. благоприятствующее событие (А — прождать более двух минут) занимает одну минуту в трехминутном интервале движения поездов. Следовательно, искомая вероятность равна отношению двух длин временных интервалов

Теория вероятностей

Теорема умножения вероятностей

Пусть рассматриваются два события А и В, для которых известны вероятности их появления Р(А) и Р(В). В общем случае вероятность произведения двух событий Р(АТеория вероятностейВ) равна произведению вероятности появления события А на условную вероятность Р(В|А) события В, т. е. вероятность, вычисленную при условии, что событие А имело место

Теория вероятностей

Теорему можно распространить на произведение n событий

Теория вероятностей

Так как Теория вероятностей то

Теория вероятностей

Для независимых событий, когда Теория вероятностей

Теория вероятностей

т. е. вероятность произведения двух независимых событий равна произведению вероятностей этих событий.

Критерий независимости двух событий

События А и В называются независимыми, если

Теория вероятностей

Несколько событий называются независимыми, если независимы любые два из них, и каждое событие, и все произведения

остальных. Вероятность произведения п независимых событий

Теория вероятностей

Задача:

Механизм собирается из трех одинаковых дета-лей и считается неработоспособным, если все три детали вышли из строя. В сборочном цехе осталось 15 деталей, из которых 5 нестандартных (бракованных). Какова вероятность того, что собранный из взятых наугад оставшихся деталей механизм будет неработоспособным. Решение. Обозначим искомое событие через А, а выбор первой нестандартной детали через Теория вероятностей второй — через Теория вероятностей и третьей -через Теория вероятностей. Событие А произойдет, если произойдет и событие Теория вероятностей и событие Теория вероятностей, и событие Теория вероятностей. Однако события Теория вероятностей зависимы и, если вероятность выбрать первую нестандартную деталь равна 5/15, то вероятность выбрать и вторую нестандартную деталь равна 4/14, и третью нестандартную деталь равна 3/13. Следовательно, используя формулу (4.2), имеем

Теория вероятностей

Теорема сложения вероятностей

Вероятность суммы двух совместных событий равна сумме вероятностей этих событий минус вероятность произведения этих же событий:

Теория вероятностей

Если события несовместны, то Теория вероятностей следовательно,

Теория вероятностей

Задача:

Производится два независимых выстрела в одну и ту же мишень. Вероятность попадания при первом выстреле 0,6, а во втором — 0,8. Найти вероятность попадания в мишень при двух выстрелах.

Решение:

Обозначим попадание при первом выстреле как событие Теория вероятностей при втором — как событие Теория вероятностей. Попадание в мишень предполагает хотя бы одно попадание: или только при первом выстреле, или только при втором, или, и при первом, и при втором. Следовательно, в задаче требуется определить вероятность суммы двух совместных событий Теория вероятностей И Теория вероятностей

Теория вероятностей

Поскольку события независимы, то Теория вероятностей

Поэтому

Теория вероятностей

Вероятность суммы трех событий

Теория вероятностей

Если события Теория вероятностей образуют полную группу, то сумма вероятностей событийТеория вероятностей равна 1.

Теория вероятностей

В частности, вероятность суммы противоположных событий равна единице, т. е.

Теория вероятностей

Вероятность «хотя бы одного события»

Если события Теория вероятностей независимы в совокупности, то вероятность появления хотя бы одного из событий Теория вероятностей вычисляется по формуле

Теория вероятностей

Пример:

Ремонтное ателье обслуживает пять клиентов. Вероятность вызова на обслуживание от каждого клиента равна 0,2. Какова вероятность того, что в данный момент ателье занято обслуживанием клиентов?

Решение:

Очевидно, что событие А «быть занятым обслуживанием клиентов» есть сумма событий Теория вероятностейТеория вероятностей«быть занятым обслуживанием i-го клиента». Противоположное событие «не быть занятым обслуживанием клиентов» определяется так: Теория вероятностей Следовательно, можно применить формулу (4.4).

Теория вероятностей

Формула полной вероятности

Пусть некоторое событие А может произойти с какой-то вероятностью только как следствие каждого из событий Теория вероятностей образующих полную группу несовместных событий. Такие события Теория вероятностей называются гипотезами. Заданы вероятности гипотез Р(Теория вероятностей) и условные вероятности наступления события А с каждой из гипотез Р(А|Теория вероятностей). Вероятность наступления события А дается формулой полной вероятности

Теория вероятностей

Пример:

Имеется три одинаковых урны. В первой два белых и один черный шар, во второй три белых и один черный шар, в третьей урне два белых и два черных шара. Из выбранной наугад урны выбирается один шар. Какова вероятность того, что он окажется белым?

Решение:

Гипотезой Теория вероятностей будем считать выбор i-ой урны. Все урны считаются одинаковыми, следовательно, вероятность выбрать i-ую урну есть Теория вероятностей Зная состав шаров в каждой из урн, легко определить вероятности вынуть белый шар из каждой урны: Теория вероятностей Подставляя найденные значения, имеем:

Теория вероятностей

Формула апостериорной вероятности

Рассмотрим ситуацию в некотором смысле противоположную предыдущей. Пусть событие А уже произошло, Требуется определить вероятность того, что событие произошло именно совместно с гипотезой Теория вероятностей. Ответ на подобный вопрос дает формула апостериорной вероятности или формула Байеса

Теория вероятностей

Задача:

Пусть сохраняются условия предыдущей задачи, но из урны уже вынут белый шар. Требуется определить вероятность, что шар вынут из первой урны.

Решение:

Поскольку все вероятности, необходимые для применения формулы найдены, то имеем

Теория вероятностей

Схема опытов Бернулли

Если проводится п одинаковых и независимых опытов, в каждом из которых событие А может произойти с вероятностью р, то говорят, что реализуется схема опытов Бернулли.

Пример:

Определить вероятность того, что в схеме опытов Бернулли событие А появится ровно m раз Теория вероятностей

Решение:

Пусть событие В заключается в том, что событие А в п опытах происходит подряд m раз, а затем (m — n) раз происходит событие Теория вероятностей т. е.

Теория вероятностей

Вероятность такого события В равна

Теория вероятностей

Поскольку в задаче не требуется учитывать порядок следования этих событий, то необходимо учесть все события типа В. Событий же подобных B и отличающихся лишь порядком наступления событий А и Теория вероятностей равно числу сочетаний из n по m. Следовательно, искомая формула, носящая имя Бернулли, имеет вид

Теория вероятностей

Пример:

Вероятность прибытия англоязычной группы туристическое агентство оценивает величиной 0,6. Прибывает шесть групп. Найти вероятность того, что: а) из прибывших ровно четыре — англоговорящие группы; б) прибудет не более двух таких групп.

Решение:

Ответ на вопрос а) получаем подстановкой данных в формулу (4.7)

Теория вероятностей

Чтобы получить ответ на вопрос б), необходимо вычислить сумму вероятностей прибытия двух, одной или ни одной группы.

Случайные величины

Определение случайной величины:

Величина, которая в результате опыта может принимать одно и только одно определенное значение, до опыта не известное и зависящее от причин, которые нельзя учесть заранее, называется случайной величиной. Названия случайных величин обычно обозначают заглавными буквами: X, Y, Z,…, а их возможные значения — прописными буквами: х, у, z,….

Рассмотрим примеры:

1.Проводятся выборы в представительные органы власти. Случайным событием является факт выбора (или не выбора) конкретного кандидата. При этом несомненный интерес представляет количество поданных за него голосов — случайная величина, количественно характеризующая результаты выборов.

2.Исследуется надежность нового прибора. Случайным событием является факт прохождения тестовых испытаний (удовлетворяет — не удовлетворяет оговоренному уровню требований). Случайной величиной может выступать процент приборов, прошедших испытание. В рамках испытаний может рассматриваться та или иная группа характеристик. Например, факты безотказной работы «не менее заданного срока» могут измеряться конкретным параметром — временем безотказной работы.

3.Проводится социальное обследование, в котором изучается социальный портрет жителей данного города. Случайно выбранному жителю задаются вопросы о его социальной принадлежности, профессии, возрасте, семейном положении, количестве детей, обеспеченности жильем и т. п. Случайным событием является факт опроса того или иного жителя. Количественным же результатом опроса выступает вектор значений, каждый компонент которого есть количественная мера соответствующей характеристики опрашиваемого.

Эти и многие другие примеры приводят к выводу, что случайные величины обычно имеют конкретный физический смысл и могут быть как скалярными, так и векторными.

Формальное определение: случайной величиной называется измеримая функция Теория вероятностей отображающая пространство элементарных событий Теория вероятностей на множество действительных чисел R.

Если множество Теория вероятностей конечно или счетно, то случайная величина называется дискретной. Примеры дискретной величины: количество уголовных дел, рассматриваемых данным судом за определенное время; количество клиентов; количество диалектов в данном языке; количество избирателей округа, которые примут участие в предстоящих выборах, количество телевизоров, которые необходимо проверить до выявления первого неисправного и т. п. Итак, дискретной считают такую случайную величину, возможные значения которой можно пронумеровать.

Непрерывной называют такую случайную величину, возможные значения которой непрерывно заполняют некоторый промежуток числовой оси, т. е. множество Теория вероятностей имеет мощность континуума. Например, время выхода из строя работающего компьютера, ошибка указателя скорости автомобиля, вес выбранного яблока и т. п.

Законы распределения случайных величин

Всякое соотношение, устанавливающее связь между возможными значениями случайной величины и соответствующими вероятностями, называется законом распределения этой случайной величины. Закон распределения — исчерпывающая характеристика случайной величины, и знание закона распределения позволяет решить практически любую задачу теории вероятности, связанную с данной случайной величиной. Знание закона распределения позволяет заранее (до опыта!).установить, какое значение случайной величины будет появляться чаще, а какое реже и на сколько.

Для дискретной случайной величины законом распределения выступает правило сопоставления каждому возможному значению Теория вероятностей случайной величины X вероятности его появления Теория вероятностей Последнее выражение читается так: «вероятность того, что случайная величина X примет значение Теория вероятностей«. Для дискретной случайной, при небольшом числе ее возможных значе-

ний величины закон распределения проще всего задавать в виде таблицы

Теория вероятностей

Вернемся к примеру предыдущей главы. Вероятность прибытия одной англоязычной группы туристов оценивается величиной Теория вероятностей В случае прибытия шести групп возможно прибытие различного числа таких групп (от 0 до 6). Можно предсказать вероятность наступления любого из исходов. Так вероятность прибытия двух англоязычных групп равна Теория вероятностей в два раза меньше, чем прибытие трех таких групп Теория вероятностей Следовательно, формула

Теория вероятностей

есть закон распределения случайной величины m: «число англоговорящих туристических групп при ожидаемом прибытии шести, каждая из которых прибывает из неизвестной заранее страны».

Закон распределения для непрерывной случайной величины

X есть всякое соотношение, сопоставляющее с каждой измеримой областью ее возможных значений Теория вероятностей соответствующую вероятность Теория вероятностей Например, если среднее время безотказной работы компьютеров данного типа равна s часов, то вероятность проработать без отказа не менее t часов, т. е. Теория вероятностей в некоторых случаях может быть выражена экспоненциальным законом распределения

Теория вероятностей

Функция распределения

Существует много различных форм законов распределения. Наиболее универсальной из форм является функция распределения.

Функцией распределения F(x) или интегральным законом распределения случайной величины X называется функция, за-

дающая вероятность выполнения неравенства Теория вероятностей

Теория вероятностей

Теория вероятностей

Функция распределения определена для случайных величин любого типа: дискретных и непрерывных. Определение функции распределения имеет наглядную геометрическую интерпретацию. Если рассматривать значение случайной величины X как случайную точку на числовой оси, то F(х) есть вероятность того, что случайная точка попадет левее выбранной величины х (рис. 4.1).

Свойства функции распределения непосредственно вытекают из ее определения:

1)F(х) — неотрицательная функция, значения которой заключены между 0 и 1, т. е. Теория вероятностей

Теория вероятностей

4)F(x) — неубывающая функция своего аргумента, т. е. если Теория вероятностей то

Теория вероятностей

Теория вероятностей

Пример:

Вероятность попадания при одном выстреле равна 0,4. Производится 4 выстрела. Какова вероятность того, что будет менее двух попаданий?

Решение:

Поиск ответа задачи заключается в определении значения функции распределения для значения аргумента Теория вероятностей Действительно, F(2) есть вероятность того, что произойдет менее двух попаданий, т. е. ни одного или одно. Следовательно,

Теория вероятностей

Здесь Теория вероятностей определена по формуле (4.7).

Пример:

Время безотказной работы прибора имеет функцию распределения Теория вероятностей

Вероятность того, что прибор безотказно проработает не менее 30 и не более 40 часов находится следующим образом

Теория вероятностей

Введение функции распределения позволяет дать точное определение непрерывной случайной величины: случайная величина. называется непрерывной, если ее функция распределения -непрерывная функция с кусочно-непрерывной производной.

Функция плотности вероятности

Непрерывную случайную величину удобнее описывать законом распределения, который называют функцией плотности вероятности или дифференциальным законом. Пусть X — непрерывная случайная величина, имеющая функцию распределения F(х). Если эта функция дифференцируема, то можно рассматривать ее производную

Теория вероятностей

Теория вероятностей

Функция f(х) — производная от функции распределения называется плотностью вероятности случайной величины X или функцией распределения вероятностей.

Из определения плотности вероятности (4.9) (см. свойство 10 определенного интеграла) и принимая во внимание (4.8), следует, что

Теория вероятностей

имеем

Теория вероятностей

Следовательно, если известна плотность распределения непрерывной случайной величины, то можно вычислить вероят-

ность попадания этой случайной величины в любой заданный промежуток ее возможных значений.

Например: Теория вероятностей Следовательно, из (4.10),

Теория вероятностей

Основные свойства функции плотности вероятности:

1.Функция плотности вероятности — неотрицательная функция Теория вероятностей

2.Интеграл в бесконечных пределах от функции плотности вероятности (если она задана на всей числовой оси) равен 1. Действительно, как частный случай формулы (4.10) имеем

Теория вероятностей

Замечание:

Если случайная величина задана только на отрезке Теория вероятностей то в (4.11) пределы интегрирования изменяются на Теория вероятностей

Теория вероятностей

Свойство (4.11) называют условием нормировки. Общий вид функции плотности вероятности представлен на рис. 4.2. Очевидно, что вероятность попадания в заданный промежуток (а, b) численно равна площади криволинейной трапеции с основанием (а, b).

Числовые характеристики случайных величин

Законы распределения являются наиболее полными характеристиками случайных величин, но на практике часто затруднительно, а иногда и просто излишне, определять законы распределения. При решении многих задач достаточно знание лишь основных суммарных характеристик случайных величин. К таким характеристикам в первую очередь относятся: математическое ожидание, дисперсия и среднее квадратичное отклонение.

Математическое ожидание

Математическое ожидание можно рассматривать как среднее вероятностное значение случайной величины. Рассмотрим пример: бросают одновременно три игральные кости. Случайная величина X — сумма выпавших очков (X меняется от 3 до 18). Легко проверить, что 18 очков будут в среднем выпадать реже, чем 15 и значительно реже, чем 12 очков. Если усреднить с некоторым «весом», учитывающим частоту появления, все возможные значения величины X, то получим число, называемое ее математическим ожиданием и являющееся «центром» распределения возможных значений рассматриваемой случайной величины.

Для дискретной случайной величины математическое ожидание вычисляется по формуле

Теория вероятностей

Для непрерывных случайных величин математическое ожидание есть величина

Теория вероятностей

Итак, математическим ожиданием (или средним) называют характеристику положения случайной величины X, которая равна средневзвешенному возможных ее значений. К формуле (4.13) относится примечание, сделанное к (4.11).

Следует подчеркнуть, что математическое ожидание есть число (неслучайная величина) — центр группирования значений случайной величины или центр рассеивания.

Пример:

Если плотность вероятности времени безотказной работы прибора есть функция Теория вероятностей то математическое ожидание, т. е. среднее время безотказной работы, есть величина

Теория вероятностей

Заметим, что размерность математического ожидания совпадает с размерностью самой случайной величины.

Пример:

Вероятность попадания при одном выстреле равна 0,4. Производится 4 выстрела. Найти математическое ожидание для числа попаданий. Согласно формуле (4.12), имеем

Теория вероятностей

Полученное число означает, что, проводя много серий по четыре выстрела в каждой, в среднем будем получать 1,6 попаданий, если вероятность попадания при одном выстреле равна 0,4. В каждой конкретной серии из четырех выстрелов будет ровно или 0, или 1, …, или 4 попадания, но в среднем за много серий их будет 1,6. Этот пример показывает, что математическое ожидание не всегда может быть равно одному из возможных значений случайной величины.

Теория вероятностей

Дисперсия

Рассмотрим две случайные величины, плотности вероятности которых представлены на рис. 4.3. Они имеют одинаковое математическое ожидание, однако значительные отклонения от центра рассеивания у первой случайной величины наблюдаются чаще, чем у второй. В этом случае говорят, что первая случайная величина имеет большее рассеивание или размытость, чем вторая.

В качестве меры рассеивания значений случайной величины используется дисперсия. Дисперсией называется математическое ожидание квадрата отклонения случайной величины от ее математического ожидания

Теория вероятностей

Для дискретной случайной величины дисперсия вычисляется по формуле:

Теория вероятностей

Для непрерывной случайной величины дисперсия вычисляется по формуле

Теория вероятностей

Из приведенных формул следует, что размерность дисперсии есть размерность случайной величины в квадрате. Для практических нужд это не всегда удобно. В этой связи чаще используется так называемое среднеквадратическое отклонение, имеющее общепринятое обозначение Теория вероятностей По определению,

Теория вероятностей

Очевидно, что для распределений, изображенных на рис. 4.3, выполняется неравенство Теория вероятностей

В качестве примера найдем дисперсию и среднеквадратическое отклонение для задачи: производится 4 выстрела, причем вероятность попадания при одном выстреле равна 0,4.

Теория вероятностей

Стандартные (наиболее распространенные) законы распределения вероятностей

В рамках теории вероятностей, на основании обобщения знаний о случайных явлениях в природе и человеческом обществе, построен ряд моделей распределения вероятностей, которые в некоторых ситуациях удовлетворительно описывают исследуемые закономерности. Для каждой модели (закона распределения) установлены и условия ее применимости. Если при рассмотрении некоторого явления исследователь считает, что имеющие место условия совпадают с условиями применимости того или иного закона, то можно воспользоваться соответ-

ствующим законом распределения и всеми знаниями о нем накопленными в рамках теории вероятности.

Биномиальное распределение

Пусть реализуется схема опытов Бернулли: проводится п одинаковых независимых опытов, в каждом из которых событие А может появиться с постоянной вероятностью р. Число появлений события А в этих n опытах есть дискретная случайная величина X, возможные значения которой: 0; 1; 2;…; m;..; n.

Законом распределения этой случайной величины является формула, определяющая вероятность того, что в конкретной серии из n опытов событие А появляется ровно m раз. Такая вероятность, а, следовательно, закон распределения, задается формулой Бернулли

Теория вероятностей

Числовые характеристики случайной величины X, распределенной по биномиальному закону:

Теория вероятностей

Пример:

Автомобиль, подъезжая к перекрестку, может продолжить движение по любой из трех дорог: А, В или С с одинаковой вероятностью. К перекрестку подъезжают пять автомобилей. Найти среднее число автомашин, которое поедет по дороге А, и вероятность того, что по дороге В поедет три автомобиля.

Решение:

Число автомашин, проезжающих по каждой из дорог, является случайной величиной. Если предположить, что все подъезжающие к перекрестку автомобили совершают поездку независимо друг от друга, то эта случайная величина распределена по биномиальному закону с Теория вероятностей

Следовательно, среднее число автомашин, которое проследует по дороге А, есть Теория вероятностей а искомая вероятность

Теория вероятностей

Распределение Пуассона

Пусть событие А может появиться в любой момент времени. При этом выполнены следующие условия:

1) события происходят независимо друг от друга;

2) появление события А на данном отрезке времени не зависит от расположения временного отрезка на оси времени;

3) вероятность появления события А за бесконечно малый интервал времени Теория вероятностей более одного раза есть бесконечно малая величина по сравнению с Теория вероятностей (в этой связи закон Пуассона называют законом редких событий).

Число появлений события А за выбранный промежуток времени t подчиняется закону Пуассона

Теория вероятностей

Здесь Теория вероятностей — среднее число событий А, появляющихся за единицу времени.

Этот закон однопараметрический, т. е. для его задания требуется знать только один параметр Теория вероятностей. Можно показать, что математическое ожидание и дисперсия в законе Пуассона численно равны: Теория вероятностей

Одним из классических примеров применения закона Пуассона является описание числа запросов на соединение, поступающих на телефонную станцию.

Пример:

Пусть в середине рабочего дня среднее число запросов равняется 2 в секунду. Какова вероятность того, что 1) за секунду не поступит ни одной заявки, 2) за две секунды поступит 10 заявок?

Решение. Поскольку правомерность применения закона Пуассона не вызывает сомнения и его параметр задан Теория вероятностей то решение задачи сводится к прямому применению формулы Пуассона (4.18)

Теория вероятностей

Теория вероятностей

Закон равномерной плотности

Пусть непрерывная случайная величина X может принимать любые значения лишь на отрезке [а,b] и нет оснований считать, что — появление одних возможных значений вероятней других. При выполнении этих условий говорят, что X распределена с равномерной плотностью. В этой связи целесообразно считать, что плотность вероятности f(х) имеет вид

Теория вероятностей

График такой функции представлен на рис. 4.4. Поскольку площадь, ограниченная любой кривой распределения, равна 1, легко найти значение константы с из равенства Теория вероятностей

Теория вероятностей

Теперь можно сказать, что случайная величина X распределена на отрезке [а,b] равномерно, если

Теория вероятностей

Основные числовые характеристики равномерно распределенной случайной величины.

Теория вероятностей

Пример:

Минутная стрелка часов делает скачок на соседнее деление, когда реальное время превышает указываемое значение на полминуты. При взгляде на часы фиксируется показываемое ими время. Какова средняя ошибка в показаниях таких часов и каков разброс этой ошибки?

Решение:

В каждый момент времени показания часов есть случайная величина, показывающая реальное время с некоторой ошибкой. Взгляд на часы производится в случайно выбранный момент, поэтому целесообразно предположить, что ошибка в показаниях часов имеет равномерную плотность распределения.

Так как рассогласование между реальным временем и показаниями часов находится в пределах от -0,5 до +0,5, то следует положить Теория вероятностей Следовательно,

Теория вероятностей

Это означает, что систематическая ошибка отсутствует,

Теория вероятностей

Нормальный закон распределения

Непрерывная случайная величина X распределена по нормальному закону, если ее плотность вероятности описывается функцией

Теория вероятностей

где m — математическое ожидание; Теория вероятностей — среднеквадратическое отклонение. Соответственно, функция распределения равна

Теория вероятностей

График плотности вероятности для нормального закона приведен на рис. 4.3. Нормальный закон возникает в тех случа-

ях, когда случайная величина X есть сумма большого числа случайных величин, распределенных по произвольному закону, но каждая из них не является доминирующей. Наиболее ярким примером является ошибка, возникающая при различных измерениях (длины, объема, массы и т.п.). Действительно, если измерительный прибор хорошо отрегулирован, то он не дает существенных систематических ошибок (иначе его следовало бы отрегулировать). Получаемые же при каждом измерении ошибки складываются из влияния множества факторов, устранить которые практически невозможно. Они зависят от изменений температуры, давления, влажности и т.п. Иногда ошибки складываются, усиливая друг друга, а иногда — компенсируя одна другую.

Для вычислений вероятности попадания случайной величины в заданный промежуток возможных значений используется приведенная функция плотности вероятности и приведенная функция распределения — функция распределения для так называемой нормированной случайной величины с Теория вероятностей Нормированная случайная величина получится, если сделать замену Теория вероятностей

Теория вероятностей

Функции f(х) и Ф*(х) табулированы. Функция Ф*(х) обладает следующими свойствами: Теория вероятностей

При вычислениях связанных с нормальным законом часто используют интеграл Лапласа, который также табулирован

Теория вероятностей

При этом следует иметь ввиду, что Теория вероятностейЕсли случайная величина распределена по нормальному закону с параметрами m и Теория вероятностей, то

Теория вероятностей

Пример:При изготовлении бумаги наблюдается отклонение ее плотности от номинала равного, 100 г на квадратный метр. Если считать, что плотность бумаги есть нормально распределенная величина с Теория вероятностей = 5 г, найти вероятность того, что плотность бумаги будет отличаться от номинала не более, чем на 2 г и не менее, чем на 3 г.

При изготовлении бумаги наблюдается отклонение ее плотности от номинала равного, 100 г на квадратный метр. Если считать, что плотность бумаги есть нормально распределенная величина с Теория вероятностей = 5 г, найти вероятность того, что плотность бумаги будет отличаться от номинала не более, чем на 2 г и не менее, чем на 3 г.

Решение задачи дается непосредственным использованием формулы (4.24)

Теория вероятностей

На практике удобно использовать правило «три сигма», которое гласит: «с вероятностью, большей, чем 0,997, случайная величина, распределенная по нормальному закону, будет принимать значения в промежутке Теория вероятностей

Пример:

Имеется партия изделий, в которой могут попадаться качественные и бракованные. Число бракованных изделий — нормально распределенная случайная величина, характеризующаяся так: среднее число бракованных изделий (математическое ожидание) составляет 12% и среднеквадратическое отклонение — 3%. Отобрано 100 изделий. Какое число бракованных изделий окажется среди них?

Решение:

Должно быть понятным, что точно ответить на такой вопрос в принципе невозможно. Однако, используя правило «три сигма», легко найти следующий ответ: можно быть практически уверенным, что бракованных деталей будет не менее Теория вероятностей и не более Теория вероятностей Формально это можно записать так Теория вероятностей

Предельные теоремы теории вероятностей

Математические законы теории вероятностей получены в результате обобщения закономерностей, свойственных массовым явлениям в обществе и в природе. Массовость понимается как большое число повторений опытов в одинаковых или сходных условиях. Было замечено, что при массовых явлениях результаты отдельных опытов практически не влияют на некоторые средние характеристики этих явлений. Этот феномен известен как устойчивость средних: «При очень большом числе испытаний средние характеристики наблюдаемых явлений перестают быть случайными и могут быть предсказаны со сколь угодно высокой точностью».

Неравенство Чебышева

Возможные значения случайной величины X группируются вокруг своего математического ожидания Теория вероятностей Однако в каждом конкретном опыте X может принять значение и весьма далекое от Теория вероятностейЕсли известен закон распределения X, то оценить вероятность далеких отклонений несложно.

Пусть задана некоторая величина Теория вероятностей Событие Теория вероятностей означает, что значение X отклонилось от своего математического ожидания больше чем на Теория вероятностей Такие отклонения будем называть «далекими». Если ограничиться только непрерывным случаем, то вероятность получить далекие отклонения можно вычислить так:

Теория вероятностей

где Теория вероятностей — область далеких отклонений: Теория вероятностей

Однако закон распределения известен далеко не всегда. Хотелось бы уметь оценивать вероятность таких событий, опираясь, например, лишь на знание дисперсии Теория вероятностей Именно такая задача и решается с помощью неравенства Чебышева.

Дисперсия случайной величины X? распределенной по непрерывному закону f(х)? равна:

Теория вероятностей

Если под интегралом заменить Теория вероятностей на величину Теория вероятностей то неравенство в (4.25) лишь усилится. Следовательно,

Теория вероятностей

Отсюда имеем неравенство Чебышева:

Теория вероятностей

Пример:

Оценить вероятность того, что значения случайной величины будут отклоняться от ее математического ожидания меньше, чем на Теория вероятностей

Решение:

Нам необходимо оценить вероятность следующего события Теория вероятностей Это событие будет противоположным тому, что рассматривается в неравенстве Чебышева, если положить Теория вероятностей Переход к противоположному событию осуществляется по правилу Теория вероятностей Следовательно, используя (4.26) при Теория вероятностей, имеем

Теория вероятностей

Мы получили очень важный результат: «Каков бы ни был закон распределения для X, вероятность того, что отклонение этой случайной величины от ее математического ожидания будет в пределах трех среднеквадратических ожиданий, не может быть менее 8/9». Иначе говоря, значения случайной величины весьма редко отклоняются от своего математического ожидания больше, чем на «три сигма». Это правило также получило название правило трех Теория вероятностей (см. главу 4.2).

На практике, получив оценки для Теория вероятностей легко оценить интервал, в котором будут находиться практически все значения

случайной величины. Этот интервал задается условием

Теория вероятностей

Величина 8/9 является нижней границей оценки. Для конкретных законов распределения попадание в интервал (4.27) значительно больше, чем 8/9. Например, для нормального закона вероятность попадая в такой Интервал оценивается величиной 0,997.

Закон больших чисел

Математическим основанием того факта, что значения случайной величины группируются около некоторых постоянных величин, является закон больших чисел.

Исторически первой формулировкой закона больших чисел стала теорема Бернулли: «При неограниченном увеличении числа одинаковых и независимых опытов n частота появления события А сходится по вероятности к его вероятности», т. е.

Теория вероятностей

где частота появления события А в n опытах Теория вероятностей

Содержательно выражение (4.28) означает, что при большом числе опытов частота события А может заменять неизвестную вероятность этого события, и чем больше число проведенных опытов, тем ближе р* к р. Интересен исторический факт. К. Пирсон бросал монету 12000 раз и герб у него выпал 6019 раз (частота 0,5016). При бросании этой же монеты 24000 раз он получил 12012 выпадений герба, т. е. частоту 0,5005.

Наиболее важной формой закона больших чисел является теорема Чебышева: при неограниченном возрастании числа независимых, имеющих конечную дисперсию и проводимых в одинаковых условиях опытов среднее арифметическое наблюденных значений случайной величины сходится по вероятности к ее математическому ожиданию. В аналитической форме эта теорема может быть записана так:

Теория вероятностей

Теорема Чебышева, кроме фундаментального теоретического значения, имеет и важное практическое применение, например, в теории измерений. Проведя n измерений некоторой величины х, получают различные несовпадающие значения: Теория вероятностейТеория вероятностей За приближенное значение измеряемой величины х принимают среднее арифметическое наблюденных значений

Теория вероятностей

При этом, чем больше будет проведено опытов, тем точнее будет полученный результат. Действительно, дисперсия величины Теория вероятностей убывает с возрастанием числа проведенных опытов, т.к. Теория вероятностей то

Теория вероятностей

Соотношение (4.30) показывает, что и при высокой неточности приборов измерения (большая величина Теория вероятностей) за счет увеличения количества измерений можно получать результат со сколь угодно высокой точностью.

Центральная предельная теорема

Очень часто значение, которое принимает исследуемая величина Y, является результатом суммарного воздействия ряда случайных величин Теория вероятностей Если эти величины слабо зависят друг от друга и ни одна из них не является доминирующей, то их сумма имеет закон распределения, весьма близкий к нормальному. Такая ситуация наиболее характерна для процессов измерения. Дело в том, что результат измерения складывается под влиянием многочисленных независимых элементарных причин, причем влияние каждой из причин на суммарный результат незначительно. Например, на величину температуры в конкретном месте комнаты оказывает влияние близость отопительных приборов, наличие и расположение в комнате людей, потоки воздуха, влажность, температура воздуха на улице, наличие источников света и их мощность, расположение окон, дверей и т.д.

Все формы центральной предельной теоремы (ЦПТ) посвящены установлению условий возникновения нормального закона. Наиболее простой формой ЦПТ является теорема Ляпунова: «Если Теория вероятностей независимые случайные величины, имеющие один и тот э/се закон распределения, то при неограниченном увеличении числа п закон распределения суммы Теория вероятностей неограниченно приближается к нормальному «.

На практике при Теория вероятностей закон распределения суммы случайных величин, имеющих какой угодно (возможно и неизвестный) закон распределения, считают нормальным.

Пример:

Случайная величина Y есть сумма 24 случайных величин, распределенных по закону равномерной плотности на отрезке [0,1]. Найти вероятность того, что Y примет значение в пределах от 10 до 13.

Решение:

Ввиду того, что закон распределения величин X равномерный, легко найти их математические ожидания и дисперсию

Теория вероятностей

Y есть сумма 24 случайных величин. Поэтому можно считать ее закон распределения близким к нормальному с параметрами:

Теория вероятностей

Требуемая вероятность может быть найдена при помощи известной формулы (4.24):

Теория вероятностей

Частным случаем ЦПТ является теорема Лапласа: «Если производится n опытов, в каждом из которых событие А появляется с вероятностью р, то справедливо соотношение

Теория вероятностей

Эта формула позволяет вести расчеты, когда реализуется система случаев при большом числе N.

Пример:

Производится компьютерный набор текста из 100 страниц. Вероятность сделать ошибку при наборе одной страницы равна 0,2. Найти вероятность того, что при наборе 100 страниц ошибок будет не более 25.

Решение примера сводится к непосредственному применению формулы (4.31) при Теория вероятностей т.е.

Теория вероятностей

Предельные теоремы образуют своеобразный мостик между теорией вероятностей как математической наукой и одной из наиболее важных и имеющих практическое применение областей современных знаний — прикладной статистикой.

Система двух случайных величин

Функции распределения:

Рассмотрим две случайные величины X и Y. Двумерной случайной величиной (системой двух случайных величин) называют случайный вектор, первый компонент которого — значение случайной величины X, а второй — значение случайной величины Y. Сами величины X и Y называют случайными координатами. Рассматриваются два основных случая: обе случайные величины X и Y дискретны или обе величины непрерывны.

Двумерная случайная величина называется дискретной, если дискретны случайные величины X и Y, т. е. Теория вероятностейТеория вероятностей Двумерная случайная величина называется непрерывной, если ее функция распределения непрерывна и плотность распределения существует и кусочно-непрерывна.

Закон распределения двумерной случайной величины обычно задают в виде таблицы, в которой указаны значения случай-

ных величин X и Y, а также вероятности Теория вероятностей появления пары Теория вероятностей причем Теория вероятностей

Теория вероятностей

Событие Теория вероятностей можно представить как сумму попарно несовместных событий Теория вероятностей Теория вероятностейследовательно, Теория вероятностей

Аналогично Теория вероятностей То есть первый и последний столбцы таблицы дают закон распределения случайной величины X, а первая и последняя строки — закон распределения случайной величины Y. Эти законы позволяют определить Теория вероятностей а также Теория вероятностейТеория вероятностей

Функцией распределения двумерной случайной величины F(х, у) называется вероятность того, что Теория вероятностей и, одновременно Теория вероятностей Справедливы следующие правила (см. главу 4.2):

Теория вероятностей

4.При Теория вероятностей функция распределения системы становится функцией распределения случайной величины X

Теория вероятностей

При Теория вероятностей функция распределения системы становится функцией распределения случайной величины Y

Теория вероятностей

5.Вероятность попадания точки в полуполосу:

Теория вероятностей

Плотностью распределения вероятностей называют вторую смешанную частную производную от функции распределения

Теория вероятностей

Свойства плотности вероятности:

Теория вероятностей

3.Одномерные плотности вероятностей случайных величин Теория вероятностей

Теория вероятностей

Условные законы распределения

Для дискретных случайных величин (см. главу 4.2) было введено понятие условной вероятности. В нашем случае также

Теория вероятностей

причем в общем случае условные вероятности Теория вероятностей и Теория вероятностей могут не совпадать с безусловными Теория вероятностей и Теория вероятностей

Условным распределением составляющей X при Теория вероятностей называют совокупность условных вероятностей

Теория вероятностей

вычисленных в предположении, что событие Теория вероятностей наступило. Аналогично находятся условные законы распределения случайной величины Y

Теория вероятностей

Для непрерывных случайных величин вводят понятие условной плотности совместного распределения составляющей X при данном значении Теория вероятностей

Теория вероятностей

Соответственно, условные плотности для составляющей Y

Теория вероятностей

Свойства условных плотностей:

Теория вероятностей

Условные математические ожидания для случайной величины Y при Теория вероятностей

для дискретной случайной величины

Теория вероятностей

для непрерывной случайной величины

Теория вероятностей

Условное математическое ожидание Теория вероятностей есть функция от х

Теория вероятностей

и называется функцией регрессии Y на X.

Аналогично определяется и функция регрессии X на Y

Теория вероятностей

Корреляционный момент и коэффициент корреляции

Корреляционным моментом Теория вероятностей называется математическое ожидание произведения отклонений случайных величин от своих математических ожиданий

Теория вероятностей

Коэффициентом корреляции r называется отношение корреляционного момента к произведению средних квадратических отклонений Теория вероятностей

Теория вероятностей

Основное свойство коэффициента корреляции: коэффициент корреляции ограничен, причем его значения лежат в пределах Теория вероятностей

Случайные величины X и Y, для которых коэффициент корреляции не равен нулю, называются коррелированными. Если коэффициент корреляции равен нулю, то случайные величины называют некоррелированными.

Условие независимости случайных величин

Две случайные величины независимы, если закон распределения одной из них не зависит от того, какое значение принимает вторая величина. Критерий независимости: две случайные величины независимы, если их функция распределения представима в виде произведения одномерных функций распределения, т. е.

Теория вероятностей

Для дискретных случайных величин это означает, что

Теория вероятностей

Для непрерывных случайных величин это означает, что

Теория вероятностей

Если случайные величины независимы, то они не коррели-рованы, коэффициент корреляции равен нулю.

Замечание:

Если случайные величины не коррелированы, т. е. коэффициент корреляции равен нулю, то нельзя сказать, зависимы они или нет. Такие величины называются «подозрительными на независимость» и проверяют их на независимость по критерию независимости. Единственным исключением является двумерное нормальное распределение. Только для него из некоррелированности следует независимость.

Средняя квадратическая регрессия. Метод наименьших квадратов

Рассмотрим систему случайных величин (X, Y), причем величины X и Y зависимы, но вид зависимости неизвестен. Подберем функцию f(Х), которая с максимальной точностью совпадала бы с величиной Y, такую функцию называют приближением величины Y. Приближение будет наилучшим, если средний квадрат отклонения случайной величины Y от функции f(Х) минимален, т. е. среди всех функций ищем такую, чтобы достигался Теория вероятностей Как правило, в качестве функции f(Х) выбирают линейную Теория вероятностей или, реже, квадратичную Теория вероятностей функции. Тем самым задачу упрощают — вместо

неизвестной функции f(Х) ищут неизвестные коэффициенты в известных функциях.

Если в качестве приближения для Y выбирают линейную функцию, то коэффициенты а и b должны быть выбраны так, чтобы достигался минимум величины

Теория вероятностей

Экстремум функции двух переменных достигается в том случае, когда обе частные производные по переменным а и b равны нулю, т.е. когда

Теория вероятностей

Минимум достигается, если Теория вероятностей при этом

Теория вероятностей

Это уравнение называют уравнением средней квадратично регрессии, а линейное приближение случайной величины Y есть

Теория вероятностей

Так как величины Теория вероятностей положительны, то знак коэффициента корреляции позволяет определить тенденции изменения случайной величины Y при увеличении случайной величины X:

если Теория вероятностей то Y имеет тенденцию к убыванию,

если Теория вероятностей то Y имеет тенденцию к возрастанию,

если Теория вероятностей то Y просто линейно зависит от X, т. е. выражение

Теория вероятностей

является точным равенством, а зависимость Y от X является функциональной.

Величина Теория вероятностей называется коэффициентом детерминации и определяет, какой процент вариации (изменения) одной величины вызван линейным влиянием другой.

Пример:

Дана система двух случайных величин. Найти: числовые характеристики системы, функции регрессии, коэффициент корреляции, уравнение регрессии, коэффициент детерминации.

Теория вероятностей

Функции регрессии

Теория вероятностей

Корреляционный момент и коэффициент корреляции

Теория вероятностей

Величины X и Y коррелированы, между ними существует линейная связь. С возрастанием одной величины другая имеет тенденцию к убыванию.

Уравнение средней квадратичной регрессии,

Теория вероятностей

Теория вероятностей

Коэффициент детерминации

Теория вероятностей вариации случайной величины Y вызвано линейным влиянием другой.

Теоремы и формулы по теории вероятностей

Многие явления в природе, экономике и других областях носят
случайный характер, т.е. невозможно точно предсказать, как
явление будет происходить. Теория вероятностей дает математическую модель для описания явлений такого рода.

Практика показывает, что, наблюдая в совокупности массы
однородных случайных явлений, мы обнаруживаем в них вполне
определенные закономерности, свойственные именно массовым
явлениям. Например, если много раз подбрасывать монету, то частота появления герба постепенно стабилизируется, приближаясь к 1/2 .

Событие — это всякий факт, который в результате опыта может произойти или не произойти. Примерами событий являются появление герба при подбрасывании монеты или появление четырех гербов подряд при четырехкратном подбрасывании монеты. Из этих примеров видно, что каждое из событий обладает какой-то степенью возможности: одни — большей, другие — меньшей. Например, появление герба при одном бросании более возможно, чем появление подряд четырех гербов при четырехкратном бросании.

Для описания степени возможности появления того или иного
события используется понятие вероятности события.

Вероятность события есть численная мера степени объективной
возможности этого события.

Аксиома:

Каждому случайному событию А поставлено в
соответствие число Р(А), которое называется вероятностью событиям,
причем Теория вероятностей

Достоверным называется событие, которое в результате опыта обязательно должно произойти.

Аксиома:

Вероятность достоверного события равна единице.

Невозможным называется событие, которое в результате опыта произойти не может.

Аксиома:

Вероятность невозможного события равна нулю.

Несколько событий в данном опыте образуют полную группу событий, если в результате опыта непременно должно произойти хотя бы одно из них.

Примером полной группы событий является выпадение герба или цифры при подбрасывании монеты.

Несколько событий называются несовместными в данном опыте, если никакие два из них не могут произойти вместе. Например, появление 1, 2, 3, 4, 5, 6 очков при бросании игральной кости.

Несколько событий в данном опыте считаются равновозможными, если по условиям симметрии есть основание считать, что ни одно из этих событий не является объективно более возможным, чем другое. Например, выпадение герба и цифры при подбрасывании монеты или появление 1, 2, 3, 4, 5, 6 очков при бросании игральной кости.

Пусть опыт повторяется n раз и при этом подсчитывается, как часто происходит интересующее нас событие А . Допустим, что оно произошло m раз. Отношение

Теория вероятностей

называется относительной частотой случайного события А в n опытах. Практика показывает, что во многих случаях при увеличении n частота приближается к некоторому постоянному значению, называемому вероятностью появления события А .

Случайной величиной называется величина, которая в результате опыта может принять то или иное значение, не известное заранее.

Случайные величины, принимающие только отдельные друг от друга значения, которые можно заранее пересчитать, называются прерывистыми, или случайными дискретными величинами (например, количество посещений магазина в течение дня).

Случайные величины, возможные значения которых непрерывно заполняют некоторый промежуток, называются случайными непрерывными величинами (например, ошибка взвешивания тела на аналитических весах).

Теоремы сложения и умножения вероятностей

Суммой двух событий А и В называется событие С, состоящее в выполнении события А или события В или обоих вместе.

Например, в магазине, торгующем холодильниками и стиральными машинами, в течение пяти минут может быть куплен холодильник (событие А) или стиральная машина (событие В). Событие С = А + В — в течение пяти минут куплен холодильник либо стиральная машина, либо и то и другое вместе.

Суммой нескольких событий называется событие, состоящее в появлении хотя бы одного из этих событий.

Произведением двух событий А и В называется событие С, состоящее в совместном появлении событий А и В.

Например, покупается холодильник, причем событие А состоит в том, что холодильник неисправен. Одновременно покупается стиральная машина. Событие В — машина неисправна. С = А*В — неисправны холодильник и стиральная машина.

Произведением нескольких событий называется событие, состоящее в совместном появлении этих событий.

Теорема сложения:

Вероятность суммы n несовместных событий равна сумме вероятностей этих событий:

Теория вероятностей

где i — номер события.

Для двух событий

Р(А + В) = Р(А) + Р(В).

Если события Теория вероятностей образуют полную группу несовместных событий, то сумма их вероятностей равна единице:

Теория вероятностей

Действительно, так как события Теория вероятностей образуют полную группу, то появление хотя бы одного из них является достоверным событием:

Теория вероятностей

Так как Теория вероятностей — несовместные события, то, используя (11.1), получим (11.2).

Противоположными называются два несовместных события, образующих полную группу.

Например, противоположными событиями являются выпадение герба и цифры при подбрасывании монеты.

Сумма вероятностей двух противоположных событий А и Теория вероятностей равна единице:

Теория вероятностей

где Теория вероятностей — событие, противоположное событию А .

Пример:

В лотерее участвует 1000 билетов, из которых на один билет выпадает выигрыш 200 руб., на 10 билетов — 50 руб., на 20 билетов — 10 руб. На остальные билеты выпадает выигрыш 1 руб. Найти вероятность выигрыша не менее 10 руб. при покупке одного билета.

Решение:

Так как события А (выигрыш 200 руб.), В (выигрыш 50 руб.) и С (выигрыш 10 руб.) несовместны, то вероятность наступления события D (выигрыш не менее 10 руб.) является суммой вероятностей первых трех событий, т.е.

Теория вероятностей

Пример:

Круговая мишень состоит из трех зон. Вероятность попадания при одном выстреле в первую зону равна 0,18, во вторую — 0,22, в третью — 0,3. Определить вероятность промаха.

Решение:

Вероятность попадания в мишень

Теория вероятностей

Вероятность промаха находим по формуле

Теория вероятностей

Для совместных событий А и В вероятность суммы этих событий вычисляется по формуле

Р(А + В) = Р(А) + Р(В)-Р(АВ), (11.4)

где Р(АВ) — вероятность совместного появления событий А и В .

Два события А и В называются независимыми, если вероятность события А не зависит от того, произошло событие В или нет. В противном случае события А и В называются зависимыми.

Пример независимого события:

При подбрасывании двух монет рассматриваются два события: А — появление герба на первой монете; В — появление герба на второй монете. Вероятность события А не зависит от того, произошло событие В или нет, т.е. событие А независимо от события В.

Пример зависимого события:

В урне два белых шара и один черный. Два лица вынимают из урны по одному шару. Рассматриваются два события: А — появление белого шара у первого лица; В — появление белого шара у второго лица. Вероятность события А до того, как известно что-либо о событии В , равна 2/3. Если стало известно, что событие В произошло, то вероятность события А становится равной 1/2. Таким образом, событие А зависит от события В .

Вероятность события А, вычисленная при условии, что имело место событие В, называется условной вероятностью события А :

Р(АВ).

Для последнего примера Р(А) = 2/3 , Р(АВ) = 1/2.

Вероятность произведения двух событий равна произведению вероятности одного из них и условной вероятности другого, вычисленной при условии, что первое событие имело место:

Теория вероятностей

Вероятность произведения n независимых событий равна произведению вероятностей этих событий:

Теория вероятностей

Пример:

В урне 2 белых и 3 черных шара. Из урны вынимают подряд два шара. Найти вероятность того, что оба шара белые.

Решение:

Обозначим событие, состоящее в появлении двух белых шаров подряд, буквой А . Событие А представляет собой произведение двух событий:

Теория вероятностей

где Теория вероятностей — появление белого шара при первом вынимании; Теория вероятностей

— появление белого шара при втором вынимании.

Теория вероятностей

Пример:

Условия примера 11.3, но при первом вынимании шар возвращают в урну и шары перемешиваются.

Решение:

Теория вероятностей

Полная вероятность

Следствием теорем сложения и умножения вероятностей является формула полной вероятности.

Пусть требуется определить вероятность некоторого события А , которое может произойти с одним из событий Теория вероятностей образующих полную группу несовместных событий. Эти события называются гипотезами.

Вероятность события А вычисляется как сумма произведений вероятностей каждой гипотезы на вероятность события при этой гипотезе:

Теория вероятностей

Пример:

Имеются три урны. В первой урне 2 белых и 1 черный шар, во второй — 3 белых и 1 черный, в третьей — 2 белых и 2 черных шара. Наугад выбирают одну из урн и из нее вынимают шар. Найти вероятность того, что этот шар белый.

Решение:

Рассмотрим три гипотезы:

Теория вероятностей — выбор первый урны;

Теория вероятностей— выбор второй урны;

Теория вероятностей — выбор третьей урны.

Появление белого шара — событие А .

Так как гипотезы равнозначны, то

Теория вероятностей

По формуле (11.7) находим

Теория вероятностей

Формула Байеса

Пусть имеется полная группа несовместных гипотез Теория вероятностейВероятность данных гипотез известна до опыта и равна Теория вероятностейВ результате проведения опыта произошло некоторое событие А . Определить, как изменятся вероятности гипотез в связи с появлением этого события.

Из теоремы умножения (11.5) имеем:

Теория вероятностей

откуда

Теория вероятностей

Подставив вместо Р(А) его значение (11.7), получим

Теория вероятностей

Формула (11.8) называется формулой Байеса, или теоремой гипотез.

Пример:

40% автомашин собирается из деталей высокого качества, а остальные — из деталей обычного качества. Если автомашина собирается из деталей высокого качества, то вероятность безотказной работы в течение трех лет равна 0,95, а если из деталей обычного качества, то эта вероятность равна 0,7. Купленная автомашина безотказно работала в течение трех лет. Найти вероятность того, что она собрана из высококачественных деталей.

Решение:

Возможны две гипотезы:

Теория вероятностей — автомашина собрана из высококачественных деталей;

Теория вероятностей — автомашина собрана из обычных деталей.

Вероятность этих гипотез до опыта: Теория вероятностей

В результате опыта наблюдено событие А — автомашина работала безотказно три года. Условные вероятности тех же событий при гипотезах Теория вероятностей и Теория вероятностей:

Теория вероятностей

По формуле (11.8) находим вероятность того, что автомашина собрана из высококачественных деталей:

Теория вероятностей

Теорема о повторении опытов

Пусть в результате каждого из n независимых опытов может появиться или не появиться некоторое событие А . Вероятность появления события А в каждом опыте равна р, а вероятность его непоявления равна q = 1 — р . Требуется найти вероятность Теория вероятностей того, что событие А в n опытах появится ровно m раз.

Результат решения поставленной задачи называется теоремой о повторении опытов.

Рассмотрим событие Теория вероятностей состоящее в том, что событие А в n опытах появится ровно m раз. Это событие можно осуществить различными способами. Разложим событие Теория вероятностей на сумму произведений событий, состоящих в появлении или непоявлении события А в отдельном опыте. Обозначим Теория вероятностей появление события А в i-м опыте, Теория вероятностей — непоявление события А в i-м опыте.

Каждый вариант появления события Теория вероятностей должен состоять из m появлений и n-m непоявлений события А , т.е. из m событий А и n — m событий Теория вероятностей с различными индексами. Таким образом,

Теория вероятностей

В каждое произведение событие А входит m раз, а событие Теория вероятностей — n-m раз. Число всех комбинаций такого рода равно числу сочетаний из n элементов по m:

Теория вероятностей

Вероятность каждой такой комбинации по теореме умножения для независимых событий равна Теория вероятностей Так как комбинации между собой несовместимы, то по теореме сложения вероятность события Теория вероятностей

Теория вероятностей

Формула для бинома Ньютона имеет вид

Теория вероятностей

Сопоставив (11.9) с формулой бинома Ньютона, видим, что вероятности Теория вероятностей представляют собой члены разложения бинома Теория вероятностей поэтому распределение вероятностей (11.9) называется биномиальным распределением. Как следует из равенства (11.10), сумма всех вероятностей Теория вероятностей (m = 0, 1, 2, …, n) равна единице:

Теория вероятностей

Во многих случаях, кроме вероятности Теория вероятностей появления некоторого события m раз, рассматривается вероятность Теория вероятностей появления этого события не менее m раз. По теореме сложения

Теория вероятностей

Переходя к противоположному событию, вероятность (11.12) вычисляют по формуле

Теория вероятностей

Пример:

Вероятность рождения мальчика равна 0,515. Kaк велика вероятность того, что из десяти наугад выбранных новорожденных будет четыре мальчика?

Решение:

Используя формулу (11.9), получим:

Теория вероятностей

Пример:

Производится четыре независимых выстрела по
мишени, причем вероятность попадания в мишень при одном выстреле равна 0,1. Найти вероятность промаха, а также вероятность одного, двух, трех и четырех попаданий.

Решение:

По формуле (11.9) находим:

Теория вероятностей

Сумма всех вероятностей равна единице. Действительно,

Теория вероятностей

Закон распределения случайной величины

Как уже указывалось, случайной называется величина, которая в
зависимости от исхода опыта принимает различные значения,
неизвестные заранее. Примерами случайных величин могут служить:

■ цифры при доставании бочонка при игре в лото;

■ число появлений герба при двух бросаниях монеты (возможные значения 0, 1, 2);

■ ошибка при измерении роста человека;

■ время безотказной работы прибора.

Обычно случайные величины обозначают прописными буквами
а их возможные значения — строчными. Например, X — число появлений герба при двух бросаниях монеты, а Теория вероятностей
возможные значения.

Пусть X — случайная величина, Теория вероятностей— ее возможные
значения, Теория вероятностей — соответствующие вероятности
возможных значений. Возможные значения Теория вероятностей образуют полную группу несовместных событий, т.е. сумма вероятностей этих значений случайной величины равна единице:

Теория вероятностей

Указанная суммарная величина каким-то образом распределена
между отдельными значениями. В теории вероятностей случайная
величина полностью характеризуется этим распределением, т.е.
вероятностью каждого из возможных значений. Так устанавливается
закон распределения случайной величины. Про случайную величину
обычно говорят, что она подчиняется данному закону распределения.

Для дискретных случайных величин простейшей формой задания закона распределения является ряд распределения случайной
величины, заданной в виде таблицы (табл. 11.1).

Таблица 11.1

Теория вероятностей

Закон распределения дискретной случайной величины может быть представлен также в виде многоугольника распределения (рис. 11.1).

Теория вероятностей

Примером ряда распределения дискретной случайной величины является биномиальное распределение (11.9).

Пример:

Производится три выстрела по мишени. Вероятность попадания при каждом выстреле равна 0,4. За каждое попадание
стрелку засчитывается одно очко. Построить ряд распределения числа выбитых очков.

Решение:

Пусть X — число выбитых очков. Возможные
значения величины X: Теория вероятностей Вероятность этих
значений находим по формуле (11.9):

Теория вероятностей

Ряд распределения представлен в табл. 11.2.

Таблица 11.2

Теория вероятностей

На рис. 11.2 представлен многоугольник распределения. ►

Теория вероятностей

Ряд распределения не является универсальной характеристикой случайной величины. Для непрерывной случайной величины нельзя построить ряд распределения, так как она имеет бесчисленное множество случайных значений. Универсальной характеристикой случайной величины является интегральная функция распределения, которая используется для описания и дискретных, и непрерывных случайных величин.

Интегральной функцией распределения (функцией распределения), или интегральным законом распределения, называется вероятность появления случайной величины меньше заданной:

F(x) = P(X<x). (11.15)

Основные свойства функции распределения:

1.Функция распределения F(x) есть неубывающая функция своего аргумента, т.е. при Теория вероятностей имеем Теория вероятностей

2.На минус бесконечности функция распределения равна нулю.

3.На плюс бесконечности функция распределения равна единице.

Функцией распределения любой дискретной случайной
величины является разрывная ступенчатая функция, скачки которой
происходят в точках, соответствующих возможным значениям
случайной величины, и равны вероятностям этих значений.

Пример:

Построить функцию распределения для примера 11.9.

Решение:

Для ряда распределения, представленного в
табл. 11.2, функция распределения имеет следующие значения:

1) при Теория вероятностей F(x)=0;

2) при Теория вероятностей F(x)=0,216;

3) при Теория вероятностей F(x)=0,648;

4) при Теория вероятностей F(x)=0,936;

5) при 3 < х F(x)=1.

График функции распределения представлен на рис. 11.3. В точках разрыва функция распределения принимает значения, отмеченные на чертеже косыми линиями. ►

Теория вероятностей

При увеличении числа возможных значений случайной величины и уменьшении интервалов между ними число скачков становится
больше, а сами скачки — меньше. В пределе функция распределения
становится непрерывной. Непрерывная функция распределения
используется для описания непрерывной случайной величины.

При помощи функции распределения можно указать вероятность попадания как дискретной, так и непрерывной случайной величины в заданный полуоткрытый промежуток

Теория вероятностей

Данная вероятность равна приращению функции распределения
на заданном промежутке:

Теория вероятностей

Пусть имеется непрерывная величина X с непрерывной и
дифференцируемой функцией распределения F(x). Вероятность
попадания этой величины в промежуток от х до Теория вероятностей равна

Теория вероятностей

Рассмотрим предел

Теория вероятностей

Функция f(x) = F'(x) называется плотностью распределения
непрерывной случайной величины X.

При заданной плотности распределения вероятность того, что
случайная величина попадет в заданной промежуток Теория вероятностей
равна (рис. 11.4)

Теория вероятностей

Теория вероятностей

Функция распределения выражается через плотность распределения по формуле

Теория вероятностей

Основные свойства плотности распределения:

1.Плотность распределения есть неотрицательная функция:

Теория вероятностей

2.Интеграл в бесконечных пределах от плотности распределения равен единице:

Теория вероятностей

Моменты распределения случайной величины

На практике применяются начальные и центральные моменты
распределения.

Начальным моментом порядка s дискретной и непрерывной
случайных величин называются соответственно соотношения вида:

Теория вероятностей

Теория вероятностей

Первый начальный момент называется математическим
ожиданием.
Математическое ожидание будем обозначать символами
М[х] или Теория вероятностей

Теория вероятностей

Пользуясь знаком математического ожидания, формулы (11.20)
и (11.21) можно объединить в одну:

Теория вероятностей

Действительно, формулы (11.20) и (11.21) по структуре
полностью аналогичны формулам (11.22) и (11.23), с той разницей, что в них вместо Теория вероятностей и х стоят соответственно Теория вероятностей

Начальным моментом порядка s случайной величины X называется
математическое ожидание степени s данной случайной величины.

Центральной случайной величиной, соответствующей величине
X, называется отклонение случайной величины X от ее
математического ожидания:

Теория вероятностей

Центральным моментом порядка s случайной величины X называется математическое ожидание степени s соответствующей центральной случайной величины:

Теория вероятностей

Второй центральный момент называется дисперсией случайной
величины
и обозначается D[X]:

Теория вероятностей

Для непосредственного вычисления дисперсий дискретной и
непрерывной случайных величин могут быть использованы формулы:

Теория вероятностей

Теория вероятностей

Дисперсия (второй центральный момент) может быть выражена
через второй начальный момент и математическое ожидание.
Действительно,

Теория вероятностей

Дисперсия случайной величины характеризует рассеивание
значений случайной величины около ее математического ожидания.

Квадратный корень из дисперсии называется средним квадратическим отклонением случайной величины:

Теория вероятностей

Третий центральный момент служит для характеристики
асимметрии плотности распределения. Если плотность распределения симметрична относительно математического ожидания, то все центральные моменты нечетного порядка равны нулю.

Моментами распределения широко пользуются тогда, когда
закон распределения случайной величины неизвестен.

Типы законов распределения случайных величин

Закон равномерной плотности

Случайная величина называется равномерно распределенной на
промежутке
[a, b], если ее плотность вероятности на этом промежутке постоянна, а вне промежутка равна нулю (рис. 11.5).

Теория вероятностей

Так как

Теория вероятностей

то

Теория вероятностей

Математическое ожидание определяется по формуле (11.23)

Теория вероятностей

Дисперсию находим по формуле (11.28):

Теория вероятностей

Среднее квадратическое отклонение определяется соотношением
(11.30):

Теория вероятностей

Пример:

Пусть случайная величина распределена
равномерно на промежутке [0; 2]. Определить значение функции f(х) на этом интервале и вероятность попадания случайной величины в
промежуток [1; 1,5], а также математическое ожидание и среднее
квадратическое отклонение этого распределения.

Решение:

Плотность вероятности на исследуемом промежутке

Теория вероятностей

Вероятность попадания случайной величины в промежуток [1; 1,5]

Теория вероятностей

Математическое ожидание

Теория вероятностей

Среднее квадратическое отклонение

Теория вероятностей

Закон Пуассона

Рассмотрим дискретную случайную величину X, которая может
принимать только целые неотрицательные значения m = 0, 1, 2, 3, …,
причем последовательность этих значений неограниченна.

Случайная величина X распределена по закону Пуассона, если
вероятность того, что она примет определенное значение m, определяется соотношением

Теория вероятностей

где а — некоторая положительная величина, называемая параметром
закона Пуассона.

Величина, заданная выражением (11.31) является членом ряда
распределения, так как его сумма Теория вероятностей Действительно, так как

Теория вероятностей

Формулу для математического ожидания (11.22) случайной
величины, распределенной по закону Пуассона, можно записать в виде

Теория вероятностей

так как первый член суммы при m = 0 равен нулю. Перепишем эту
формулу в виде

Теория вероятностей

Таким образом, математическое ожидание случайной величины
равно параметру закона Пуассона а.

Дисперсия распределения случайной величины, распределенной
по закону Пуассона, определяется по формуле (11.29).
Предварительно найдем второй начальный момент:

Теория вероятностей

Учитывая, что для распределения Пуассона Теория вероятностей получим

D[X] = a.

Таким образом, дисперсия случайной величины, распределенной по
закону Пуассона, равна ее математическому ожиданию.

Пример:

На автоматическую телефонную станцию
поступают вызовы со средней плотностью 300 вызовов в час. Число вызовов на любом участке времени распределено по закону Пуассона. Найти вероятность того, что:

1) за две минуты на станцию поступит ровно три вызова;

2) за две минуты на станцию поступит хотя бы один вызов;

3) за две минуты на станцию поступит не менее трех вызовов.

Решение:

При заданных условиях параметр закона Пуассона,
равный среднему числу вызовов за две минуты,

Теория вероятностей

1.Вероятность поступления на станцию ровно трех вызовов за две
минуты определяется по формуле (11.31):

Теория вероятностей

2.Прежде чем рассчитать вероятность поступления хотя бы одного
вызова за две минуты определим вероятность того, что за эти две
минуты не поступит ни одного вызова:

Теория вероятностей

Вероятность поступления за две минуты хотя бы одного вызова

Теория вероятностей

3.При расчете вероятности поступления не менее трех вызовов за
две минуты предварительно найдем вероятность поступления не более двух вызовов:

Теория вероятностей

Вероятность поступления за две минуты не менее трех вызовов равна

Теория вероятностей

Нормальный закон распределения

Это наиболее часто встречающийся на практике закон.

Нормальный закон распределения, или закон Гаусса, характеризуется плотностью распределения вида

Теория вероятностей

Вычислим основные характеристики случайной величины,
подчиняющейся нормальному закону распределения.

Математическое ожидание

Теория вероятностей

Введем замену:

Теория вероятностей

Тогда

Теория вероятностей

Известно, что первый интеграл равен нулю, а второй, называемый интегралом Эйлера—Пуассона, —

Теория вероятностей

Таким образом,

М[Х] = m.

Дисперсия

Теория вероятностей

Используя замену (11.33), получим

Теория вероятностей

Интегрируя по частям, найдем

Теория вероятностей

Первое слагаемое в скобках равно нулю, а второе Теория вероятностей (см.
(11.34)). Отсюда

Теория вероятностей

Таким образом, входящие в нормальный закон распределения
постоянные являются математическим ожиданием, дисперсией и
средним квадратическим отклонением. Размерности
математического ожидания и среднего квадратического отклонения совпадают с размерностью случайной величины X.

Кривая распределения по нормальному закону имеет
симметричный холмообразный вид (рис. 11.6).

Теория вероятностей

Функция распределения (11.19) нормального закона определяется соотношением:

Теория вероятностей

Введем замену:

Теория вероятностей

Тогда

Теория вероятностей
Интеграл (11.35) не выражается через элементарные функции и
называется функцией нормального распределения. Функция F(x)
табулирована. Иногда эту функцию представляют в виде

F(х) = 0,5 + Ф(х),

где

Теория вероятностей

Функция Ф(х) называется интегралом вероятности.

Решим следующую задачу. От точки х=m отложим отрезки
длиной Теория вероятностей (рис. 11.7)

Теория вероятностей

Вычислим вероятность попадания случайной величины в промежутки Теория вероятностей Искомая вероятность находится по формуле (11.36) при равенстве верхнего предела соответственно 1, 2 и 3. Используя таблицу для интеграла вероятностей [8, с. 578], находим:

Теория вероятностей

Вероятность попадания случайной величины в промежуток Теория вероятностей

Теория вероятностей

Отсюда следует, что для нормального распределения с точностью 2,7% все рассеивание укладывается на участке Теория вероятностейЭто позволяет, зная среднее квадратическое отклонение и математическое ожидание случайной величины, ориентировочно указать промежуток ее возможных значений. Такой способ оценки возможных значений называется «правилом трех сигм».

Пример:

При измерении расстояния возможная ошибка распределена по нормальному закону. Среднее квадратическое отклонение этой случайной величины равно 0,8 см. Найти вероятность того, что отклонение измеренного значения от истинного не превзойдет по абсолютной величине 1,6 см при следующих условиях:

1) математическое ожидание ошибки равно 1,2 см (систематическая ошибка в сторону завышения);

2) математическое ожидание равно нулю.

Решение:

Закон распределения ошибки запишем в виде

Теория вероятностей

1.Для этого случая m = 1,2. Тогда искомая вероятность

Теория вероятностей

Пусть Теория вероятностей Тогда

Теория вероятностей

Из таблицы для функции нормального распределения [10, с. 754]
находим

F(0,5) = 0,69146; F(-3,5) = 0,00023 .

Тогда

Р(-1,6<Х<1,6) = 0,69123.

2.При m = 0 искомая вероятность

Теория вероятностей

Введем замену: Теория вероятностейОтсюда

Теория вероятностей

Закон распределения системы случайных величин

Во многих случаях теории вероятностей результат опыта
описывается не одной, а двумя или более случайными величинами. При этом систему двух случайных величин (X, У) можно изображать
случайными точками на плоскости с координатами х и у или
случайными векторами на плоскости. Систему из n случайных величин представляют точками в пространстве n измерений.

Функцией распределения системы двух случайных величин (X, F)
называется вероятность совместного выполнения двух неравенств X<х и Y<у:

F(x,y) = P((X<x)(Y<y)). (11.37)

Геометрический смысл функции распределения двух случайных
величин состоит в том, что эта функция дает вероятность попадания
случайной точки (X, У) в бесконечный квадрант с вершиной в точке (х, у), лежащей левее и ниже ее (рис. 11.8).

Теория вероятностей

Основные свойства функции распределения системы из двух
случайных величин:

1.Функция распределения F(x,y) есть неубывающая функция
своих аргументов, т.е.

■ при Теория вероятностей имеем Теория вероятностей

■ при Теория вероятностей имеем Теория вероятностей

2.Повсюду на Теория вероятностей функция распределения равна нулю

Теория вероятностей

3.При одном из аргументов, равном Теория вероятностей, функция распределения системы превращается в функцию распределения случайной
величины, соответствующей другому аргументу:

Теория вероятностей

где Теория вероятностей и Теория вероятностей — функции распределения случайных величин X и Y соответственно.

4.Если оба аргумента равны Теория вероятностей, то функция распределения
системы равна единице:

Теория вероятностей

Найдем вероятность попадания случайной точки в
прямоугольник D с координатами вершин (а, b), (а, с), (d, с), (d, b)
(рис. 11.9)

Теория вероятностей

Условимся включать в прямоугольник D его нижнюю и левую
границы (полужирные линии) и не включать верхнюю и правую.
Выразим вероятность попадания случайной точки (X, Y) в
прямоугольник D через функцию распределения системы. Для этого
рассмотрим на плоскости хОу четыре бесконечных квадранта с
вершинами в точках (а, b), (а, с), (d, с), (d, b). Вероятность попадания в прямоугольник R равна вероятности попадания в квадрант
(d, с) минус вероятность попадания в квадрант (а, с) минус
вероятность попадания в квадрант (d, b) плюс вероятность попадания в квадрант (а, b), так как мы дважды вычли вероятность попадания в этот квадрант. Таким образом,

Теория вероятностей

Пусть функция распределения системы двух случайных величин F(x, у) непрерывна и дифференцируема.

Плотностью распределения f(x, у) системы называется вторая смешанная производная от функции распределения:

Теория вероятностей

Геометрически плотность распределения системы двух случайных величин можно изобразить в виде поверхности в трехмерной системе координат, называемой поверхностью распределения. Линии уровня поверхности распределения называются кривыми равной плотности.

Вероятность попадания случайной точки системы двух случайных величин в произвольную область D, расположенную на плоскости хОу, может быть выражена при помощи плотности распределения. В этом случае вероятность попадания равна объему цилиндрического тела, ограниченного сверху поверхностью распределения и опирающегося на область D . Таким образом,

Теория вероятностей

Вероятность попадания точки в прямоугольную область (рис. 11.10) может быть представлена в виде

Теория вероятностей

Теория вероятностей

Из формулы (11.39) вытекает соотношение для функции распределения системы двух случайных величин F(x,y) через плотность распределения f(x,y):

Теория вероятностей

Так как попадание точки на поверхность всей плоскости хОу
является достоверным событием, то

Теория вероятностей

В соответствии с третьим свойством функции распределения
двух случайных величин функция распределения каждой из
величин, входящих в систему, определяется соотношением:

Теория вероятностей

Дифференцируя по х первое выражение и по у второе,
получим соотношения для плотности распределения случайных величин X и Y:

Теория вероятностей

Таким образом, зная плотность распределения системы двух
случайных величин, можно получить плотность распределения
каждой из них. Решить обратную задачу, т.е. по плотности
распределения каждой из двух случайных величин найти плотность
распределения системы этих величин, в общем случае нельзя. Для решения обратной задачи нужно знать также зависимость между
величинами, входящими в систему. Эта зависимость может быть
охарактеризована с помощью условного закона распределения.

Условным законом распределения случайной величины X, входящей в систему (X, Y), называется закон ее распределения, вычисленный при условии, что другая случайная величина Y приняла определенное значение у.

Условные закон распределения и плотность распределения
обозначаются соответственно F(x|y) и f(x|y).

Плотность распределения системы двух случайных величин равна
плотности распределения одной из величин, входящих в систему,
умноженной на условную плотность распределения другой величины, вычисленной при условии, что первая величина приняла заданное значение:

Теория вероятностей

При изучении системы случайных величин важно определить
степень и характер их зависимости.

Случайные величины X и Y называются независимыми, если
закон распределения каждой из них не зависит от того, какое
значение приняла другая, т.е. Теория вероятностей Отсюда следует, что

Теория вероятностей

Таким образом, если плотность распределения системы двух
случайных величин распадается на произведение двух функций, то
эти величины независимые.

Вероятностная зависимость может быть более или менее тесной.
По мере увеличения тесноты вероятностной зависимости она все
более приближается к функциональной.

Аналогично моментам, введенным для одной случайной
величины, вводятся моменты для системы случайных величин.

Начальным моментом порядка k, s системы (X, Y) называется
математическое ожидание произведения Теория вероятностей

Теория вероятностей

Центральным моментом порядка k, s системы (X, Y) называется математическое ожидание произведения степени к и s соответствующих центрированных величин:

Теория вероятностей

Для непосредственного подсчета моментов дискретных случайных величин используются формулы:

Теория вероятностей

где Теория вероятностей — вероятность того, что система
случайных величин (X, Y) примет значение Теория вероятностейСуммирование
распространяется по всем возможным значениям случайных величин X и Y.

Для непрерывных случайных величин

Теория вероятностей

На практике обычно применяются только первые и вторые моменты. Первые начальные моменты являются математическими ожиданиями случайных величин X и Y, входящих в систему:

Теория вероятностей

Точка с координатами Теория вероятностей называется центром совместного
распределения вероятностей (центром рассеяния).

Второй смешанный центральный момент называется корреляционным моментом, или ковариацией случайных величин Х и Y :

Теория вероятностей

Остальные вторые центральные моменты представляют собой
дисперсии случайных величин X и Y:

Теория вероятностей

Корреляционный момент описывает связь между случайными
величинами и их рассеяние. Для независимых случайных величин
корреляционный момент равен нулю.

Нормированное значение корреляционного момента называется
коэффициентом корреляции

Теория вероятностей

Здесь Теория вероятностей и Теория вероятностей — средние квадратические
отклонения случайных величин X и Y. Коэффициент корреляции может принимать значения в пределах

Теория вероятностей

Примерами корреляционной связи между случайными величинами могут служить рост и вес человека (положительная корреляция) или время, потраченное на регулировку прибора при подготовке его к работе, и количество неисправностей, обнаруженных при работе данного прибора (отрицательная корреляция).

На рис. 11.11 показан пример экспериментально полученных пар значений двух случайных величин X и Y, имеющих отрицательную корреляцию, а на рис. 11.12 — случай практически некоррелированных случайных величин.

Теория вероятностей

Плотность нормального распределения двух случайных величин выражается соотношением

Теория вероятностей

Здесь Теория вероятностей являются математическими
ожиданиями, средними квадратическими отклонениями и
коэффициентом корреляции случайных величин X и Y соответственно.

Для некоррелированных случайных величин X и Y, у которых
r = 0, плотность распределения (11.53) принимает вид

Теория вероятностей

На практике часто встречаются системы, состоящие из n
случайных величин. Полной характеристикой такой системы служит
закон распределения, который задается функцией распределения
или плотностью распределения.

Функцией распределения системы из n случайных величин
Теория вероятностей называется вероятность совместного выполнения n неравенств вида Теория вероятностей

Теория вероятностей

Плотностью распределения системы n непрерывных случайных величин называется n-я смешанная частная производная от функции распределения, взятая один раз по каждому аргументу

Теория вероятностей

Функция распределения частной системы случайных величин Теория вероятностейвыделенный из системы Теория вероятностей имеет вид

Теория вероятностей

Плотность распределения каждой из случайных величин
системы может быть определена из плотности распределения системы.

Например,

Теория вероятностей

Плотность распределения частной системы случайных величин Теория вероятностей

Теория вероятностей

Условным законом распределения частной системы случайных
величин
Теория вероятностей называется закон ее распределения при условии, что остальные величины приняли конкретные значения Теория вероятностей

Теория вероятностей

Случайные величины называются независимыми, если закон
распределения каждой случайной величины не зависит от того,
какие значения приняли остальные случайные величины. Плотность
распределения системы независимых случайных величин равна
произведению плотностей распределений отдельных случайных
величин:

Теория вероятностей

Вероятность попадания случайной точки системы случайных
величин Теория вероятностей в пределы n-мерной области D
выражается n-кратным интегралом:

Теория вероятностей

Для неполного описания системы из п случайных величин
вместо закона распределения используются числовые характеристики, к числу которых относятся:

■ n математических ожиданий Теория вероятностей

■ n дисперсией Теория вероятностей

■ n(n-1) корреляционных моментов Теория вероятностей

Дисперсия является частным случаем корреляционного момента
при i = j. Действительно,

Теория вероятностей

Корреляционные моменты и дисперсии часто представляют в
виде матрицы корреляционных моментов, состоящей из Теория вероятностей элементов:

Теория вероятностей

Так как Теория вероятностей то элементы корреляционной матрицы
расположены симметрично по отношению к главной диагонали.

Иногда вместо матрицы корреляционных моментов (11.57)
используется нормированная корреляционная матрица, или матрица коэффициентов корреляции:

Теория вероятностей

Теория случайных процессов

Случайным процессом называется случайная функция X(t) от
независимой переменной t .

Случайная функция в результате опыта может принимать тот
или иной конкретный вид, неизвестный заранее.

Конкретный вид, принимаемый случайной функцией в результате опыта, называется реализацией случайной функции.

При проведении над случайным процессом серии опытов
получим группу, или семейство, реализаций. Переменная t может быть как дискретной, так и непрерывной величиной.

Примером случайного процесса является цена конкретной
ценной бумаги на бирже в зависимости от времени в течение торговой сессии длительностью Т . Обозначим этот процесс через S(t), а его отдельные реализации — через Теория вероятностейНа графике
эти реализации приняли вид, показанный на рис. 11.13.

Теория вероятностей

Аргументом случайных функций могут быть не только время,
но и другие величины. Например, в высотном профиле
температуры воздуха аргументом является высота.

На практике часто встречаются случайные процессы, зависящие не
от одного, а от нескольких аргументов (например, зависимость
температуры воздуха от высоты и географических координат).

Для случайной величины Теория вероятностей представляющей сечение
случайной функции для конкретного значения аргумента Теория вероятностей может
быть определен закон распределения. Например, плотность
распределения этого сечения представляется функцией Теория вероятностей
Плотность распределения Теория вероятностей для конкретного значения аргумента Теория вероятностей не является исчерпывающей характеристикой случайного процесса X(t). Наиболее полно этот процесс описывается двухмерной плотностью распределения Теория вероятностей где Теория вероятностей и Теория вероятностей — два конкретных значения аргумента t, представляющих собой два сечения случайного процессах X(t). Для n сечений плотность распределения приобретает вид

Теория вероятностей

Для анализа случайных процессов чаще всего используются не
законы распределения, а отдельные характеристики этих процессов.

Математическим ожиданием случайной функции X(t) называется
неслучайная функция

Теория вероятностей

которая при каждом значении аргумента t равна математическому
ожиданию соответствующего сечения случайной функции.

Дисперсией случайной функции X(t) называется неслучайная функция

Теория вероятностей

значение которой при каждом значении аргумента t равно дисперсии
соответствующего сечения случайной функции.

Среднее квадратичное отклонение и дисперсия случайной
функции связаны соотношением

Теория вероятностей

Теория вероятностей

На рис. 11.14, 11.15 представлены две случайные функции
Теория вероятностей и Теория вероятностей с одинаковыми математическими ожиданиями
(кривая а). Однако характер этих функций совершенно различен. Для
более подробного их описания используют корреляционную
(автокорреляционную) функцию, характеризующую степень зависимости между сечениями случайных функций.

Корреляционной функцией случайного процесса X(t) называют
неслучайную функцию двух аргументов Теория вероятностей которая при каждой паре значений t, t’ равна корреляционному моменту соответствующих сечений случайного процесса:

Теория вероятностей

Как следует из вида реализаций случайных процессов Теория вероятностей и
Теория вероятностей рис. 11.14, 11.15, имеющих одинаковые математические
ожидания, их корреляционные функции различаются. Для процесса
рис. 11.14 корреляционная функция убывает медленно, по мере
увеличения промежутка (t, t’), а для процесса рис. 11.15
корреляционная функция убывает быстрее.

При t = t’ корреляционная функция обращается в дисперсию.

Действительно,

Теория вероятностей

Корреляционная функция симметрична относительно своих
аргументов:

Теория вероятностей

Нормированной корреляционной функцией называется отношение

Теория вероятностей

При t=t’ имеем

Теория вероятностей

Случайная функция называется стационарной, если все ее вероятностные характеристики не зависят от аргумента t. Таким образом, для стационарной случайной функции имеем

Теория вероятностей

Корреляционный момент стационарной случайной функции не зависит от того, где на оси абсцисс взят промежуток (t, t’), а зависит только от длины этого отрезка. Поэтому в выражении (11.61) вместо переменной t’ можно положить Теория вероятностей Тогда обозначение корреляционного момента принимает вид Теория вероятностей Так как значение корреляционной функции стационарного процесса не зависит от t, то ее можно представить как функцию одного аргумента:

Теория вероятностей

Дисперсия стационарной случайной функции определяется соотношением

Теория вероятностей

Нормированная корреляционная функция

Теория вероятностей

При Теория вероятностей имеем

Теория вероятностей

Для определения характеристик реальных случайных процессов используются, как правило, опытные данные в виде набора реализаций. Однако существуют стационарные случайные процессы, все характеристики которых можно определить по одной достаточно длинной реализации. Про такие процессы говорят, что они обладают эргодическим свойством. Таким образом, математическое ожидание, дисперсия, корреляционная функция эргодического стационарного процесса определяются по одной реализации.

Математическое ожидание эргодического стационарного случайного процесса по одной реализации x(t) на промежутке (0, T) рассчитывается по формуле

Теория вероятностей

Корреляционная функция эргодического стационарного случайного процесса по одной реализации x(t) на промежутке (0, T) определяется соотношением

Теория вероятностей

где Теория вероятностей

На практике интегралы (11.70) и (11.71) обычно заменяют конечными суммами. Для этих целей промежуток (0, Т) разбивают на n равных частей длиной Теория вероятностей и середины полученных отрезков обозначают через Теория вероятностей (рис. 11.16). Математическое ожидание (11.70), т.е. площадь под кривой x(t), может быть представлено как сумма площадей элементарных фигур с площадью Теория вероятностей

Теория вероятностей

Теория вероятностей

При вычислении корреляционной функции придаем аргументу Теория вероятностей значения

Теория вероятностей

где j = 1, 2,…, n. Так как Теория вероятностейто (11.71) можно записать в виде суммы:

Теория вероятностей

Пример корреляционной функции, построенной по точкам, приведен на рис. 11.17.

Теория вероятностей

Вычисления корреляционной функции проводят до тех пор,
пока она не станет практически равной нулю.

Основные определения вероятности

В отличие от математических дисциплин, изучающих
“точные” закономерности, предметом теории вероятностей являются специфические закономерности, наблюдаемые при
анализе случайных явлений. Эти закономерности проявляются в
массовых явлениях, и позволяют предсказывать с той или иной
вероятностью исход испытаний. Тогда как, в единичном случае
можно только предположить исход события.

Мы можем наблюдать широкий круг явлений, когда при
многократном осуществлении комплекса условий Теория вероятностей доля той
части случаев, когда событие А происходит, лишь изредка уклоняется сколько-нибудь значительно от некоторой средней цифры, которая таким образом может служить характерным показателем массовой операции (многократного повторения комплекса Теория вероятностей) по отношению к событию А. Закономерности этого рода называются вероятностными или стохастическими закономерностями.

Итак, имеется схема для различных событий, наступающих при неизменном комплексе условий: достоверное – случайное – невозможное. Ясно, что большая часть событий в мире находится между достоверностью и невозможностью (интуитивное понимание!).

По мере развития теории вероятностей, а также областей её приложения, развивались и представления об основном понятии этой теории – вероятности.

В настоящее время существует четыре подхода к определению вероятности:

  1. Определение математической вероятности как количественной меры “степени уверенности” познающего объекта.
  2. Определения, сводящие понятие вероятности к понятию “равновозможности” как к более примитивному понятию
    (так называемое “классическое” определение вероятности).
  3. Определения, основанные на “частоте” появления события в большом количестве испытаний (“статистическое”
    определение).
  4. Аксиоматический подход, на основе теории множеств,
    формализующий теорию вероятностей.

Вероятностью события P(А) называют отношение числа
благоприятных исходов испытания m к общему числу всех
равновозможных несовместных элементарных исходов n:

Теория вероятностей

Это определение вероятности базируется на классическом
подходе и часто применяются для решения конкретных задач,
поэтому мы часто будем обращаться к нему далее. Остановимся
подробнее на аксиоматическом подходе к определению вероятности события.

Аксиоматическое определение вероятности

Прежде, чем рассмотреть вероятность с указанной позиции,
вспомним, что аксиома – это исходное утверждение какой-либо
научной теории, которое берется в качестве недоказуемого, и из
которого выводятся все остальные предложения теории по принятым в ней правилам вывода.

Построение аксиом теории вероятностей А.Н. Колмогоровым означало переход от полуэмпирического, интуитивного понимания вероятности к строгому формализованному. Для введения аксиом нам необходимо принять следующие соглашения.

Зафиксируем комплекс условий Теория вероятностей и рассмотрим некоторую систему S событий А, В, С, …, каждое из которых должно при
каждом осуществлении комплекса Теория вероятностей произойти или не произойти. Далее введём соглашения, которые, как увидит
внимательный читатель, являются соглашениями теории множеств и математической логики.

1) Событие, состоящее в наступлении обоих событий А и В,
будем называть произведением событий А и В и обозначать
АВ (или Теория вероятностей).
2) Событие, состоящее в наступлении хотя бы одного из событий А и В, будем называть суммой событий А и В и обозначать А+В (или Теория вероятностей).
3) Событие, состоящее в том, что событие А происходит, а событие В не происходит, будем называть разностью событий А и В и обозначать А – В.
4) Если при каждом осуществлении комплекса условий Теория вероятностей, при
котором происходит событие А, происходит и событие В,
то мы будем говорить, что А влечет за собой В, и обозначать это символом Теория вероятностей или Теория вероятностей
5) Если А влечет за собой В и в то же время В влечет за собой
А, то есть если при каждой реализации комплекса условий Теория вероятностей
события А и В оба наступают или оба не наступают, то
мы будем говорить, что события А и В равносильны, и
обозначим это А=В. Равносильные события могут заменять
друг друга или, по-другому, они тождественны.
6) Два события Теория вероятностей и Теория вероятностейназываются противоположными,
если для них одновременно выполняются два соотношения:
7) Теория вероятностей (достоверное событие),
Теория вероятностей (невозможное событие)

Пусть U – достоверное событие. Все достоверные события
равносильны между собой.
V – невозможное событие. Все невозможные события тоже
равносильны между собой.
8) Два события А и В называются несовместимыми, если их
совместное появление невозможно, то есть если А*В = V.

Если Теория вероятностей и события Теория вероятностейпопарно несовместимы,
то есть Теория вероятностейпри Теория вероятностейто говорят, то событие А подразделяется на частные случаи Теория вероятностейНапример, при бросании игральной кости событие С, состоящее в выпадении четного числа очков, подразделяется на частные случаи Теория вероятностейсостоящие соответственно в выпадении 2, 4 и 6 очков.
События Теория вероятностейобразуют полную группу событий, если хотя бы одно из них непременно должно произойти (при каждом осуществлении комплекса Теория вероятностей), то есть если Теория вероятностейТеория вероятностей

Пример:

В порту имеется два причала для приема судов.
Можно рассмотреть три события: Теория вероятностей – отсутствие судов у
причалов, Теория вероятностей– присутствие одного судна у одного из причалов,
Теория вероятностей– присутствие двух судов у двух причалов. Эти три события
образуют полную группу.

В каждой задаче теории вероятностей приходится иметь дело с каким-либо определенным комплексом условий Теория вероятностей и с какой-либо определенной системой S событий, наступающих или не наступающих после каждой реализации комплекса условий Теория вероятностей. Относительно этой системы целесообразно сделать следующие допущения:
а) если системе S принадлежат события А и В, то ей принадлежат также события АВ, А+В, А-В (замкнутость относительно операций);
б) система S содержит достоверное и невозможное события (“единица” и “ноль” в замкнутой системе).
Система событий, удовлетворяющая этим допущениям
(1- 9), называется полем событий.

Всегда можно выделить такие события, которые не могут
быть разложены на более простые: выпадение определенной
грани при бросании игральной кости, попадание в определенную точку квадрата при рассмотрении диаграммы Венна (рис.1).
Назовем такие неразложимые события – элементарными
событиями.

Теория вероятностей

Для построения математической теории вероятностей требуется дополнительная формализация.

Введем понятие – пространство элементарных событий,
которое состоит из множества всех возможных элементарных
событий. Элементами пространства могут быть точки евклидова
пространства, функции одной или нескольких переменных и
т. д. Множество точек пространства элементарных событий образуют случайные события. Имеется в виду любая доступная комбинация из элементарных событий, полученная в результате легальных операций. Событие, состоящее из всех точек пространства элементарных событий, называется достоверным событием.

Для пространства элементарных событий, определенных
выше указанным способом, имеют место следующие законы,
пришедшие из алгебры (табл. 1).

Теория вероятностей

Столь долгая процедура потребовалась, чтобы перейти на
язык теории множеств и формальной алгебры. Следующим шагом будет выделение условий для ввода аксиом теории вероятностей.

Пусть задано некоторое множество Теория вероятностей. Элементы этого множества называются элементарными событиями. Предположим, что фиксирована некоторая система подмножеств множества Теория вероятностей; эти подмножества названы просто событиями. События обозначаются А, В, С и так далее. При этом потребуем, что:

Множество Теория вероятностей называют пространством элементарных событий.
Два события А и В, не имеющие (как два подмножества) общих элементов, называются несовместными.
События Теория вероятностей и Теория вероятностей называются противоположными.
Событие Теория вероятностей называется достоверным, событие Теория вероятностей (то есть пустое множество) – невозможным.

Аксиомы Колмогорова, задающие понятие вероятности:

Аксиома:

Каждому событию А поставлено в соответствие
неотрицательное число р(А), называемое вероятностью события А.
Аксиома:

Если события Теория вероятностей попарно несовместны, то вероятность наступления хотя бы одного из них равна сумме
вероятностей этих событий.

Теория вероятностей

Для случая, когда пространство Теория вероятностей конечно (аксиома 2), может быть заменено более слабым требованием:
р(А+В) = р(А) + р(В) , если А и В несовместны.

Аксиома:

Теория вероятностейВероятность полной группы событий равна 1.

Примеры полной группы событий

Приобретены два билета денежно-вещевой лотереи. Обязательно произойдет одно и только одно из следующих событий: «выигрыш выпал на первый билет и не выпал на второй», «выигрыш не выпал на первый билет и выпал на второй», «выигрыш выпал на оба билета», «на оба билета выигрыш не выпал». Эти события образуют полную группу попарно несовместных событий.

Стрелок произвел выстрел по цели. Обязательно произойдет одно из следующих двух событий: попадание, промах. Эти два несовместных события образуют полную группу. Все теоремы теории вероятностей выводятся из аксиом 1-3.

Задача:

Из колоды игральных карт, содержащей 36 листов, наугад выбирается одна карта. Найти вероятность того, что:
а) карта окажется красной масти; б) карта окажется картинкой;
в) карта окажется дамой; г) эта карта туз буби.

Решение:
а) n = 36, m = 18; P(A) = 18/36=1/2.
б) n = 36, m = 16; P(A) = 16/36=4/9.
в) n = 36 , m = 4; P(A) = 4/36=1/9.
г) n = 36, m = 1; P(A) = 1/36.

Задача:

Пусть вероятность того, что студент получит на экзамене по статистике «пятерку» равна 0,17, «четверку» – 0,38,
«тройку» – 0,32, а «двойку» – 0,13. Найти вероятность того, что
очередной студент получит оценку, не меньше тройки.

Решение:

Искомое событие D произойдет, если будет получена оценка 5 (событие А), оценка 4 (событие В), или оценка 3 (событие С), то есть событие D есть сумма событий А, В, С. События А, В и С несовместимы. Поэтому, применяя теорему сложения вероятностей, получим:
P(D) = P(A+B+C) = P(A) + P(B)+ P(C) = 0,17 + 0,38 + 0,32= 0,87.

Задача:

Испытатель проводит опыты с пирамидкой, подбрасывая ее и определяя какая грань выпадет при очередном испытании. В результате опытов были определены вероятности выпадения каждой из четырех граней: 1/3, 1/6, 1/3, 1/6. Определите вероятность полной группы событий.

Решение:

Вероятность полной группы событий определяется как сумма вероятностей всех элементарных исходов данной группы:

Теория вероятностей

Следствия из аксиом 1-3:

Следствие:

Вероятность достоверного события равна
единице. Действительно, если событие достоверно, то каждый
исход испытания благоприятствует событию, то есть m=n, а значит и его вероятность P(A)=m/n=1.

Следствие:

Вероятность невозможного события равна нулю. Раз ни один из исходов испытания не благоприятствует событию, то m=0, а тогда P(A)=m/n=0.

Следствие:

Вероятность случайного события есть положительное число, заключенное между 0 и единицей. Поскольку случайному событию благоприятствует лишь часть из общего числа элементарных исходов испытания. То есть 0<m<n, а значит 0< m/n<1.

Теория вероятностей

Таким образом, введя аксиоматическое понятие вероятности, мы получили возможность использовать для доказательства теорем и следствий аппарат формальной логики.

Классическая и статистическая модели вероятности

Цель: Сформировать у студентов понятие «вероятности» с точки зрения классического и статистического подходов, познакомить с методами решения задач.

Основные теоремы классической теории вероятностей

Рассмотрим вероятность с тех же позиций, с каких она исследовалась несколько веков математиками.

Классический подход тоже имеет дело с событием, как основным понятием теории вероятностей. Выделяется достоверное, невозможное и случайное события. Однако, если при построении аксиоматической теории единственным ограничением для событий, составляющих поле, было требование достоверности всех событий (т.е. по крайней мере, одно их них, должно произойти при выполнении заданного комплекса условий Теория вероятностей), то классический подход основан на дополнительном требовании – равновозможности всех событий в выбранном поле событий.

Равновозможность означает равноправность (симметрию)
отдельных исходов испытания относительно некоторого комплекса условий.

Например, имея колоду игральных карт из 36 листов, можно вытащить из нее любую карту. Вероятность вытащить каждую
из 36 карт одинакова и равна 1/36. Это событие является равновозможным и несовместимым с другими. Требование равновозможности является жестким ограничением и сужает круг задач, которые можно решить при помощи классического подхода.

Вероятность события А равна отношению числа случаев m,
благоприятствующих этому событию, к общему числу единственно возможных, равновозможных и несовместимых исходов испытания n:

P(A) = m/n.

Пример:

Брошены две игральные кости (кубики с нумерованными гранями). Найти вероятность, что сумма очков на выпавших гранях – четная, причем на грани хотя бы одной из костей появится шестерка.

Решение:

На выпавшей грани “первой” кости может появиться одно очко, два очка, …, шесть очков. Аналогичные шесть
элементарных исходов возможны при бросании “второй” кости.
Каждый из исходов бросания “первой” кости может сочетаться с
каждым из исходов бросания “второй”. Таким образом, общее
число возможных элементарных исходов испытания равно 6*6 = 36 Эти исходы единственно возможны и, в силу симметрии костей, равновозможны.

Элементарное событие – событие, которое не может быть разложено на составляющие события.

Благоприятными относительно интересующего нас события
(хотя бы на одной грани появится шестерка и сумма выпавших
очков – четная) являются следующие пять исходов:

6, 2; 6+2 = 8 2, 6; 2+6 = 8
6, 4; 6+4 = 10 4, 6; 4+6 = 10
6, 6; 6+6 = 12

Первым записано число очков, выпавших на “первой” кости, вторым – число очков, выпавших на “второй” кости, а затем указана сумма очков.
Искомая вероятность равна отношению числа исходов,
благоприятствующих событию, к числу всех исходов:

P(A) = 5/36.

Если теперь обратиться к аксиоматическому определению
вероятности и сравнить его с классическим, то мы увидим, что
классическое определение является частным случаем аксиом.
Действительно, если аксиому 1 дополнить требованием равенства введенных чисел Теория вероятностейдруг другу, а их сумму ограничить единицей (аксиома 3), то мы получим классическое определение.

Теоремы классической теории вероятностей

Для эффективного использования вычислений вероятности, основанной на классическом подходе было доказано несколько
теорем и следствий из них. Как видно, эти теоремы перекликаются с теоремами аксиоматического подхода.

Теорема:

Вероятность любого события не может быть отрицательной и больше единицы.

Теорема:

Вероятность достоверного события равна единице.

Теорема:

Вероятность невозможного события равна нулю.

Теорема:

Теорема сложения вероятностей для несовместных событий.
Вероятность появления одного из нескольких несовместных событий равна сумме вероятностей этих событий.

Теория вероятностей

Теорема:

Теорема сложения вероятностей для совместных
событий. Вероятность появления одного из нескольких совместных событий равна сумме вероятностей этих событий минус
вероятность произведения этих событий:

Теория вероятностей

Теорема:

Теорема произведения вероятностей для независимых событий. Вероятность произведения независимых событий равна произведению вероятностей каждого события

Теория вероятностей

Вероятность произведения зависимых событий вычисляется
по формуле условной вероятности.

Задача:

В урне 30 шаров: 10 красных, 5 синих и 15 белых.
Из урны наудачу вытащили один шар. Найти вероятность появления цветного шара.

Решение:

Появление цветного шара обозначает появление или красного, или синего шара. Вероятность появления красного шара событие А: Р(А)=10/30=1/3.
Вероятность выпадения синего шара (событие В):
Р(В)=5/30=1/6.
События А и В несовместны, поэтому применима теорема
сложения вероятностей 4.
P(A+B) = P(A) + P(B) = 1/3 + 1/6 = 1/2.

Задача:

Для отправки груза со склада может быть выделена
только одна из двух машин различного вида. Известны вероятности выделения каждой машины: Теория вероятностейОпределить вероятность того, что к складу будет подана хотя бы одна из этих машин.

Решение:

Поскольку одновременное выделение двух машин для отправки груза – событие невозможное, а это значит, что события «выделение машины первого вида» и «выделение машины второго вида» являются несовместными. Тогда, вероятность того, что к складу будет подана хотя бы одна из этих машин будет:

Теория вероятностей

Задача:

Бросают два кубика с нумерованными гранями. Определить вероятность того, что в очередном испытании хотя бы одна из выпавших граней будет четной.

Решение:

На каждом кубике – шесть граней. Три из них
являются четными, поэтому вероятность выпасть четной грани
на первом кубике P(A)=1/2 и на втором кубике P(B)=1/2. Появление четного количества очков на первой и второй кости – события совместные, поэтому вероятность появления четной грани хотя бы на одной кости определяется по теореме сложения совместных событий:

Теория вероятностей

Вероятность одновременного появления четных граней на обоих костях:

Теория вероятностей

Итак, вероятность появления четной грани хотя бы на одной кости равна

Теория вероятностей

Задача:

По мишени стреляют три стрелка. Вероятности попадания соответственно равны 0,7; 0,8 и 0,9. Найти вероятность того, что попадут все три.

Решение:

Пусть событие А – «попадание в мишень первого стрелка», событие В – «попадание в мишень второго стрелка», а событие С – «попадание в мишень третьего стрелка». Поскольку эти события независимые, то применяя теорему произведения вероятностей, получим:

Теория вероятностей

Задача:

Три стрелка производят по одному выстрелу. Вероятности попадания в цель каждого стрелка равны 0,9; 0,8; 0,85 соответственно. Найти вероятность того, что в цель попадут только два стрелка.

Решение:

Искомое событие произойдет, если случатся такие
события:
D — попадут первый и второй стрелки, третий промажет;
E — попадут первый и третий стрелки, второй промажет;
F — попадут второй и третий стрелки, первый промажет.
Попадания каждого стрелка в цель не зависят друг от друга, поэтому с помощью теоремы умножения вероятностей для независимых
событий можно рассчитать все возможные исходы испытания А, В и С

Вероятность события D:

Теория вероятностей

вероятность события E:

Теория вероятностей

вероятность события F:

Теория вероятностей

События D, E и F несовместимы. Тогда, применяя теорему сложения вероятностей, получим, что вероятность того, что в цель попадут два стрелка

Теория вероятностей

Статистическое определение вероятности

Иной подход к определению вероятности предлагается, если
воспользоваться понятием статистических испытаний, проведенных при условии соблюдения комплекса условий Теория вероятностей и с фиксацией наступления или не наступления интересующего нас события. Поле событий, таким образом, будет ограниченно только
количеством испытаний, а вероятность благоприятного события
определяется post factum, т.е. по фактическому результату, в
предположении, что все исходы – равновероятны. Дополнительным требованием будет требование независимости каждого последующего испытания от предыдущего. Таким образом, статистическая вероятность фактически является относительной частотой интересующего нас события в серии испытаний.

Относительная частота события А или статистическая
вероятность определяется равенством Теория вероятностей , где m – число испытаний, в которых событие А наступило; n – общее число
произведенных испытаний.

Считается, что при достаточно большом количестве испытаний статистическая вероятность (частота) приближается асимптотически к классической вероятности.

Задача:

Игральная кость брошена десять раз. Шесть очков выпало 3 раза. Какова вероятность и частота выпадения грани с шестью очками?

Решение:

Вероятность выпадения шести очков определяется как отношение Р(А)=1/6 (из шести возможных исходов при подбрасывании кости выпадению шестерки благоприятствует
один), а частота выпадения шести очков равна W(A)=3/10 (событие наступило три раза в десяти испытаниях).

Геометрическая вероятность

Пусть Теория вероятностей – некоторая область, имеющая меру Теория вероятностей(длину,
площадь, объем и т. д.), такую, что Теория вероятностей Точка равномерным образом попадает в Теория вероятностей (реализуется принцип геометрической вероятности), если вероятность Р(А) попадания ее в каждую область А, являющейся подобластью Теория вероятностей, пропорциональна мере этой области Теория вероятностей

Проще говоря, геометрическая вероятность определяется
отношением площадей: общей – фигуры, и области, в которую должна попасть точка.

Задача:

В круг вписан правильный шестиугольник. Найти
вероятность того, что точка, наудачу брошенная в круг, не попадет в правильный шестиугольник, вписанный в него.

Решение:

Пусть радиус круга равен R, тогда сторона шестиугольника тоже равна R. При этом площадь круга Теория вероятностей а площадь шестиугольника равна s:

Теория вероятностей

Вероятность искомого события:

Теория вероятностей

Теория вероятностей

Задача:

Два человека договорились встретиться у кинотеатра между 12 и 14 часами и договорились, что тот, кто придет первым, ждет другого в течение 30 минут, после чего уходит. Найти
вероятность их встречи, если приход каждого в течение указанного
времени может произойти в любой момент, а приходы людей не
зависят друг от друга?

Теория вероятностей

Решение:

Обозначим моменты прихода людей через значения x и y на соответствующих осях координат. Пусть Т – интервал времени возможной встречи, равный в примере 120 минутам, а t – время ожидания, равное 30 минутам. Ясно, что для того, чтобы встреча произошла, нужно, чтобы разность между моментами прихода людей была меньше или равна 30 мин.

Изобразим х и у как точки внутри квадрата со сторонами Т. Тогда исходы, благоприятные для встречи, будут соответствовать
заштрихованной области А. Согласно принципу геометрической
вероятности, искомая вероятность равна отношению площади
заштрихованной фигуры к площади всего квадрата:

Теория вероятностей

Элементы комбинаторики в теории вероятностей

Многие задачи, основанные на классическом подходе, и как мы уже знаем, понятии равновозможности, сводятся к вычислению количества всех возможных событий, наступающих при выполнении комплекса условий Теория вероятностей, а также количества интересующих нас благоприятных событий. Такие вычисления достаточно часто сводятся к расчету количества различных комбинаций элементов из какого-либо множества. В этом случае мы имеем дело с разделом математики – комбинаторикой. Введем
основные определения и соотношения комбинаторики.

Пусть имеется k групп элементов, причем i-я группа состоит из Теория вероятностейэлементов. Выберем по одному элементу из каждой группы. Тогда общее число N способов, которыми можно произвести такой выбор, определяется соотношением:

Теория вероятностей – основная формула комбинаторики.

Пусть имеется 4 аудитории, в которых стоят компьютеры. В
первой аудитории Теория вероятностейкомпьютеров, во второй Теория вероятностей
компьютеров, в третьей Теория вероятностейкомпьютеров, а в четвертой Теория вероятностейкомпьютеров. Для расчета количества комбинаций этих компьютеров, взятых из каждого класса по одному, нужно рассчитать

Теория вероятностей

Перестановкой из n элементов называется любой упорядоченный набор этих элементов. Например, перестановки чисел 1, 2, 3: (1,2,3), (1,3,2), (2,1,3), (2,3,1), (3,1,2), (3,2,1).

Теория вероятностей

Размещением из n элементов по m называется любой упорядоченный набор из m различных элементов, выбранных из
общей совокупности в n элементов.
Например, размещения из четырех чисел 1, 2, 3, 4 по два: (1,2), (1,3), (1,4), (2,1), (2,3), (2,4), (3,1), (3,2), (3,4), (4,1), (4,2), (4,3).

Теория вероятностей

Сочетанием из n элементов по m называется любой неупорядоченный набор из m различных элементов, выбранных из общей совокупности в n элементов.
Сочетаниями из четырех чисел 1, 2, 3, 4 по два:
(1,2), (1,3), (1,4), (2,3), (2,4), (3,4).

Теория вероятностей

Задача:

Перестановка. На пяти карточках написаны буквы “п”, “л”, “а” ,“м”, “а”. Карточки перемешиваются и выкладываются в ряд. Найти вероятность того, что образовавшееся слово будет “лампа” (событие А).

Решение:

В соответствии с комбинаторными принципами
для определения общего числа элементарных исходов нужно
подсчитать число упорядоченных наборов из четырех букв. Мы
имеем дело с числом перестановок, поэтому число элементарных исходов n = 5! = 120. Слово “лампа” образует две перестановки, то есть число благоприятных для события А элементарных исходов m = 2. Поэтому P(A) = m/n = 2/120=1/60.

Задача:

Набирая номер телефона, абонент забыл последние две цифры и, помня лишь, что эти цифры различны, набирал их наудачу. Найти вероятность того, что набраны нужные цифры.

Решение:

Обозначим через через В событие – набраны две нужные цифры. Всего можно набрать столько различных цифр, сколько может быть составлено размещений из десяти цифр по две, то есть

Теория вероятностей

Задача:

В коробке сидят 12 котят. Среди них три белых котенка, три черных, три серых и три рыжих. Из коробки наудачу вытаскивают трех котят. Определить вероятность вытащить из коробки трех котят так, чтобы один из них был непременно рыжий, а два других имели различный окрас и не были бы рыжими.

Решение:

Общее число элементарных исходов испытания равно числу сочетаний из двенадцати элементов по три, то есть С из 12 по 3.

Теория вероятностей

Число исходов, благоприятствующих появлению двух котят различного окраса (и не рыжих), равно числу сочетаний из одиннадцати элементов по два, то есть С из 11 по 2.

Теория вероятностей

Искомая вероятность равна отношению числа исходов, благоприятствующих интересующему нас событию, к общему числу возможных элементарных исходов:

Теория вероятностей

Задача:

В партии из 10 деталей 7 стандартных. Найти вероятность того, что среди взятых наудачу шести деталей четыре (4) оказались стандартными.

Решение:

Общее число возможных исходов испытания равно числу способов, которыми можно извлечь 6 деталей из десяти, то есть числу сочетаний из 10 элементов по 6 – количество сочетаний Теория вероятностей

Число благоприятствующих исходов (среди 6 взятых деталей 4 стандартных). Четыре стандартных деталей можно взять из 7 стандартных Теория вероятностейспособами, при этом остальные 6-4=2 должны быть нестандартными. Взять же 2 нестандартные детали из 10-7=3 нестандартных можно Теория вероятностейспособами. Следовательно, число благоприятствующих исходов равно Теория вероятностей.
Искомая вероятность равна отношению числа исходов, благоприятствующих событию, к числу всех элементарных исходов:

Теория вероятностей

Задача:

Известно, что в поступившей партии из 30 швейных машинок 10 имеют внутренний дефект. Определить вероятность того, что из пяти наудачу взятых машинок три окажутся бездефектными.

Решение:

Введем следующие обозначения: N – общее число машинок, n – число бездефектных машинок, m – число отобранных в партию машинок, k – число бездефектных машинок в отобранной партии.

Общее число комбинаций по m машинок (общее число возможных исходов) будет равно числу сочетаний из N элементов по m, т. е. Теория вероятностей. Но в каждой отобранной комбинации должно содержаться по три бездефектные машинки. Число таких комбинаций равно числу сочетаний из n элементов по k, т. е. .Теория вероятностей.

Оставшиеся дефектные машинки (элементы) тоже образуют множество комбинаций, число которых равно числу сочетаний из
N – n элементов по m – k, т. е. Теория вероятностей.

Это значит, что общее число благоприятствующих исходов
определяется произведением

Теория вероятностей.

Теория вероятностей

Подставив в эту формулу численные значения данного примера, получим

Теория вероятностей

Условная вероятность. Полная вероятность. Формула Байеса

Цель: Сформировать у студентов понятия зависимых и
независимых событий, условной и полной вероятности, научить
решать задачи, в которых определяются полная вероятность и
вероятность гипотезы.

При совместном рассмотрении двух случайных событий А и В часто возникает вопрос: насколько связаны эти события друг с другом, в какой мере наступление одного из них влияет на возможность наступления другого?

Событие В называют независимым от события А, если появление события А не изменяет вероятности события В.
Произведением двух событий А и В называют событие АВ, состоящее в совместном появлении (совмещении) этих событий. Например, если А – деталь годная, В – деталь окрашенная, то АВ – деталь годна и окрашена.

Теорема произведения вероятностей:

Вероятность совместного появления нескольких независимых событий равна произведению вероятностей этих событий:

Теория вероятностей

Пример:

Три человека договорились о встрече в определенном месте, в определенный час. Известны вероятности прихода на
встречу каждого человека:

Теория вероятностей

Определить вероятность того, что все три человека придут на
встречу.

Решение:

Вероятность того, что все эти люди придут на встречу, будет определяться по формуле произведения вероятностей независимых событий:

Теория вероятностей

Случайное событие – это событие, которое при осуществлении совокупности условий S может произойти или не произойти. Если при вычислении вероятности события никаких других ограничений, кроме условий S, не налагается, то такую вероятность называют безусловной; если же налагаются и другие дополнительные условия, то вероятность события называют условной. Например, часто вычисляют вероятность события В при дополнительном условии, что произошло событие А. Заметим, что и безусловная вероятность, строго говоря, является условной, поскольку предполагается осуществление условий S.

Условной вероятностью Теория вероятностейназывают вероятность события В, вычисленную в предположении, что событие А уже наступило.
Условная вероятность события В при условии, что событие А уже наступило, по определению, равна

Теория вероятностей

Рассмотрим два события: А и В; пусть вероятности Р (А) и Теория вероятностей
известны. Как найти вероятность совмещения этих событий, т. е. вероятность того, что появится и событие А и событие В? Ответ на этот вопрос дает теорема умножения зависимых друг от друга событий.

Теорема:

Вероятность совместного появления двух событий равна произведению вероятности одного из них на условную вероятность другого, вычисленную в предположении, что первое событие уже наступило:

Теория вероятностей

Доказательство

Применив формулу (1) к событию ВА, получим

Теория вероятностей

или, поскольку событие ВА не отличается от события АВ,

Теория вероятностей

Сравнивая формулы (1) и (2), заключаем о справедливости равенства

Теория вероятностей

Пример:

У продавца имеется 3 красных воздушных шарика и 7 синих шариков. Продавец взял наугад один шарик из мешка, а затем второй. Найти вероятность того, что первый из взятых шариков – красный, а второй – синий.

Решение:

Вероятность того, что первый шарик окажется красным
(событие A), Р (А) = 3/10. Вероятность того, что второй шарик окажется синим (событие В), вычисленная в предположении, что первый шарик – красный, т. е. условная вероятность Теория вероятностей

По теореме умножения, искомая вероятность

Теория вероятностей

Заметим, что сохранив обозначения, легко найдем:

Теория вероятностей

Теория вероятностей

что наглядно иллюстрирует справедливость равенства (3).

Пример:

В урне 5 белых, 4 зеленых и 3 синих кубика. Каждое испытание состоит в том, что наудачу извлекают один кубик, не возвращая его обратно. Найти вероятность того, что при первом испытании появится белый кубик (событие A), при втором – зеленый (событие В) и при третьем – синий (событие С).

Решение:

Вероятность появления белого кубика в первом испытании
Р (A) = 5/12.

Вероятность появления зеленого кубика во втором испытании, вычисленная в предположении, что в первом испытании появился белый кубик, т. е. условная вероятность

Теория вероятностей

Вероятность появления синего кубика в третьем испытании, вычисленная в предположении, что в первом испытании
появился белый кубик, а во втором – зеленый, т. е. условная вероятность

Теория вероятностей

Искомая вероятность:

Теория вероятностей

Формула полной вероятности

Предположим, что событие А может наступить только вместе с одним из нескольких попарно несовместных событий

Теория вероятностей

Условимся называть эти события по отношению к А гипотезами.
Общая гипотеза – это научно обоснованное предположение о законах и закономерностях природных и общественных явлений, а также закономерностях психической деятельности человека. Приведем примеры некоторых гипотез.

  • Гипотеза Демокрита: «Вещество состоит из атомов».
  • Гипотеза биохимической эволюции: «Жизнь это результат длительной эволюции углеродных соединений» (Авторы биохимик А. И. Опарин в 1924 г. и Дж. Холдейном в 1929 г.).
  • Гипотеза большого взрыва: «Наша вселенная произошла около 13 млрд лет назад в результате взрыва чрезвычайно плотного и горячего вещества – космологической сингулярности».

Для определения полной вероятности события используются вероятности гипотез и условные вероятности событий.

Формула полной вероятности:

Теория вероятностей

Вероятность события А равна сумме произведений условных вероятностей этого события по каждой из гипотез на вероятность самих гипотез.

Пример:

Имеются три урны. В первой находятся 5 белых и 3 черных шара, во второй – 4 белых и 4 черных, в третьей – 8 белых. Наугад выбирается одна из урн и из нее вытаскивается один шар. Какова вероятность того, что он окажется черным (событие А)?

Решение:

Шар может быть вытащен из первой урны, либо из второй, либо из третьей; обозначим эти события Теория вероятностей

Так как имеются одинаковые шансы выбрать любую из урн, то

Теория вероятностей

Далее находим вероятности события А при каждом из условий Теория вероятностей

Теория вероятностей

Во многих задачах на полную вероятность, рассматриваемый опыт можно представить происходящим в два этапа; гипотезы Теория вероятностей исчерпывают все возможные предположения
относительно исхода первого этапа, событие же А есть один их
возможных исходов второго этапа. В рассмотренном примере
первый этап заключался в выборе урны, второй – в извлечении
из нее шара.

Формула Т. Байеса

Формула священника и математика Томаса Байеса была опубликована в 1764 г. Ученые и религиозные деятели того времени искали ответы на сложнейшие вопросы мироздания. Среди них: возникновение и устройство космоса, эволюция, появление человека – на многие трудные вопросы должен был быть найден математический ответ. Результатом таких исследований и стала
формула Т. Байеса, которая используется в теории вероятностей до сих пор.

Формула Байеса относится к той же ситуации, что и формула полной вероятности, но определяет вероятность гипотезы, которая привела к наступлению события. Событие А может наступить только вместе с одним из попарно несовместных событий Теория вероятностей

Пусть произведен опыт, в результате которого произошло событие А. Сам по себе этот факт еще не позволяет сказать, какое из событий Теория вероятностей имело место в проделанном опыте. Поставим задачу: найти вероятности Теория вероятностейкаждой из гипотез в предположении что событие А наступило. Эта задача решается при помощи формулы Байеса.
В примере из предыдущего раздела вероятность гипотезы Теория вероятностей – шар извлечен из третьей урны – до того, как произведен опыт, равнялась 1/3. Однако, если опыт произведен и наступило событие А – вытащенный шар оказался черным, то это снижает шансы гипотезы Теория вероятностей до нуля. Послеопытная, “апостериорная” вероятность гипотезы Теория вероятностей будет в данном случае ниже, чем доопытная, “априорная”.

Теория вероятностей

– формула Байеса.

Вывод формулы весьма прост – из формулы полной вероятности.

Теория вероятностей

или, если воспользоваться формулой полной вероятности,

Теория вероятностей

Пример:

В студенческом стройотряде 2 бригады первокурсников и одна – второкурсников. В каждой бригаде первокурсников 5 юношей и 3 девушки, а в бригаде второкурсников 4 юношей и 4 девушки. По жеребьевке из отряда выбрали одну из бригад и из нее одного человека для поездки в город. а) Какова вероятность того, что выбран юноша? б) Выбранный человек оказался юношей. Какова вероятность, что он первокурсник?

Решение:

Обозначим через А событие – выбор группы студентов для поездки в город. Можно выдвинуть две гипотезы:
Н1 – выбрана группа первокурсников, а поскольку первокурсников 2 группы, то Р(Н1) = 2/3;
Н2 – выбрана группа второкурсников, причем Р(Н2) = 1/3.
Условная вероятность того, что выбранный человек является юношей первокурсником: Р(А/Н1) = 5/8.
Условная вероятность того, что выбранный человек является юношей второкурсником: Р(A/Н2) = 1/2.
Вероятность того, что наудачу выбранный человек – юноша определяется по формуле полной вероятности:
Р(А) = Р(A/Н1)Р(Н1) + Р(A/Н2)Р(Н2) = 5/8*2/3 + 1/2 *1/3 =7/12.
Искомая вероятность того, что выбранный человек – юноша с первого курса определяется по формуле Байеса:

Теория вероятностей

Распределения дискретных случайных величин

Цель: получить представление о случайных величинах,
освоить законы распределения дискретных случайных величин.

Все процессы, происходящие в природе, делятся на непрерывные и дискретные. Примерами непрерывных процессов являются различные природные объекты и их свойства: температура, давление и влажность воздуха, объекты технологических производственных процессов: давление и температура теплоносителя в ядерном реакторе.

В определенный момент времени непрерывная случайная величина может быть выражена в численной форме, но впоследствии это значение будет непрерывно изменяться.
Дискретными являются сигналы тревоги, языковые сообщения в виде звука и письма, жесты и т.п. Например, такие величины, как количество человек в студенческой группе, число солнечных дней в году, высота горы, уровень интеллекта являются дискретными величинами, потому что имеют конкретный количественный признак, который некоторое время не изменяется.

Для обработки непрерывной и дискретной информации существуют и разные вычислительные машины: аналоговые и цифровые. Аналоговые машины следят за постоянно изменяющимся аналоговым сигналом. Например, в отделениях интенсивной терапии такие ЭВМ могут измерять давление, снимать кардиограмму и др.

Рассмотрим вероятностное пространство Теория вероятностей то есть пространство элементарных исходов Теория вероятностей, Теория вероятностей-алгебру событий (определенную нами на пространстве путем введения замкнутых операций), вероятность Р (как меру нашего множества). Множества вида Теория вероятностей являются событиями.

Случайной называют величину, которая в результате испытания примет одно и только одно возможное значение, заранее не известное и зависящее от случайных причин, которые не могут быть заранее учтены.

Например, число родившихся мальчиков среди 100 новорожденных есть случайная величина, которая имеет возможные значения: 0,1,2,3…100.
Расстояние, которое пролетит снаряд при выстреле орудия – есть случайная величина, которая зависит от прицела, силы и
направления ветра, температуры воздуха. Возможные значения
этой величины принадлежат промежутку (a,b).
Далее будем обозначать случайные величины прописными
буквами Х, Y, Z, а их возможные значения x,y,z. Например случайная величина Х имеет три возможных значения Теория вероятностей

Случайной величиной Теория вероятностейназывается произвольная функция,
ставящая в соответствие каждому элементарному исходу (событию)
Теория вероятностейчисло Теория вероятностей

Так как Теория вероятностей — ограничено своим набором возможных событий, то случайная величина Теория вероятностей принимает не более чем счетное число значений: Теория вероятностей

Распределением дискретной случайной величины Теория вероятностей назовем таблицу 2.

Теория вероятностей

Теория вероятностей

Таким образом, с точки зрения функционального анализа,
случайная величина представляет собой обычную числовую
функцию, заданную на пространстве элементарных исходов (событий) Теория вероятностей. Специфика теории вероятностей проявляется в том,
что на пространстве Теория вероятностей задана также вероятность Р.

Пример:

Два игрока играют в “орлянку” на следующих
условиях: если при подбрасывании монеты выпадает “орел”, то
первый игрок платит второму $1, если “решка”, то второй игрок
платит первому $2. Опишем случайную величину Теория вероятностей, равную выигрышу первого игрока в этой игре (при одном подбрасывании
монеты).

Решение:

Пространство элементарных исходов (событий) Теория вероятностей
состоит из двух исходов: Теория вероятностей– выпадение “орла” и Теория вероятностей– “решки”.
Теория вероятностейАлгебра событий насчитывает 4 события: Теория вероятностей

Это следует из аксиом Колмогорова. Предполагая, что монета
симметричная, найдем вероятности всех событий из множества
алгебры событий:

Теория вероятностей

Вероятностное пространство – определено.

Вероятностное пространство, как было определено выше, включает в себя пространство элементарных событий, Теория вероятностей— алгебру, вероятность Р – как меру, ограничение.

Случайная величина Теория вероятностей принимает значения: -1, если выпал
“герб” Теория вероятностей и 2, если выпала “цифра” Теория вероятностей

Теория вероятностей

Функция распределения случайной величины

Теория вероятностей

Функцией распределения (вероятностей) случайной величины Теория вероятностей называется функция F(x), значение которой в точке х равно вероятности события Теория вероятностейт.е. события, состоящего из тех и только тех элементарных исходов Теория вероятностей, для которых Теория вероятностей

Теория вероятностей

Или говорят, что значение функции распределения в точке х равно вероятности того, что случайная величина Теория вероятностей примет значение, меньшее х.

Свойства функции распределения

1.Функция F(x) является ограниченной, то есть ее значения
лежат в интервале от 0 до 1.

Теория вероятностей

2. Функция F(x) является неубывающей. Если Теория вероятностейто Теория вероятностей так как вероятность любого события неотрицательна.

3.Поскольку событие Теория вероятностейявляется невозможным, а событие Теория вероятностей– достоверным, то имеем

Теория вероятностей

3.Вероятность попадания случайной величины Теория вероятностей на отрезок Теория вероятностейопределяется формулой:

Теория вероятностей

Событие Теория вероятностейпри Теория вероятностейпредставляет собой объединение двух непересекающихся событий: Теория вероятностей– случайная величина Теория вероятностей приняла значение, меньшее Теория вероятностей, и Теория вероятностей– случайная величина приняла значение, лежащее в интервале Теория вероятностей.

Поэтому из аксиомы сложения получаем:

Теория вероятностей

Зная функцию распределения F(x), можно однозначно определить вероятность попадания случайной величины Теория вероятностей не только на интервал [x1, x2], но и в любое множество на прямой.

Итак, с любой случайной величиной связана ее функция распределения. После общего определения функции распределения случайной величины, перейдем к частным случаям – дискретной и непрерывной функциям распределения.

Дискретные случайные величины

Случайные величины могут быть дискретными т.е. принимать
только конечное или счетное множество определенных значений
(например, число очков при бросании игральной кости; число телефонных звонков, поступающих конкретному абоненту в течение суток). У таких величин F(x) имеет разрывы в точках,
соответствующих принимаемым значениям. Такие величины удобнее характеризовать указанием возможных значений и их вероятностей.

Теория вероятностей

Теория вероятностей

Теория вероятностей

Хотя случайная величина принимает только дискретные значения, ее функция распределения определена для любых х.

Например: F(-1) = 0, F(0) = 0, F(0.999) = 0, F(1.001) = 1/6,
F(3.5) = 3/6, F(7) = 1.

Дискретную случайную величину удобно характеризовать рядом распределения.

Теория вероятностей

Теория вероятностей – все возможные значения случайной величины.
Р – вероятности Теория вероятностейтого, что случайная величина
примет эти значения.

Теория вероятностей

Пример:

Дискретное распределение.
В неком обществе организована лотерея. Разыгрываются две вещи стоимостью по $10 и одна стоимостью $30. Составить закон распределения суммы чистого выигрыша для субъекта, который приобрел один билет за $1; всего продано 50 билетов.

Решение:

Искомая случайная величина X может принимать три значения: -1, (если субъект не выиграет, а фактически проиграет $1, уплаченный за билет); $9, $29. Первому результату благоприятны 47 случаев из 50, второму – 2 из 50, третьему – 1 из 50. Следовательно, вероятности, соответствующие этим случаям равны:
Р(Х=-1) = 47/50 = 094; P(X=9) = 2/50 = 0,04; P(X=29) = 1/50 = 0,02.
Закон распределения Х имеет вид:

Теория вероятностей

Виды дискретных функций распределения

Биноминальное распределение

Биноминальное распределение является распределением числа успехов Теория вероятностей в n испытаниях Бернулли с вероятностью успеха p и неудачи q = 1 – p.

Схема Бернулли

Рассмотрим последовательность независимых одинаковых испытаний: появление или не появление некоторого наблюдаемого события в каждом испытании не будет зависеть от исходов предыдущих испытаний. Это и есть схема Бернулли.

Опыт состоит в n-кратном повторении одинаковых испытаний, в каждом из которых может с вероятностью р наступить
некоторое событие (будем говорить в этом случае, что произошел “успех”) или с вероятностью q = 1 – p не наступить (произошла “неудача”). Результат каждого опыта можно записать в виде последовательности УНН…У, “У” – успех, “Н” – неудача.

Вычислим вероятность Теория вероятностей получить в n испытаниях ровно m успехов. Событие Теория вероятностей – в n испытаниях произошло ровно m успехов – состоит из тех элементарных исходов, в которых буква “У” появляется ровно m раз. Число таких исходов совпадает с числом сочетаний (не важен порядок выпадения “У”). С другой стороны, каждый элементарный исход, в котором буква “У” встречается ровно m раз имеет вероятность Теория вероятностейОкончательно получаем:

Теория вероятностей

Данное выражение носит также название биноминального
закона, поскольку Теория вероятностейможно получить как коэффициент при Теория вероятностей бинома Теория вероятностей

Теория вероятностей

Дискретная случайная величина распределена по биномиальному закону (рис. 8), если она принимает значения 0, 1, 2, …, n в соответствии с рядом распределения, представленным в табл. 6, где 0 < p, q < 1 и p + q = 1.

Теория вероятностей

Последний член разложения Теория вероятностей определяет вероятность наступления рассматриваемого события n раз в n независимых испытаниях; предпоследний член определяет вероятность наступления события рассматриваемого события n раз в n независимых испытаниях; предпоследний член определяет вероятность наступления события n-1 раз, а первый член определяет вероятность того, что событие не появится ни разу.

Основные характеристики распределения:

Теория вероятностей
Теория вероятностей

Задача:

Монета брошена 2 раза. Определить закон распределения случайной величины Х – числа выпадений герба.

Решение:

Вероятность появления герба при каждом бросании монеты равна Теория вероятностейследовательно, вероятность непоявления герба равна q=1-p=1-1/2=1/2.
При бросании монеты герб может появиться или 2 раза или 1 раз или совсем не появиться. Найдем вероятности этих событий по формуле Бернулли.

Теория вероятностей

Задача:

На зачете студент получил n = 4 задачи. Вероятность решить правильно каждую задачу p = 0,8. Определим ряд распределения и построим функцию распределения случайной
величины Теория вероятностей – числа правильно решенных задач.

Решение:

В данном случае мы имеем дело с биноминальным законом:

Теория вероятностей

Пуассоновское распределение

Распределение Пуассона моделирует случайную величину, представляющую собой число событий, произошедших за фиксированное время, при условии, что данные события происходят
с фиксированной средней интенсивностью и независимо друг от
друга (рис. 9).
Дискретная случайная величина Теория вероятностей распределена по закону
Пуассона и принимает целые неотрицательные значения с вероятностями, представленными рядом распределения (см. табл. 8).
Параметр пуассоновского распределения Теория вероятностейопределяет интенсивность поступления событий и определяется формулой: Теория вероятностейгде n – общее число испытаний, а Р – вероятность благоприятного исхода испытания.

Теория вероятностей

Распределение Пуассона носит также название закона редких событий, поскольку оно всегда появляется там, где производится большое число испытаний, в каждом из которых с малой
вероятностью происходит “редкое” событие. По закону Пуассона распределены, например, число вызовов, поступивших на телефонную станцию; число метеоритов, упавших в определенном
районе; число распавшихся нестабильных частиц и т. д. При условии

Теория вероятностей

закон распределения Пуассона является предельным случаем биномиального закона.

Основные характеристики распределения:

Теория вероятностей

Распределение Пуассона

Теория вероятностей

Формула Пуассона

Формула Пуассона применяется тогда, когда наряду с большим значением числа испытаний n “мала” вероятность успеха р.
Она относится к приближенным формулам для вычисления Теория вероятностейпри больших n. Формула Пуассона наиболее простая из
них.
Строго математически теорема Пуассона опирается на понятие схемы серий, здесь приведена “инженерная” интерпретация теоремы.

Теорема Пуассона:

Пусть число испытаний n в схеме Бернулли “велико”, а вероятность успеха р в одном испытании “мала”, причем “мало” также произведение Теория вероятностейТогда Теория вероятностей определяется по приближенной формуле (формула Пуассона)

Теория вероятностей

Доказательство. Формула Бернулли

Теория вероятностей

или с учетом обозначения Теория вероятностей

Теория вероятностей

При больших Теория вероятностейКроме того, если n – велико, то

Теория вероятностей

Поэтому приходим к доказываемой формуле.

Задача:

Завод отправил на базу 5000 доброкачественных изделий. Вероятность того, что в пути изделие повредится равно 0,0002. Найти вероятность того, что на базу прибудут 3 негодных изделия.

Решение:

По условию n =5000, р=0,0002, k=3.
Найдем Теория вероятностей

По формуле Пуассона искомая вероятность равна

Теория вероятностей

Потоком событий называют последовательность событий, которые наступают в случайные моменты времени. Например, поступление вызовов на станцию скорой помощи, прибытие самолетов в аэропорт, клиентов в пункт сервиса, покупателей в магазин и т.д.
Простейшим (Пуассоновским) – называется поток событий, обладающий следующими свойствами:

  • вероятность появления двух и более событий ничтожно
    мала по сравнению с вероятностью появления только
    одного события (ординарность);
  • отсутствие последствий: вероятность появления m событий на любом промежутке времени не зависит от того, появлялись ли события раньше;
  • стационарность: в одинаковые промежутки времени
    вероятность происхождения события одинакова.

Интенсивностью потока Теория вероятностей называют среднее число событий, которые появляются в единицу времени.

Задача:

Среднее число вызовов, поступающих на АТС в одну минуту, равно – 2. Найти вероятности того, что за 5 минут поступит: а) 2 вызова; б); в) не менее 2 вызовов.

Решение:

По условию Теория вероятностей

По формуле Пуассона:

Теория вероятностей

А) Вероятность, что за 5 минут поступят 2 вызова:

Теория вероятностей

Это событие практически невозможно.
Б) События «не поступило не одного вызова» и «поступил 1 вызов» – несовместны, поэтому по теореме сложения вероятностей: вероятность того, что за 5 минут поступят менее 2 вызовов, равна:

Теория вероятностей

Геометрическое распределение

Рассмотрим схему Бернулли. Пусть Теория вероятностей – число испытаний, которое необходимо провести, прежде чем появится первый успех. Тогда Теория вероятностей – дискретная случайная величина, принимающая
значения 0, 1, 2,…. n,….. Определим вероятность события Теория вероятностейТеория вероятностей

Очевидно, что Теория вероятностей = 0, если в первом же испытании произойдет
успех. Поэтому Теория вероятностейДалее, Теория вероятностей = 1 в том случае, когда в
первом испытании произошла неудача, а во втором – успех. Вероятность такого события – qp, то есть Теория вероятностей

Аналогично,Теория вероятностей, если в первых двух испытаниях произошли неудачи, а в третьем – успех: есть Теория вероятностей

Продолжая эту процедуру, получим ряд распределения:

Теория вероятностей

Случайная величина с таким рядом распределения называется распределенной по геометрическому закону (рис. 10).

Геометрическое распределение

Теория вероятностей

Задача:

Из орудия производится стрельба по цели до первого попадания. Вероятность попадания в цель р=0,6. Найти вероятность того, что попадание произойдет при третьем выстреле.

Решение:

По условию, р=0,6, q=0,4, k=3. Искомая вероятность определяется по формуле:

Теория вероятностей

Задача:

Вероятность поражения цели равна 0,6. Производится стрельба по мишени до первого попадания (число патронов не ограничено). Требуется составить ряд распределения числа сделанных выстрелов, найти математическое ожидание и
дисперсию этой случайной величины. Определить вероятность
того, что для поражения цели потребуется не более трёх патронов.

Решение:

Случайная величина X – число сделанных выстрелов – имеет геометрическое распределение с параметром p=0,6. Ряд распределения X имеет вид:

Теория вероятностей

Вероятность того, что для поражения цели потребуется не более
трёх патронов равна

Теория вероятностей

Гипергеометрическое распределение

Определение:

Дискретная случайная величина X имеет гипергеометрическое распределение, если она принимает значения
0, 1, 2,… min {n, M} с вероятностями

Теория вероятностей

где

Теория вероятностей

– натуральные числа.
N – общее количество объектов в генеральной совокупности;
M – количество объектов с определенным свойством в генеральной совокупности;
n – объем выборки;
m – количество деталей с определенным свойством.

Гипергеометрическое распределение имеет случайная величина X=m – число объектов, обладающих данным свойством, среди n объектов, случайно извлечённых (без возврата) из совокупности N объектов, M из которых обладают этим свойством.

Гипергеометрическое распределение широко используется в
практике статистического приёмочного контроля качества промышленной продукции, в задачах, связанных с организацией
выборочных обследований, и некоторых других областях.

Задача:

В национальной лотерее «6 из 45» денежные призы получают участники, угадавшие от трёх до шести чисел из случайно отобранных 6 из 45 (размер выигрыша увеличивается с увеличением числа угаданных чисел). Найти закон распределения, математическое ожидание и дисперсию случайной величины X – числа угаданных чисел среди случайно отобранных шести. Какова вероятность получения денежного приза?

Решение:

Случайная величина X – число угаданных чисел среди случайно отобранных шести – имеет гипергеометрическое распределение с параметрами n=6, N=45, M=6. Ряд распределения X, рассчитанный по формуле:

Теория вероятностей

Теория вероятностей

Распределения непрерывных случайных величин

Цель: изучить свойства непрерывных случайных величин и
явления, в которых они наблюдаются. Исследовать графики
плотности распределения непрерывных случайных величин.

Случайную величину назовем непрерывной, если ее функция распределения не имеет скачков и разрывов.
Непрерывной называется случайная величина Теория вероятностей, функцию распределения которой F(x) можно представить в виде:

Теория вероятностей

Функция p(x) называется плотностью распределения (вероятностей) случайной величины Теория вероятностей (рис. 11).

Практически все, реально встречающиеся плотности распределения являются непрерывными функциями, и, следовательно, для них p(x) = F’ (x), то есть производную от функции
распределения.

Теория вероятностей

Функция Теория вероятностей, обладающая вышеперечисленными свойствами, называется плотностью распределения случайной величиныТеория вероятностей .
Определим простейшие свойства плотности распределения р(x) непрерывной случайной величины:

  1. Плотность распределения непрерывной случайной вели-
    чины неотрицательная функция, поскольку она является производной от функции распределения, а функция распределения – неубывающая р(x)>0 для всех х.
  2. Площадь, целиком заключенная под всей кривой плотности распределения, равна единице.
    Теория вероятностей
  3. Вероятность попадания случайной величины на интервал
    [a; b] численно равна площади криволинейной трапеции
    (рис. 12):
Теория вероятностей

Теория вероятностей Следовательно, Теория вероятностей

Равномерное распределение

Равномерно распределенная на отрезке [a,b] случайная величина имеет функцию распределения (рис. 13):

Теория вероятностей

Функция равномерного распределения имеет вид:

Теория вероятностей

Плотность равномерного распределения представлена ниже (рис.14).

Теория вероятностей
Теория вероятностей
Теория вероятностей

Вероятность попадания равномерно распределенной случайной величины на интервал Теория вероятностейлежащий внутри отрезка (a,b), равна

Теория вероятностей

то есть пропорциональна длине этого интервала. Равномерное распределение реализует принцип геометрической вероятности при бросании точки на отрезок (a,b).

Теория вероятностей

Экспоненциальное распределение

Случайная величина подчиняется экспоненциальному (показательному) закону, если она имеет функцию распределения:

Теория вероятностей

Плотность экспоненциального распределения можно получить интегрированием функции распределения:

Теория вероятностей

Экспоненциально распределенная случайная величина может принимать только положительные значения. Экспоненциальному распределению подчинено время распада атомов различных элементов. При этом число Теория вероятностей носит название среднего времени распада. Кроме того, употребляется также число Теория вероятностей – называемое периодом полураспада.

Экспоненциально распределенная случайная величина Теория вероятностей
обладает свойством – отсутствием последействия. Это можно
трактовать как независимость поведения случайной величины в
момент времени Теория вероятностейот того, что с ней произошло до этого.

Основные характеристики распределения:

Теория вероятностей

Нормальное распределение

Случайная величина распределена по нормальному или гауссову закону, если она имеет плотность распределения

Теория вероятностей

Нормальное распределение зависит от двух параметров: где m – математическое ожидание или среднее значение нормального закона; Теория вероятностей среднее квадратичное отклонение (рис. 17).

Теория вероятностей

Графики для плотности распределения с одинаковым средним арифметическим и различными среднеквадратическими отклонениями. Плотность нормального распределения зависит от значений среднего арифметического и среднеквадратичного отклонения (рис. 18).

Теория вероятностей

Параметр m определяет положение центра нормальной плотности, а Теория вероятностей разброс относительно центра. Если m = 0,Теория вероятностей то такой нормальный закон называется стандартным и его функция распределения обозначается через Ф(х).

Основные характеристики распределения:

Теория вероятностей

Нормальное распределение возникает обычно в явлениях,
подверженных действию большого числа “малых” случайных
воздействий.

Распределение Вейбулла

Случайная величина распределена по закону Вейбулла, если она имеет плотность распределения

Теория вероятностей

Семейство распределений Вейбулла является двухпараметрическим и описывает положительные случайные величины (рис.19). Считается, что распределению Вейбулла подчиняются времена безотказной работы многих технических устройств. Если Теория вероятностей то распределение Вейбулла превращается в экспоненциальное распределение, а если Теория вероятностей – в так называемое распределение Релея.

Теория вероятностей

Гамма-распределение

Другим распределением, также достаточно хорошо описывающим времена безотказной работы различных технических устройств, является гамма-распределение с плотностью:

Теория вероятностей

где Теория вероятностей– гамма-функция Эйлера. Свойства гамма-функции:

Теория вероятностей

Для целых n. Функция и плотность Гамма-распределения представлены на рис. 20.

Теория вероятностей

Если Теория вероятностей– полуцелое, а Теория вероятностейто гамма-распределение
превращается в так называемое распределение Теория вероятностей(хи-квадрат).
Параметр k – называется в этом случае числом степеней свободы распределения Теория вероятностей.

Функции от случайной величины

Для дальнейшего развития понятия – случайная величина, следует перейти к понятию – функция от случайной величины. Пусть на вероятностном пространстве Теория вероятностей задана случайная величина Теория вероятностей Возьмем обычную (измеримую) числовую функцию g(x) числового аргумента х. Сопоставляя каждому элементарному исходу Теория вероятностей число Теория вероятностейпо формуле Теория вероятностейполучим новую случайную величину Теория вероятностей, которую
назовем функцией Теория вероятностей от случайной величины Теория вероятностей.

Функция Теория вероятностейот дискретной случайной величины также
является дискретной случайной величиной, поскольку она не
может принимать больше значений, чем случайная величина Теория вероятностей.
Ряд распределения случайной величины Теория вероятностей можно представить таблицей:

Теория вероятностей

При этом, если в верхней строке таблицы появляются одинаковые значения Теория вероятностейто соответствующие столбцы надо объединить в один, приписав им суммарную вероятность.

Функция Теория вероятностей от непрерывной случайной величины Теория вероятностей
может быть как непрерывной, так и дискретной (дискретной она
будет, например, если множество значений функции g(x) не более, чем счётно). Найдем функцию распределения Теория вероятностейпо заданной плотности Теория вероятностей По определению Теория вероятностей представляет собой вероятность события Теория вероятностейсостоящего из тех элементарных исходов Теория вероятностей, для которых Теория вероятностей В свою очередь, вероятность события Теория вероятностейможно определить, используя аксиому сложения вероятностей, “просуммировав” вероятности всех возможных значений y (промежуточная переменная) случайной величины Теория вероятностей, для которых g(y)<x. Так как вероятность случайной
величине Теория вероятностей принять значение в промежутке от y до y+dy приближенно равна Теория вероятностей то, заменяя сумму на интеграл, получаем

Теория вероятностей

Рассматривая любую функцию от случайной величины (как, впрочем, и любую другую функцию), необходимо четко представлять область определения этой функции. В случае
функции распределения область определения строится на тех
значениях х (то есть случайных величинах, полученных на множестве событий Теория вероятностей), для которых g(y) < x.

Числовые характеристики случайных величин

Цель: изучение количественных характеристик моментов высших порядков.

Основной целью статистического анализа является выяснение некоторых свойств изучаемой генеральной совокупности.
Если генеральная совокупность конечна, то наилучшая процедура – рассмотрение каждого ее элемента. Однако в большинстве интересных задач используются либо бесконечные генеральные совокупности, либо конечные, но трудно обозримые. В этой ситуации необходимо отобрать из генеральной совокупности подмножество из n элементов, называемое выборкой объема n, исследовать его свойства, а затем обобщить эти результаты на всю генеральную совокупность. Это обобщение называется статистическим выводом.

Генеральная совокупность (популяция) W – полный набор объектов w, с которыми связана данная проблема. Эти объекты могут быть людьми, животными, изделиями и так далее. С каждым объектом связана величина (или величины), называемая исследуемым признаком (Теория вероятностей).

Различные значения признака, наблюдающиеся у членов генеральной совокупности (или выборки), называются вариантами, а числа, показывающие сколько раз встречается каждый вариант – их частотами.

В данном определении предполагается дискретное изменения признака. Однако, если мы измеряем непрерывную величину, то точность измерения и количество измерений в единицу
времени тоже дадут некий дискретный набор.

Мы предполагаем, что измеряемый или исследуемый признак изменяется некоторым случайным образом. Произведя серию измерений, получим набор данных, которые, скорее всего, будут случайной выборкой из генеральной совокупности. Чтобы провести первичную обработку этой выборки, необходимо построить экспериментальное распределение данных по частотам или (если данные имеют явно непрерывный характер) по интервалам частот.

Пример:

При регистрации размеров продаваемой магазином женской верхней одежды были получены данные о 100 покупках (табл. 11).

Теория вероятностей
Теория вероятностей

Построим экспериментальное распределение данных по частотам. Для этого нужно определить количество признаков или интервалов частот, а затем подсчитать сколько вариантов в выборке соответствуют каждому интервалу, то есть частоту. Результаты этих расчетов для удобства заносят в таблицу, аналогичную табл. 12.

Теория вероятностей

По выборочным частотам строят гистограмму частот, которая характеризует опытное распределение (рис. 21).

Теория вероятностей

Другое представление получается, если значения на оси Y, соответствующие значениям на оси X, соединить ломаной кривой. Эта фигура называется полигоном частот или многоугольником распределения. Полигон частот дает информацию о законе распределения генеральной совокупности.

Теория вероятностей

Графическая иллюстрация статистических данных, геометрическая интерпретация отдельных вопросов статистики дает им наглядность, а в ряде случаев позволяет подвергнуть их анализу в наиболее простой и доступной форме. В примере использованы графики, полученные в результате обработки данных в Excel.

Под формой статистического распределения понимается форма его графика – полигона частот (рис. 22). Различают симметричные формы и несимметричные (асимметричные).

Распределение называется симметричным, если веса любых вариантов, равноотстоящих от среднего, равны между собой.

На практике такого совпадения для всех вариантов обычно нет и симметричными считаются распределения, в которых веса вариантов, равноотстоящих от среднего, отличаются незначительно. Полигон частот из пример похож на симметричное распределение, но совпадение приблизительное, поэтому данное
распределение является умеренно асимметричным.

Асимметричные распределения можно разбить на три вида:

  • умеренно асимметричные – распределения у которых часто-
    ты, находящиеся по одну сторону от наибольшей, больше
    (или меньше) частот, находящихся по другую сторону от
    наибольшей на таком же “расстоянии”;
  • крайне асимметричные – распределения, у которых частоты
    или все время возрастают, или все время убывают.
  • U-образные – частоты сначала убывают, а затем возрастают.
    Числовые характеристики статистического распределения

В качестве характеристик измеримого признака вместо исходных значений величин или таблиц их частот используют числовые характеристики, называемые также статистическими мерами.

  • Среднее арифметическое Теория вероятностей определяется по формуле
Теория вероятностей
Теория вероятностей
  • Среднее линейное отклонение d – среднее арифметическое
    абсолютных величин отклонений вариантов от их средней
    арифметической.
Теория вероятностей
  • Дисперсия D – среднее арифметическое квадратов отклонений вариантов от их средней:
Теория вероятностей
  • Среднее квадратичное отклонение – квадратный корень
    из дисперсии.

Каждая случайная величина характеризуется своей функцией распределения. С точки зрения наблюдателя, две случайные величины, имеющие одинаковые функции распределения,
неразличимы, несмотря на то, что они могут быть заданы на различных вероятностных пространствах и описывать разные явления.

Функция распределения или плотность распределения вероятностей являются наиболее полными характеристиками случайных величин. Однако во многих задачах оказывается трудно или даже невозможно полностью описать функцию распределения. В то же время, для решения многих задач достаточно знать лишь некоторые параметры, характеризующие случайную величину с той или иной точки зрения. Наиболее распространенными числовыми параметрами, получившими название числовых характеристик или моментов случайных величин, являются математическое ожидание и дисперсия или среднеквадратичное отклонение.

Математическое ожидание случайной величины

Математическим ожиданием (средним значением) Теория вероятностейдискретной случайной величины Теория вероятностей называется сумма произведений значений Теория вероятностей случайной величины на вероятности Теория вероятностейТеория вероятностей с которыми эти значения принимаются:

Теория вероятностей

Если рассматривать экспериментальные данные, аналогом
математического ожидания является среднее арифметическое
значение набора данных. Среднее арифметическое значение
приближается к математическому ожиданию при увеличении
числа испытаний.

Пример:

Найдем математическое ожидание случайной величины Теория вероятностей распределенной по биноминальному закону (число
успехов в n испытаниях Бернулли с вероятностью успеха р):

Теория вероятностей

Следовательно,Теория вероятностейПри выводе использовалось свойство вероятности – сумма вероятностей всех событий равна 1.

Пример:

Пусть Теория вероятностейимеет распределение Пуассона. Тогда
математическое ожидание этой величины равно:

Теория вероятностей

При больших Теория вероятностей

Таким образом, параметр Теория вероятностей пуассоновского распределения совпадает с математическим ожиданием.

Математическим ожиданием (средним значением) Теория вероятностей
непрерывной случайной величины Теория вероятностей называется интеграл

Теория вероятностей

где p(x) – плотность распределения. Случайная величина Теория вероятностей принимает значение х с вероятностью р(х)dx.

Пример:

Пусть непрерывная случайная величина Х задана
плотностью распределения Теория вероятностей в интервале Теория вероятностейвне этого интервала φ(х)=0. Найти математическое ожидание случайной величины Теория вероятностей

Воспользуемся формулой:

Теория вероятностей

Свойства математического ожидания

1.Если случайная величина Теория вероятностейпринимает всего одно значение С с вероятностью единица. Математическое ожидание постоянной величины равно этой постоянной:

Теория вероятностей

2. Пусть Теория вероятностей– случайная величина, выраженная линейной функцией, тогда математическое ожидание этой случайной величины равно:

Теория вероятностей

Рассматривая пример доказательства этого свойства для непрерывного случая:

Теория вероятностей

Аналогично свойство 2 доказывается для дискретной случайной величины (постоянные величины выносятся за знаки суммирования).

3. Пусть Теория вероятностей– случайная величина, которая является суммой
двух других величин: Теория вероятностей Тогда математическое ожидание
суммы двух случайных величин равно сумме математических
ожиданий каждой из этих величин:

Теория вероятностей

Рассмотрим доказательство этого свойства на примере дискретной случайной величины:

Теория вероятностей

4.Если Теория вероятностей и Теория вероятностей независимы, то математическое ожидание их произведений Теория вероятностей равно произведению их математических ожиданий:

Теория вероятностей

Дисперсия. Моменты высших порядков

Наряду со средним значением необходимо иметь число, характеризующее “разброс” случайной величины вокруг своего
среднего. Такой характеристикой обычно служит дисперсия.

Следует помнить, что существует много характеристик разброса, в частности, центральные моменты любого четного порядка.
Существует много случайных величин, которые имеют одинаковые математические ожидания, но различные возможные значения. Например, дискретные случайные величины, заданные законами распределения:

Теория вероятностей

Найдем математическое ожидание этих величин:

Теория вероятностей

Здесь математические ожидания обеих величин одинаковы, причем Х – близки к математическому ожиданию, а Y – далеки. Таким образом, зная математическое ожидание, нельзя судить о значениях случайной величины, ни о том, как рассеяны возможные значения случайной величины вокруг ее математического ожидания. Для оценки отклонения случайной величины от ее математического ожидания используют дисперсию.

Отклонением называют разность между случайной величиной и ее математическим ожиданием Х – М(Х).
На практике для оценки рассеяния случайной величины вокруг ее среднего значения пользуются величиной, названной дисперсией. Например, в артиллерии важно знать, насколько кучно лягут снаряды вокруг цели, которая должна быть поражена.

Дисперсия характеризует разброс значений случайной величины вокруг ее математического ожидания.

Дисперсия Теория вероятностейдискретной случайной величины Теория вероятностей определяется формулой

Теория вероятностей

На первый взгляд может показаться, что достаточно вычислить все возможные значения отклонения и найти их среднее значение, но такой путь ничего не даст, поскольку отклонения бывают отрицательными и их среднее значение равно 0. Поэтому лучше взять абсолютные значения отклонений или их квадраты.
Пусть случайная величина задана законом распределения:

Теория вероятностей

Тогда квадрат отклонения имеет следующий закон распределения:

Теория вероятностей

По определению дисперсии,

Теория вероятностей

Из определения следует, что дисперсия случайной величины есть неслучайная (постоянная) величина.

Вторым (начальным) моментом Теория вероятностейслучайной величины Теория вероятностей
называется математическое ожидание квадрата
Теория вероятностей

Теория вероятностей

Дисперсией непрерывной случайной величины называют математическое ожидание квадрата ее отклонения. Если возможные значения Х принадлежат отрезку [a,b], то

Теория вероятностей

Среднее квадратичное отклонение непрерывной случайной
величины определяется равенством:

Теория вероятностей

Дисперсия Теория вероятностейпредставляет собой второй момент случайной величины Теория вероятностей , из которой вычтено ее математическое ожидание
Теория вероятностей то есть центрированной (имеющей нулевое математическое
ожидание) случайной величины Теория вероятностей

Поэтому дисперсию иногда называют вторым центральным моментом.

Свойства дисперсии

Определим некоторые свойства дисперсии (без вывода).

1.Если случайная величина Теория вероятностей с вероятностью 1 принимает одно и тоже постоянное значение С, то из свойства 1 математического ожидания Теория вероятностей получаем

Теория вероятностей

2.Постоянный множитель можно выносить за знак дисперсии,
возведя его в квадрат:

Теория вероятностей

3.Дисперсия суммы случайной величины и постоянной равна:

Теория вероятностей

4.Дисперсия суммы двух независимых случайных величин Теория вероятностей и Теория вероятностей
равна сумме дисперсий каждой из этих величин:

Теория вероятностей

Дисперсия Теория вероятностей имеет размерность квадрата размерности случайной величины Теория вероятностей. Для практических целей удобно иметь
меру разброса, размерность которой совпадает с размерностью Теория вероятностей . В качестве такой меры естественно использовать Теория вероятностей,
которую называют средним квадратичным отклонением случайной величины Теория вероятностей (или стандартным отклонением).

Моменты высших порядков

В теории вероятностей и математической статистике, помимо математического ожидания и дисперсии, используются и другие числовые характеристики случайных величин. В первую очередь это начальные и центральные моменты.

Начальным моментом k-го порядка случайной величины называется математическое ожидание k-й степени случайной величины x , то есть:

Теория вероятностей

Заметим, что математическое ожидание случайной величины – начальный момент первого порядка, Теория вероятностей

Центральным моментом k-го порядка случайной величины x называется величина Теория вероятностейопределяемая формулой Теория вероятностей

Теория вероятностей

Дисперсия является центральным моментом второго порядка.
Существуют формулы, позволяющие выразить центральные моменты случайной величины через ее начальные моменты,
например:

Теория вероятностей

где Теория вероятностей – центральный момент третьего порядка.
Если плотность распределения вероятностей непрерывной случайной величины симметрична относительно прямой x = Mx,
то все ее центральные моменты нечетного порядка равны нулю.

Асимметрия

В теории вероятностей и в математической статистике в качестве меры асимметрии распределения является коэффициент
асимметрии, который определяется формулой:

Теория вероятностей

Эксцесс

Нормальное распределение наиболее часто используется в
теории вероятностей и в математической статистике, поэтому
график плотности вероятностей нормального распределения
стал своего рода эталоном, с которым сравнивают другие
распределения. Одним из параметров, определяющих отличие
распределения случайной величины x от нормального распределения, является эксцесс.

Эксцесс случайной величины определяется равенством:

У нормального распределения, естественно, Kэ = 0.
Если Kэ > 0, то это означает, что график плотности вероятностей
Теория вероятностей сильнее “заострен”, чем у нормального распределения, если же Kэ < 0, то “заостренность” графика Теория вероятностей меньше, чем у нормального распределения.

Предельные теоремы теории вероятностей

Цель: Познакомить студентов с законом больших чисел и
областями его применения.

Существует много явлений и ситуаций, когда при проведении подобных испытаний многократно наблюдается одна и та же случайная величина. Практика изучения случайных явлений показывает, что хотя результаты отдельных наблюдений, даже проведенных в одинаковых условиях, могут сильно отличаться, в то же время средние результаты для достаточно большого числа наблюдений устойчивы и слабо зависят от результатов отдельных наблюдений.

Устойчивость испытаний состоит в том, что особенности каждого отдельного случайного явления почти не сказываются на среднем результате большой массы подобных явлений, а характеристики случайных событий и случайных величин, наблюдаемых в испытаниях, при неограниченном увеличении числа испытаний становятся практически не случайными.

Теоретическим обоснованием этого замечательного
свойства случайных явлений является закон больших чисел.
Теоремы закона больших чисел устанавливают зависимость
между случайностью и необходимостью. Названием «закон
больших чисел»
объединена группа теорем, устанавливающих
устойчивость средних результатов большого количества
случайных явлений и объясняющих причину этой устойчивости.
Простейшая форма закона больших чисел и исторически
первая теорема этого раздела – теорема Бернулли. Эта теорема
устанавливает связь между вероятностью появления события и
его относительной частотой появления и позволяет при этом
предсказать, какой примерно будет эта частота в n испытаниях.

Теорема Бернулли

Теорема Бернулли:

Если вероятность события А
одинакова во всех испытаниях и равна Р(А), то при достаточно
большом n для произвольного e >0 справедливо неравенство:

Теория вероятностей

а при переходе к пределу получаем:

Теория вероятностей

Из теоремы видно, что с увеличением числа испытаний W – частота события А стремится к вероятности события Р(А) и перестает быть случайной.

Иногда (при решении практических задач) требуется оценить вероятность того, что отклонение количества благоприятных исходов испытания m в общем числе испытаний n от ожидаемого результата не превысит определенного числа Теория вероятностей

Для данной оценки неравенство переписывают в виде:

Теория вероятностей

Пример:

Монету подбрасывают 1000 раз. Оценить вероятность отклонения частоты появления герба от вероятности его появления меньше чем на Теория вероятностей

Решение:

Вероятность появления герба р=0,5, тогда q = 1-
0,5=0,5; n= 1000, Теория вероятностей

Теория вероятностей

Раскрывая модуль и решая неравенство относительно m, получим: 400<m<600. Итак, вероятность небольшого отклонения частоты выпадения герба (±100) от его классической вероятности 0,5 равна 0,975. Значит, вероятность большего отклонения крайне мала и равна 0,025.

Теорема Пуассона

Теорема Пуассона утверждает, что частота события в серии независимых испытаний стремится к среднему арифметическому его вероятностей и перестает быть случайной.

Теория вероятностей

При большом количестве испытаний вычисления по формуле Бернулли становятся затруднительными. Однако в ряде случаев их можно заменить более простыми асимптотическими формулами, например формулой Пуассона (когда npq<9). Если производится n независимых опытов и вероятность появления события в каждом опыте равна рi, то при увеличении n, 0 частота события Теория вероятностей, где Теория вероятностей и стремится к среднему арифметическому вероятностей Теория вероятностей события А.

Пример:

В здании 1000 лампочек. Вероятность выхода из строя одной лампочки в течение года p =0.003. Найдем вероятность того, что в течение одного года выйдет из строя более трех ламп. Выполним вычисления используя формулу Бернулли и по теореме Пуассона.

Решение:

Для вычисления вероятности используем формулу Пуассона:

Теория вероятностей
Теория вероятностей

Параметр лямбда считаем по формуле:

Теория вероятностей
Теория вероятностей

Ответ: вероятность того, что в течение года выйдет из строя больше трех лампочек 0,37.
Предельные теоремы теории вероятностей объясняют природу устойчивости частоты появлений события. Природа эта состоит в том, что предельным распределением числа появлений события при неограниченном возрастании числа испытаний (если вероятность события во всех испытаниях одинакова) является нормальное распределение.

Центральная предельная теорема

Центральная предельная теорема объясняет широкое
распространение нормального закона распределения. Теорема
утверждает, что всегда, когда случайная величина образуется в
результате сложения большого числа независимых случайных
величин с конечными дисперсиями, закон распределения этой
случайной величины оказывается практически нормальным законом.

Закон больших чисел утверждает, что при большом числе испытаний среднее арифметическое случайной величины
стремится к математическому ожиданию и перестает быть
случайным.

Теорема Ляпунова

Теорема Ляпунова объясняет широкое распространение нормального закона распределения и поясняет механизм его
образования. Теорема позволяет утверждать, что всегда, когда
случайная величина образуется в результате сложения большого
числа независимых случайных величин, дисперсии которых
малы по сравнению с дисперсией суммы, закон распределения
этой случайной величины оказывается практически нормальным
законом. А поскольку случайные величины всегда порождаются
бесконечным количеством причин и чаще всего ни одна из них
не имеет дисперсии, сравнимой с дисперсией самой случайной
величины, то большинство встречающихся в практике
случайных величин подчинено нормальному закону
распределения.
В основе качественных и количественных утверждений
закона больших чисел лежит неравенство Чебышева. Оно
определяет верхнюю границу вероятности того, что отклонение
значения случайной величины от ее математического ожидания
больше некоторого заданного числа. Замечательно, что
неравенство Чебышева дает оценку вероятности события Теория вероятностей для случайной величины, распределение которой
неизвестно, известны лишь ее математическое ожидание и
дисперсия.

Схема описания всех этих явлений с единых вероятностных позиций выглядит следующим образом: имеется последовательность независимых, одинаково распределенных случайных величин (генеральная совокупность — в терминах математической статистики) и из нее образуется среднее арифметическое первых n членов (выборка из генеральной совокупности). Спрашивается, как будет вести себя это среднее арифметическое, если n велико? Оказывается, что при большом n оно теряет свойство случайности и приближается к математическому ожиданию. Данный факт мы априорно используем уже давно, перейдя к основным понятиям математической статистики. Этот факт носит название закона больших чисел.

Исторически закон больших чисел доказывается, опираясь на неравенство Чебышева, которое является родоначальником
многих других неравенств, широко применяемых в современной
теории вероятностей.

Дальнейшее уточнение закона больших чисел происходило в двух направлениях. Первое связано с динамикой поведения средних арифметических. К основным результатам этого направления следует отнести усиленный закон больших чисел и закон повторного логарифма, полученные А.Н.Колмогоровым. Исходным пунктом второго направления, называемого иногда центральной предельной проблемой, являются теоремы Муавра-Лапласа.

Теорема Муавра-Лапласа

Локальная теорема: Если в схеме Бернулли число испытаний n “велико”, то для всех m справедлива приближенная формула (локальная формула Муавра-Лапласа)

Теория вероятностей

Интегральная теорема: Если в схеме Бернулли число
испытаний n “велико”, то для вероятности Теория вероятностей того, что
число “успехов” Теория вероятностей заключено в пределах от Теория вероятностей до Теория вероятностейсправедливо приближенное соотношение интегральная формула Муавра-Лапласа)

Теория вероятностей

Теория вероятностей

— функция стандартного нормального распределения.

Решение центральной предельной проблемы позволило
описать класс всех распределений, которые могут выступать в
качестве предельных для функций распределения сумм
независимых случайных величин в том случае, когда вкладом
каждого слагаемого можно пренебречь, найти необходимые и
достаточные условия сходимости к каждому распределению
этого класса, оценить скорость сходимости скорость сходимости, как и сама вероятность сходимости ряда наблюдений к некоторой постоянной величине — является важным критерием устойчивости случайного процесса).

Неравенство Чебышёва

Рассмотрим случайную величину Теория вероятностей, имеющую дисперсию Теория вероятностейДисперсия является показателем разброса Теория вероятностей вокруг
математического ожидания Теория вероятностей

Однако с точки зрения исследователя, разброс естественнее характеризовать вероятностью Теория вероятностейслучайной величины Теория вероятностей от Теория вероятностей на величину, большую некоторого заданного Теория вероятностей

Следующее неравенство позволяет оценить эту вероятность через
дисперсию Теория вероятностей

Неравенство Чебышёва. Для каждой случайной величины Теория вероятностей, имеющей дисперсию Теория вероятностейпри любом Теория вероятностей справедливо
неравенство

Теория вероятностей

Доказательство для непрерывной случайной величины Теория вероятностей с плотностью распределения р(х). По определению

Теория вероятностей

Поскольку подынтегральное выражение неотрицательно, то при
уменьшении области интегрирования интеграл может только
уменьшиться. Поэтому

Теория вероятностей

Учитывая теперь, что Теория вероятностей если Теория вероятностейполучаем

Теория вероятностей

Последний интеграл представляет собой вероятность события Теория вероятностей и, значит

Теория вероятностей

откуда и следует неравенство Чебышёва. Аналогично неравенство
доказывается и для дискретного случая, при этом нужно только
заменить интеграл на сумму.
Ясно, что применять неравенство Чебышёва имеет смысл
только тогда, когда Теория вероятностейв противном случае оно дает
тривиальную оценку. Неравенство Чебышёва дает грубую
оценку
того, что исследуемая величина примет некоторое
значение в заданном диапазоне.
Рассмотрим последовательность Теория вероятностейнезависимых одинаково распределенных случайных величин (так как случайные величины Теория вероятностейодинаково распределены, то все их числовые характеристики, в частности математические ожидания и дисперсии, равны между собой). Эта последовательность удовлетворяет (слабому) закону больших чисел, если для некоторого а и любого Теория вероятностей

Теория вероятностей

Иными словами, выполнение закона больших чисел отражает предельную устойчивость средних арифметических случайных величин: при большом числе испытаний они практически перестают быть случайными и с большой степенью достоверности могут быть предсказаны.

Иногда вместо выражения “последовательность Теория вероятностей… удовлетворяет закону больших чисел” говорят “среднее арифметическое случайных величин Теория вероятностейсходится по
вероятности к некоторой предельной постоянной а”.

Теорема (закон больших чисел)

Если последовательность Теория вероятностей независимых
одинаково распределенных случайных величин такова, что
существуют
Теория вероятностейто для любого Теория вероятностей.

Теория вероятностей

Доказательство является элементарным следствием неравенства Чебышёва: по свойствам математического ожидания и дисперсии

Теория вероятностей

Воспользовавшись теперь неравенством Чебышёва, получаем, что для любого Теория вероятностей

Теория вероятностей

Таким образом, показано, что для последовательности
Теория вероятностей выполняется закон больших чисел, причем
постоянная а совпадает с математическим ожиданием Теория вероятностей

Дополнительные лекции:

  1. Случайные события и их вероятности
  2. Случайные величины
  3. Функции случайных величин
  4. Числовые характеристики случайных величин
  5. Законы больших чисел
  6. Статистические оценки
  7. Статистическая проверка гипотез
  8. Статистическое исследование зависимостей
  9. Теории игр
  10. Вероятность события
  11. Теорема умножения вероятностей
  12. Формула полной вероятности
  13. Теорема о повторении опытов
  14. Нормальный закон распределения
  15. Определение законов распределения случайных величин на основе опытных данных
  16. Системы случайных величин
  17. Нормальный закон распределения для системы случайных величин
  18. Вероятностное пространство
  19. Классическое определение вероятности
  20. Геометрическая вероятность
  21. Условная вероятность
  22. Схема Бернулли
  23. Многомерные случайные величины
  24. Предельные теоремы теории вероятностей
  25. Оценки неизвестных параметров
  26. Генеральная совокупность

Задачи по теории вероятностей с решениями

В разделе размещены подробно разобранные задачи по теории вероятностей и математической статистике, перед решением которых излагается теория в сжатом виде, где содержаться основные формулы разбираемой темы. Примеры упорядочены в соответствии с содержанием курса теории вероятностей в ВУЗах. Задачи будут полезны для студентов экономических и технических специальностей.

О платной помощи студентам с учебой можно почитать на странице Как заказать решение задач по теории вероятностей и математической статистике

    Случайные события

      Комбинаторика — основные формулы

      В краткой форме раскрыты основные понятия — перестановки,
      размещения, сочетания, и приведены основные формулы комбинаторики. После каждой формулы приводится пример решения задачи.

      Классическая вероятность

      В краткой форме рассмотрено понятие вероятности случайного события и
      дано классическое определение вероятности. На подробном примере решения задач о бросании игральных костей и извлечении шаров из урны раскрыто одно из важнейших
      определений теории вероятностей.

      Геометрическое определение вероятности

      Изложено геометрическое определение вероятности и приведен пример решения
      широко известной задачи о встрече.

      Статистическое определение вероятности

      Приведено определение относительной частоты и изложено статистическое определение вероятности. Приведены примеры решения задач.

    Сложение и умножение вероятностей

    Повторение испытаний

      Формула Бернулли

      Страница содержит краткое изложение теории повторных независимых
      испытаний и приведен пример решения задачи на формулу Бернулли.

      Локальная теорема Муавра — Лапласа

      Изложены краткие теоретические сведения по локальной
      теореме Муавра — Лапласа, рассмотрены условия ее применимости, а также приведен пример решения задачи.

      Интегральная теорема Лапласа

      В краткой форме раскрыто содержание интегральной теоремы
      Муавра — Лапласа, рассматриваются условия ее применимости. Приводится образец задачи с подробным решением.

      Следствия интегральной теоремы Муавра-Лапласа

      Рассматривается на подробном примере решения задачи отклонение
      относительной частоты от постоянной вероятности в независимых испытаниях.

      Наивероятнейшее число

      Рассматривается на подробном примере решения задачи понятие наивероятнейшего
      числа и как найти вероятность появления наивероятнейшего числа.

      Формула Пуассона

      Рассматривается формула Пуассона и условие ее
      применимости. Приведен пример решения задачи теории вероятностей на формулу Пуассона.

    Случайные величины

      Дискретная случайная величина

      На странице рассмотрен закон распределения дискретной случайной
      величины, изложена схема вычислений математического ожидания и дисперсии одномерной дискретной случайной величины. Приведен пример решения задачи с построением
      функции распределения.

      Непрерывная случайная величина

      На странице рассматривается непрерывная случайная величина,
      ее функция распределения и плотность распределения. Перечислены свойства плотности вероятности, приведены формулы для вычисления
      математического ожидания и дисперсии НСВ. Даны образцы решения задач на расчет характеристик и построение графиков функции распределения
      и плотности распределения непрерывной случайной величины.

      Функции распределения случайных величин

      Рассматриваются функции распределения дискретных и непрерывных случайных величин — определение, свойства, графики функции распределения.

      Плотность распределения вероятностей

      Рассматривается плотность распределения вероятностей непрерывной случайной величины — определение, свойства, графики плотности распределения вероятностей.

      Математическое ожидание и его свойства

      Рассматривается математическое ожидание случайной величины — одно из важнейших
      понятий теории вероятностей. Приведены примеры решения задач. Кратко излагается что такое математическое ожидание и каковы его свойства.
      Математическое ожидание суммы и произведения случайных величин.

      Дисперсия и ее свойства

      Излагается определение дисперсии случайной величины и
      среднего квадратического отклонения, которые являются важными понятиями в курсе теории вероятностей и математической статистики. Описываются свойства дисперсии —
      дисперсия суммы случайных величин, дисперсия постоянной величины.

      Начальные и центральные моменты случайной величины

      На странице рассмотрены определения начальных и центральных моментов случайной величины, приведены формулы их взаимосвязи. Даны понятия об асимметрии и эксцессе непрерывной и дискретной случайных величин.

      Мода и медиана случайной величины. Квантиль уровня случайной величины

      На странице рассматривается мода и медиана случайной величины в теории вероятностей, вычисление моды и медианы на примере непрерывных и дискретных случайных величин. Изложено понятие квантилей и процентных точек СВ. Приведены примеры.

      Функции одного и двух случайных аргументов

      Излагается понятие функции одного и двух случайных аргументов, а также понятие композиции случайных величин. Приведены примеры.

    Закон больших чисел

      Неравенство Маркова

      На странице рассматривается неравенство Маркова (лемма Чебышева) и приведены примеры решения задач.

      Неравенство Чебышева

      Рассмотрен пример решения задачи
      на закон больших чисел (неравенство Чебышева).

    Основные законы распределения

      Биномиальное распределение

      Страница содержит определение биномиального закона
      распределения, формулу для вычисления математического ожидания и дисперсии случайной величины, распределенной по биномиальному закону.
      Приведен пример решения задачи.

      Геометрическое распределение

      Излагается понятие геометрического закона распределения
      дискретной случайной величины и рассматривается пример решения задачи. Приведены формулы математического ожидания и дисперсии случайной величины, распределенной по
      геометрическому закону.

      Закон распределения Пуассона

      Излагается понятие пуассоновского закона распределения
      дискретной случайной величины и рассматривается пример решения задачи. Приведены формулы характеристик распределения.

      Простейший поток событий

      Простейший поток событий и его свойства — теория и примеры решения задач.

      Гипергеометрическое распределение

      Рассматривается гипергеометрическое распределение, моделирующее количество
      удачных выборок без возвращения из конечной совокупности. Страница содержит определение гипергеометрического закона распределения, формулы для вычисления математического
      ожидания и дисперсии случайной величины, распределенной по гипергеометрическому закону, а также образец решения задачи.

      Нормальный закон распределения

      Рассматривается нормальное распределение случайной величины —
      его плотность и функция распределения, а также правило трех сигм. Приведены необходимые теоретические сведения и
      образцы решения задач на нормальный закон распределения.

      Показательный закон распределения

      Рассмотрен экспоненциальный (показательный) закон распределения
      случайной величины, приведены необходимые теоретические сведения и примеры решения задач. Излагаются понятия математического ожидания, дисперсии и параметра
      показательного закона распределения.

      Равномерное распределение

      Излагается понятие закона равномерного распределения случайной величины.
      Приведены необходимые теоретические сведения, рассмотрены математическое ожидание и дисперсия случайной величины, распределенной равномерно и
      приведен пример решения задачи на эту тему.

    Двумерная случайная величина

      Дискретная двумерная случайная величина

      Рассматривается
      двумерная дискретная случайная величина и ее числовые характеристики — математическое ожидание, дисперсия, среднее квадратическое отклонение,
      а также условные законы распределения, коэффициенты ковариации и корреляции.

      Непрерывная двумерная случайная величина

      Рассматривается
      двумерная непрерывная случайная величина.
      Функция распределения двумерной СВ и ее свойства.
      Плотность распределения двумерной СВ и ее свойства, а также условные и безусловные законы распределения.

    Выборочный метод

    Корреляционный и регрессионный анализ

      Парная линейная регрессия и метод наименьших квадратов (МНК)

      На странице даны образцы решения задач на построение парной линейной регрессии методом наименьших квадратов (МНК). Решение задач предваряют краткие теоретические сведения, где подробно рассматривается соответствующая система нормальных уравнений и следующие из нее формулы для нахождения параметров парной линейной регрессии.

      Линейный коэффициент корреляции

      Рассмотрены формула и смысл коэффициента линейной корреляции. Страница содержит типовой пример по расчету выборочного линейного коэффициента корреляции и проверке его значимости.

      Нелинейные модели парной регрессии

      Рассматриваются нелинейные уравнения парной регрессии — степенные, гиперболические, показательные и параболические. Приведены соответствующие системы нормальных уравнений и решены задачи, в которых, помимо параметров уравнения, рассчитаны для каждого вида модели коэффициенты детерминации и эластичности.

      Коэффициент ранговой корреляции Спирмена

      Содержится краткая теория и пример решения задачи на ранговую корреляцию. Дано понятие ранговой корреляции, показан расчет коэффициента ранговой корреляции Спирмена.

      Коэффициент ранговой корреляции Кендалла

      На странице рассмотрено применение ранговой корреляции и коэффициента ранговой корреляции Кендалла в статистике. Приведена краткая теория, а также задача с примером расчета коэффициента Кендалла с проверкой гипотезы о его значимости.

    Статистическая проверка статистических гипотез

      Проверка гипотезы о равенстве средних

      На примере решения задачи подробно рассматривается проверка гипотезы о
      равенстве средних значений, понятия нулевой и конкурирующей гипотезы.

      Проверка гипотезы о нормальном распределении

      Рассматривается проверка гипотезы о распределении генеральной совокупности по нормальному закону. На примере решения задачи вычислены теоретические частоты нормального распределения и осуществлена проверка гипотезы о нормальном распределении СВ с помощью критерия Пирсона.

      Проверка гипотезы о показательном распределении

      Рассматривается проверка гипотезы о распределении генеральной совокупности по экспоненциальному (показательному) закону. На примере решения задачи вычислены теоретические частоты показательного распределения и осуществлена проверка гипотезы об экспоненциальном распределении СВ с помощью критерия Пирсона.

      Проверка гипотезы о распределении по закону Пуассона

      Рассматривается проверка гипотезы о распределении генеральной совокупности
      по закону Пуассона. Показано вычисление теоретических частот и применение критерия Пирсона на примере решения задачи.

    Дисперсионный анализ

      Однофакторный дисперсионный анализ

      Даны краткие теоретические сведения о дисперсионном анализе.
      Рассмотрен пример решения задачи на однофакторный дисперсионный анализ с вычислениями факторной и случайной дисперсии.

    Статистические таблицы


Большое количество типовых задач по теории вероятностей для самостоятельного решения.

  • Часть первая: 108 типовых задач по теории вероятностей
  • Часть вторая: 137 типовых задач по теории вероятностей
  • Часть третья: 114 типовых задач по теории вероятностей
  • Часть четвертая: 89 типовых задач по теории вероятностей

Понравилась статья? Поделить с друзьями:
  • Задачи по теме конус егэ с ответами
  • Задачи по теме кинематика егэ
  • Задачи по стереометрии на егэ презентация
  • Задачи по стереометрии егэ с решениями профильный уровень
  • Задачи по стереометрии егэ с решениями базовый уровень